Вы находитесь на странице: 1из 200

• Simple Harmonic Motion

• Waves
• Sound Wave
Copyright © reserved with Motion Edu. Pvt. Ltd. and Publications
All rights reserved. No part of this work herein should be reproduced or used either
graphically, electronically, mechanically or by recording, photocopying, taping, web
distributing or by storing in any form and retrieving without the prior written permission of
the publisher. Anybody violating this is liable to be legally prosecuted.

Corporate Head Office


394 - Rajeev Gandhi Nagar Kota, (Raj.)
Ph. No. : 08003899588, 0744-2209671
IVRS No : 0744-2439051, 52, 53,
www. motioniitjee.com , info@motioniitjee.com
CONTENTS
SIMPLE HARMONIC MOTION
THEORY AND EXERCISE BOOKLET
S.NO. TOPIC PAGE NO.

 THEORY WITH SOLVED EXAMPLES ............................................................. 05 – 37


 EXERCISE - I ................................................................................................... 38 – 47

 EXERCISE - II .................................................................................................. 48 – 59
 EXERCISE - III ................................................................................................. 60 – 68
 EXERCISE - IV ................................................................................................ 69 – 80
 ANSWER KEY ................................................................................................ 81 – 82

WAVES
THEORY AND EXERCISE BOOKLET
S.NO. TOPIC PAGE NO.

 THEORY WITH SOLVED EXAMPLES ............................................................. 83 – 114


 EXERCISE - I ................................................................................................... 115 – 121
 EXERCISE - II .................................................................................................. 112 – 128

 EXERCISE - III ................................................................................................. 129 – 136


 EXERCISE - IV ................................................................................................ 137 – 141
 ANSWER KEY ................................................................................................ 142 – 143

SOUND WAVE
THEORY AND EXERCISE BOOKLET
S.NO. TOPIC PAGE NO.

 THEORY WITH SOLVED EXAMPLES ............................................................. 144 – 170


 EXERCISE - I ................................................................................................... 171 – 176
 EXERCISE - II .................................................................................................. 177 – 184

 EXERCISE - III ................................................................................................. 185 – 190


 EXERCISE - IV ................................................................................................ 191 – 198
 ANSWER KEY ................................................................................................ 199 – 200
Page # 4 S.H.M.

SYLLABUS

• Simple Harmonic Motion

Linear and angular simple harmonic motions.

• WAVES & SOUND WAVE

Wave motion (plane waves only), longitudinal and transverse waves, Superposition
of waves; progressive and stationary waves.

Vibration of strings and air columns. Resonance; Beats; Speed of sound in gases
Doppler effect (in sound). Vibration of strings and air columns. Resonance; Beats;
Speed of sound in gases; Doppler effect (in sound).

Corporate Head Office : Motion Education Pvt. Ltd., 394 - Rajeev Gandhi Nagar, Kota-5 (Raj.)
S.H.M. Page # 5

SIMPLE HARMONIC MOTION


1. PERIODIC MOTION :
When a body or a moving particle repeats its motion along a definite path after regular
intervals of time its motion is said to be Periodic Motion and interval of time is called time
period (T). The path of periodic motion may be linear, circular, elliptical or any other curve.
For example rotation of earth around the sun.
2. OSCILLATORY MOTION :
To and fro type of motion is called Oscillatory Motion. A particle has oscillatory motion when
it moves about stable equilibrium position. It need not be periodic and need not have fixed
extreme positions.
The oscillatory motions in which energy is conserved are also periodic.For example motion of
pendulum of a wall clock.
The force / torque (directed towards equillibrium point) acting in oscillatory motion is called
restoring force/torque Damped Oscillations are those in which energy consumed due to
some resistive forces and hence total mechanical energy decreases and after some time
oscillation will stop.

Oscillatory Equation : Consider a particle free to move on x-axis is being acted upon by a
force given by
F = – kxn
Above equation is called oscillatory equation. Here k is a positive constant and x is the
displacement from mean position
Now following cases are possible depending on the value of n.
(i) If n is an even interger (0, 2, 4......... etc)n force is always along negative x-axis whether x is
positive or negative Hence, the motion of the particle is not oscillatory. If the particle is
released from any position on the x-axis (except x = 0) a force in – ve direction of x-axis acts
on it and it moves rectilinearly along – ve x axis.
(ii) If n is an odd integer (1, 3, 5 ........... etc), force is along – ve x-axis for x > 0 and along +ve
x-axis for x < 0 and zero for x = 0. Thus the particle will oscillate about stable equillibrium
position x = 0. The force in this case is called the restoring force.
If n = 1 i.e., F = – kx the motion is said to be SHM (Simple Harmonic Motion)
If the restoring force / torque acting on the body in oscillatory motion is directly proportional
to the displacement of body / particle w.r.t. mean position and is always directed towards
equillibrium position then the motion is called Simple Harmonic motion. It is the simplest form
of oscillatory motion.

3. TYPES OF SHM :
(a) Linear SHM : When a particle moves to and fro about an equilibrium point, along a
straight line here A and B are extreme positions and M is mean position so AM = MB =
Amplitude.
M
A B
(b) Angular SHM : When body/particle is free to rotate about a given axis and executing
angular oscillations.

4. ANALYSIS OF MOTION IN LINEAR SHM :


When the particle is moved away from the mean position or equillibrium position and released,
a force (–kx) comes into play to pull it back towards mean position. By the time it gets at
mean position it has picked up some kinetic energy and so it overshoots, stopping some
where on the other side and it is again pulled back towards the mean position.

: 0744-2209671, 08003899588 | url : www.motioniitjee.com, : info@motioniitjee.com


Page # 6 S.H.M.

It is necessary to study the change in speed and acceleration of particle during SHM. Let us
consider a particle whose position is x = 0 at t = 0 and v = v0. Then we divide the motion of
particle in one time period in four parts.
extreme Mean Extreme
position Position position

v=v0
A B
v=0 v=0
C

Amplitude
x=0 t=0
x
(A) from A to B (B) from B to A (C) from A to C (D) from C to A

NOTE :In the figure shown, path of the particle is a straight line.
(1) Motion of a particle from A to B :
Initially the particle is at A (mean position) and is moving towards +ve x direction with speed
v0. As the particle is moving towards B, force acting on it towards A is increasing. Consequently
its acceleration towards A is increasing in magnitude while its speed decreases and finally it
comes to rest momentarily at B.

(2) Motion of a particle from B to A :


Now the particle starts moving towards A with initial speed v = 0. As the particle is moving
towards A, force is acting on it towards A and decreasing as it approaches A. Consequently
its acceleration towards A is decreasing in magnitude while its speed increases and finally it
comes to A with same speed v = v0.

(3) Motion of a particle from A to C :


The motion of a particle from A to C is qualitatively same as motion of a particle from A to B.

(4) Motion of a particle from C to A :


It is qualitatively same as motion of a particle from B to A.

Summary :
Motion from Velocity (Direction/Magnitude) Acceleration (Direction/
Magnitude)
AB V a
BA V a
AC V a
CA V a

5. CHARACTERISTICS OF SHM :

(1) Mean Position : It is the position where net force on the particle is zero.
(2) Extreme Point : Point where speed of the particle is zero.
(3) Displacement : It is defined as the distance of the particle from the mean position at that
instant.

Corporate Head Office : Motion Education Pvt. Ltd., 394 - Rajeev Gandhi Nagar, Kota-5 (Raj.)
S.H.M. Page # 7

(4) Amplitude : It is the maximum value of displacement of the particle from its mean position.
Extreme position – Mean position = Amplitude.
It depends upon the energy of the system.
(5) Frequency : The frequency of SHM is equal to the number of complete oscillations per unit
time.

1 
f  sec –1 or Hz.
T 2
(6) Time Period : Smallest time interval after which the oscillatory motion gets repeated is called
time period.

2
T=

Ex.1 Describe the motion of a particle acted upon by a force.


(A) F = 3x + 3 (B) F = – 3x – 3 (C) F = – 3x + 3 (D) F
= 3x – 3
Sol. (a) Given F = 3x + 3 ...(i)
We find the mean position at which net force on the particle is zero.
 3x + 3 = 0  x=–1
If we put x = 0 in eq. (i) then
M.P.
F = 3N (away from M.P.) ...(a)
x = –1
Now put x = –2 in eq. (i)
F=– 3N (away from M.P.) ...(b)
From (a) and (b) we conclude that particle doesn't perform S.H.M.
(b) Given F = – 3x – 3 ...(i)
at M.P. F = 0
 x = –1
Now put x = 0 in eq. (i)
 F = – 3N (towards M.P.)
If x = – 2 ; F = 3N (towards M.P.)
We conclude from the above calculation that in every case (whether the particle is left from
M.P. or right from M.P.) force acts towards M.P. so the particle performs S.H.M.
(c) Given F = – 3x + 3
when F=0
x = 1 (M.P.)
Now put x = 0
Then F = 3N (towards M.P.)
If x = 2 F=–3 (towards M.P.)
i.e. particle performs S.H.M.
(D) Given F = 3x – 3
Mean position at x = 1.
When x = 0 ; F = – 3N (away from M.P.)
x = 2 ; F = 3N (away from M.P.)

: 0744-2209671, 08003899588 | url : www.motioniitjee.com, : info@motioniitjee.com


Page # 8 S.H.M.

Particle doesn't perform S.H.M.


6. EQUATION OF SIMPLE HARMONIC MOTION :

The necessary and sufficient condition for SHM is


F = – kx
we can write above equation in the following way:
ma = – kx

d2 x
m  –kx
dt 2
d2 x k
2
 x  0 ...(1)
dt m
Equation (1) is Double Differential Equation of SHM.

d2 x
Now  2 x  0
dt 2
It's solution is x = A sin( t  )

k
where  = angular frequency =
m
x = displacement from mean position
k = SHM constant.
The equality (t + ) is called the phase angle or simply the phase of the SHM and  is the
initial phase i.e., the phase at t = 0 and depends on initial position and direction of velocity at
t = 0.
To understand the role of  in SHM, we take two particles performing SHM in the following
condition:

v0 A
x x
–A M.P. –A M.P. A

figure I figure II

Suppose we choose t = 0 at an instant when the particle is passing through its mean position
towards right (i.e.positive direction) as shown in figure Ist then
In figure I at t=0 x=0
i.e., x = A sin t
 The particle is at its mean position.
In figure II at t=0 x = A and the particle is moving towards the mean position.
i.e., x = A sin (t + /2)

Corporate Head Office : Motion Education Pvt. Ltd., 394 - Rajeev Gandhi Nagar, Kota-5 (Raj.)
S.H.M. Page # 9

Here /2 is the only phase possible.

Ex.2 A particle starts from mean position and moves towards positive extreme as shown
below. Find the equation of the SHM. Amplitude of SHM is A.
t=0
–A O A
Sol. General equation of SHM can be written as x = A sin (t + )
At t = 0, x = 0
 0 = A sin 
  = 0,    [0, 2)
Also; at t = 0, v = + ve
 A  cos  = + ve or, =0
Hence, if the particle is at mean position at t = 0 and is moving towards +ve extreme, then
the equation of SHM is given by x = A sin t.
Similarly
for particle moving towards –ve extreme then
t=0
–A +A
=
 equation of SHM is x = A sin (t + )
or, x = – A sin t

Ex.3 Write the equation of SHM for the situation shown below :
t=0
–A O A/2 A
Sol. General equation of SHM can be written as
x = A sin (t + )
At t = 0, x = A/2
A
 = A sin 
2
  = 30° , 150°
Also at t = 0, v = – ve
A cos  = – ve   = 150°

7. VELOCITY :

It is the rate of change of particle displacement with respect to time at that instant.
Let the displacement from mean position is given
by x = A sin (t + )
dx
velocity v  A cos( t  )
dt
v = A cos (t + )
v =  A 2 – x2
At mean position (x = 0), velocity is maximum.
Vmax =  A
At extreme position (x = A), velocity is minimum.
vmin = zero.

: 0744-2209671, 08003899588 | url : www.motioniitjee.com, : info@motioniitjee.com


Page # 10 S.H.M.

7.1 Graph of Velocity (v) V/S Displacement (x) : Velocity (v)


2 2 2 2
v   A2 – x2 v   (A – x ) A

v2 x2
v 2  2 x 2  2 A 2 2 2
 1
 A A2
Graph would be a half ellipse. A
–A x

8. ACCELERATION :

It is the rate of change of particle's velocity w.r.t. time at that instant.

dv d
Acceleration, a  = [ A cos( t  )]
dt dt
a = – 2A sin (t + )
a = – 2 x

Note : Negative sign shows that acceleration is always directed towards the mean position. At mean
position (x =0), acceleration is minimum.
amin = zero
At extreme position (x = A), acceleration is maximum.
|amax | = 2A

8.1 Graph of Acceleration (A) v/s Displacement (x):

a
2 A

A
x
a=–x 2 –A
– 2 A

9. GRAPHICAL REPRESENTATION OF DISPLACEMENT, VELOCITY & ACCELERATION IN SHM:

Displacement, x = A sin t


Velocity, v = A  cos t = A  sin (t  )
2

or v   A2 – x2
Acceleration, a = – 2A sin t = 2A sin (t + )
or a = – 2 x
Note :
• v   A2 – x2
a = – 2 x
These relations are true for any equation of x.

Corporate Head Office : Motion Education Pvt. Ltd., 394 - Rajeev Gandhi Nagar, Kota-5 (Raj.)
S.H.M. Page # 11

tim e , t 0 T/4 T/2 3T/4 T


displa ce m e nt, x 0 A 0 –A 0
Ve locity, v A 0 –A  0 A
a cce le ra tion, a 0 2A 0 2
 A 0
–

x T/4 T/2 3T/4 T 5T/4 3T/2

t
–A
v
A
t
–A
a
2 A
t

– 2 A

1. All the three quantities displacement, velocity and acceleration vary harmonically
with time, having same period.
2. The maximum velocity is  times the amplitude (Vmax = A).
3. The acceleration is 2 times the displacement amplitude (amax = 2A).

4. In SHM, the velocity is ahead of displacement by a phase angle of .
2

5. In SHM, the acceleration is ahead of velocity by a phase angle of .
2

 
Ex.4 The equation of particle executing simple harmonic motion is x  (5m)sin(s –1 )t   .
 3
Write down the amplitude, time period and maximum speed. Also find the velocity at
t = 1 s.
Sol. Comparing with equation x = A sin (t + ), we see that the amplitude = 5m,

2 2
and time period = =  2s
  s –1

The maximum speed = A  = 5 m ×  s–1 = 5  m/s

The veloity at time t  dx = A  cos (t + )


dt
At t = 1 s,

  5
v = (5 m) ( s–1) cos    = – m/s
 3 2

: 0744-2209671, 08003899588 | url : www.motioniitjee.com, : info@motioniitjee.com


Page # 12 S.H.M.

Ex.5 A particle executing simple harmonic motion has angular frequency 6.28 s–1 and
amplitude 10 cm. Find (a) the time period, (b) the maximum speed, (c) the maximum
acceleration, (d) the speed when the displacement is 6 cm from the mean position,
(e) the speed at t = 1/6 s assuming that the motion starts from rest at t = 0.
2 2
Sol. (a) Time period = = s = 1 s.
 6.28
(b) Maximum speed = A = (0.1m) (6.28 s–1)
(c) Maximum acceleration = A2
= (0.1m) (6.28 s–1)2
= 4 m/s2
(d) v   A 2 – x 2 = (6.28 s–1) (10cm) 2 – ( 6cm)2 = 50.2 cm/s.
(e) At t = 0, the velocity is zero i.e., the particle is at an extreme. The equation for
displacement may be written as
x = A cos t.
The velocity is v = – A  sin t.
1  6.28 
At t  s, v = – (0.1 m) (6.28 s–1) sin 
6  6 

= (–0.628 m/s) sin
= – 54.4 cm/s. (towards mean position)
3
Note : If mean position is not at the origin, then we can replace x by x – x0 and the eqn.
becomes
x – x0 = – A sin t, where x0 is the position co-ordinate of the mean position.
Ex.6 A particle of mass 2 kg is moving on a straight line under the action force F = (8 – 2x)
N. It is released at rest from x = 6m.
(A) Is the particle moving simple harmonically?
(B) Find the equilibrium position of the particle.
(C) Write the equation of motion of the particle.
(D) Find the time period of SHM.
Sol. F = 8 – 2x
or F = –2(x – 4)
for equilibrium position F = 0
 x = 4m is equilibrium position.
Hence the motion of particle is SHM with force constant 2 and equilibrium position x =4.

(a) Yes, motion is SHM.


(b) Equilibrium position is x = 4m.
(c) At x = 6 m, particle at rest i.e. it is one of the extreme position. Hence amplitude is A = 2
m and initially particle at the extreme position.
 Equation of SHM can be written as

k 2
x – 4 = 2 cos t, where     1 (sec)–1
m 2
i.e. x = 4 + 2 cos t

(d) Time period, T  2  2 sec .


Corporate Head Office : Motion Education Pvt. Ltd., 394 - Rajeev Gandhi Nagar, Kota-5 (Raj.)
S.H.M. Page # 13

10. SHM AS A PROJECTION OF UNIFORM CIRCULAR MOTION.

Consider a particle Q, moving on a circle of radius )


A with constant angular velocity . The projection  t =0
(at )
of Q on a diameter BC is P. It is clear from the
A Q 0
t =t
figure that as Q moves around the circle the
 (at
projection P excecutes a simple harmonic motion t Q t
on the x-axis between B and C. The angle that B
O P0 Pt C
the radius OQ makes with the +ve vertical in
x(t)
clockwise direction in at t = 0 is equal to phase
constant ().
Let the radius OQ0 makes an angle t with the OQt
at time t. Then
x(t) = A sin (t + )
In the above discussion the foot of projection is –A (0,0) +A
x-axis so it is called horizontal phasor. Similarly M.P.
the foot of perpendicular on y axis will also
x(t)
executes SHM of amplitude A and angular
frequency  [y(t) = Acost]. This is called vertical
phasor. The phaser of the two SHM differ by /2.
Problem solving strategy in horizontal phasor:
(1) First assume circle of radius equal to amplitude of S.H.M.
(2) Assume a particle rotating in a circular path moving with constant  same as that of S.H.M in
clockwise direction.
(3) Angle made by the particle at t = 0 with the upper vertical is equal to phase constant.
(4) Horizontal component of velocity of particle gives you the velocity of particle performing
S.H.M. for example
0)
 t t= )
(a t t=t
0
Q Q t( a
 v(t)
t ( t  )
v  A

–A A
from figure
v(t) = A  cos (t + ) Q0
(5) Component of acceleration of particle in horizontal t  
a(t) Qt  
direction is equal to the acceleration of particle   – (t  )
t 2 
performing S.H.M. The acceleration of a particle 2 A
in uniform circular motion is only centripetal and
has a magnitude a = 2 A.
From figure
a(t) = – 2A sin (t + )

: 0744-2209671, 08003899588 | url : www.motioniitjee.com, : info@motioniitjee.com


Page # 14 S.H.M.

Ex.7 A particle starts from A/2 and moves towards positive extreme as shown below. Find
the equation of the SHM. Given amplitude of SHM is A. T
R
O t=0
–A A/2 +A A v(t)
Sol. We will solve the above problem with the help of
horizontal phasor. O' A/2 Q
Step 1. Draw a perpendicular line in upward direction from
point P on the circle which cuts it at point R & S S
v(t)
Step 2. Horizontal compoment of v(t) at R gives
O P
the direction P to A while at S gives P to O. So at
–A (0,0) A/2 A
t = 0 particle is at R M.P.(at t=0)

Step 3. In  O' RQ T R

A/2
cos  =  60   = 30° A
A
So equation of the SHM 

is x = A sin (t + 30°) Q
O' A/2
– 3
Ex.8 A particle starts from point x  A and move towards negative extreme as shown
2
t=0
O
–A – 3 +A
A
2
(a) Find the equation of the SHM.
(b) Find the time taken by the particle to go directly from its initial position to negative
extreme.
(c) Find the time taken by the particle to reach at mean position.
Sol. Figure shows the solution of the problem with the help of phasor R
Horizontal component of velcoity at Q gives the
required direction of velocity at t = 0. 

3 / 2A 3  S ( 3 / 2) A O
In  OSQ cos      
A 2 6 

3  8 4 A
Now  –  
2 6 6 3
Q
So equation of SHM is
 4 
x  A sin t   –A – 3 / 2 A (0,0) A
 3 
(b) Now to reach the particle at left extreme point it will travel angle  along the circle. So
time taken.
  T
t   t sec
 6 12
  2
(c) To reach the particle at mean position it will travel an angle  =  
2 6 3
 T
So, time taken = = sec
 3

Corporate Head Office : Motion Education Pvt. Ltd., 394 - Rajeev Gandhi Nagar, Kota-5 (Raj.)
S.H.M. Page # 15

Ex.9 Two particles undergoes SHM along parallel lines with the same time period (T) and
equal amplitudes. At a particular instant, one particle is at its extreme position while
the other is at its mean position. They move in the same direction. They will cross
each other after a further time.

B O A

B’ O’ A’

(A) T/8 (B) 3T/8 (C) T/6 (D) 4T/3


Sol. This problem is easy to solve with the help of phasor diagram.
First we draw the initial position of both the particle on the phasor as shown in figure.

P

B' II I A' at t = 0
–A A
From above figure phase difference between both the particles is /2.
They will cross each other when their projection from the circle on the horizontal diameter
meet at one point.
Let after time t both will reach at P'Q' point having phase difference /2 as shown in figure.

v(t)
Q'

A
45º P
45º 

v(t) A
Q
P'

B' I A' after t = t


R
–A II O +A
–A / 2

Both will meet at – A/ 2


When they meet angular displacement of P is
 = /2 + /4 = 3/4

So they will meet after time t  3


4

3 3T
t T  sec
4  2 8

: 0744-2209671, 08003899588 | url : www.motioniitjee.com, : info@motioniitjee.com


Page # 16 S.H.M.

Ex.10 Two particles execute SHM of same amplitude of 20 cm with same period along the
same line about the same equilibrium position. If phase difference is /3 then find out
the maximum distance between these two.
Sol. Let us assume that one particle starts from mean position and another starts at a distance x
having  = /3. This condition is shown in figure.
P

Q
/3


A/2 A/2
A 3 A A
 /3
2 /6
Q P

–A x A A

40cm A
Fig ii
Fig(i)
Above figure shows the situation of maximum distance between them.
So maximum distance = A = 10 cm. (as 2A = 20 cm)

Ex.11 Two particles execute SHM of same time period but different amplitudes along the
same line. One starts from mean position having amplitude A and other starts from
extreme position having amplitude 2A. Find out the time when they both will meet?
Sol. We solve the above problem with the help of phasor diagram.
First we draw the initial position of both the particle on the phasor.

(t=0)
A 

A
B (t=0)
2A

–A O A
–2A O 2A

From figure phase difference between both the particle is /2.


They will meet each other when their projection from the circle on the horizontal diameter
meet at one point.

Corporate Head Office : Motion Education Pvt. Ltd., 394 - Rajeev Gandhi Nagar, Kota-5 (Raj.)
S.H.M. Page # 17

Now from figure:


EF = A cos  = 2A sin 
 A'(t)
1 
tan  = A E F
2 90  

 1
  tan 1   2A
 2 B'(t)
So time taken by the particle to cross each other

angle travelled by A /2   P Q


t  t –A A
  –2A 2A
P' Q'
Ex.12 Two particles have time periods T and 5T/4. They start SHM at the same time from the
mean postion. After how many oscillations of the particle having smaller time period,
they will be again in the same phase ?
Sol. They will be again at m.p. and moving in same direction when the particle having smaller time
period makes n1 oscillations and the other one makes n2 oscillations.
5T
 n1T =  n2
4
n1 5
  n1 = 5, n2 = 4
n2 4
11. ENERGY OF SHM :
, 11.1 Kinetic Energy (KE):
1 1
K.E. = mv
v2 = mA22 cos2(t + )
2 2
1 k
= m 2 (A2 – x2)  2 =
2 m
1
 K.E = K( A 2 – x 2 )
2
1 2
K.Emax = KA (at x = 0)
2
1 1 2
K.Emin = 0 (at x = A) ; KEkA2 = ; KE 0–A
 kA
0–T 3
4
Frequency of KE = 2 × (frequency of SHM)
11.2 Potential Energy (PE):
Simple harmonic motion is defined by the equation
F = – kx
The work done by the force F during a displacement from x to x + dx is
dW = Fdx = – kx dx
The work done in a displacement from x = 0 to x is
x
1
W  ( kx ) dx   kx 2
 2
0

kx

M.P. x
dx

: 0744-2209671, 08003899588 | url : www.motioniitjee.com, : info@motioniitjee.com


Page # 18 S.H.M.

Let U(x) be the potential energy of the system when the displacement is x. As the change in
potential energy corresponding to a conservative force is the negative of the work done by
that force.
1 2
U(x) – UM.P. = – W = kx
2
Let us choose the potential energy to be zero when the particle is at the mean position
oscillation x = 0.
1 2
Then UM.P. = 0 and U( x)  kx
2
 k = m2
1
 U( x )  m 2 x 2
2
1
U= m 2 A 2 sin 2 ( t  )
2
But x = A sin (t + )
Kinetic energy of the particle at any instant is
1 1 1
K mv 2  mA 2 2 cos 2 (t  )  m2 ( A 2  x 2 )
2 2 2
So the total mechanical energy at time ‘t’ is

1
E=U+K  E m2 A 2
2

Note : Umin = UM.P. (which is not always = 0)

1 1
m2 A 2  kA 2  E
2 2
Energy

2
U = Umaxcos t
(i) 2
K = Kmaxsin t
O t
Potential, Kinetic and total energy plotted as function of time
Energy

22
E =1/2m A = Constant
1
U( x )  m2 x 2
2
K(x)
(ii) U(x)
1
K( x )  m2 ( A 2  x 2 )
x=0 2
x= –A x=A x
Potential, Kinetic and total energy are plotted as a function of displacement from the mean
position.

Ex.13 A particle of mass 0.50 kg executes a simple harmonic motion under a force F = – (50
N/m)x. If it crosses the centre of oscillation with a speed of 10 m/s, find the amplitude
of the motion.

Corporate Head Office : Motion Education Pvt. Ltd., 394 - Rajeev Gandhi Nagar, Kota-5 (Raj.)
S.H.M. Page # 19

Sol. The kinetic energy of the particle when it is at the centre of oscillation is

1 1
E mv 2 = (0.50kg) (10 m / s )2 = 2.5 J.
2 2
The potential energy is zero here. At the maximum displacement x = A, the speed is zero and
1 2
hence the kinetic energy is zero. The potential energy here is kA . As there is no loss of
2
energy,
1 2
kA  2.5 J
2
The force on the particle is given by
F = – (50 N/m) x.
Thus the spring constant is k = 50 N/m.
Equation (i) gives

1 1
(50 N / m)A 2  2.5J or, A= m.
2 10

12. METHOD TO DETERMINE TIME PERIOD AND ANGULAR FREQUENCY IN SIMPLE HARMONIC
MOTION :
To understand the steps which are usually followed to find out the time period we will take
one example.

Ex.14 A mass m is attached to the free


end of a massless spring of spring
constant k with its other end fixed
to a rigid support as shown in
figure. Find out the time period of
the mass, if it is displaced slightly
by an amount x downward.
m
Sol. The following steps are usually followed in this method:
Step 1. Find the stable equillibrium position which is usually known as the mean position. Net force or
torque on the particle at this position is zero. Potential energy is minimum.
In our example initial position is the mean position.

n.

x0

Natural Length M.P.


x0
kx0 x k(x+x0)
m m
Mean Position
mg mg

: 0744-2209671, 08003899588 | url : www.motioniitjee.com, : info@motioniitjee.com


Page # 20 S.H.M.

Step 2. Write down the mean position force relation. In above figure at mean position
kx0 = mg ...(1)
Step 3. Now displace the particle from its mean position by a small displacement x (in linear SHM) or
angle  (in case of an angular SHM) as shown in figure.
Step 4. Write down the net force on the particle in the displaced position.
From the above figure.
Fnet = mg – k (x + x0) ...(2)
Step 5. Now try to reduce this net force equation in the form of F = – kx (in linear S.H.M.) or  = –
k (in angular SHM) using mean position force relation in step 2 or binomial theorem.
from eq. (2) Fnet = mg – kx – kx0
Using eq (i) in above equation
Fnet = – kx ...(3)
Equation (3) shows that the net force acting towards mean position and is proportional to x,
but in this S.H.M. constant KS.H.M. is replaced by spring constant k. So
m m
T  2  2
K S.H.M. k

Note : If we apply constant force on the string then time period T is always same T  2 m
K S.H.M.

k
k
m

m F0

 m 

In above both cases T =  2 k 
 

Ex.15 The string, the spring and the pulley shown in figure are light.
Find the time period of the mass m.

m
Sol. Let in equilibrium position of the block, extension in spring is x0. k
 kx0 = mg ...(1)

Natural length
x0
m Equilibrium position
k x
m

Now if we displace the block by x in the downward direction, net force on the block towards
mean position is
F = k (x + x0) – mg = kx using (1)
Hence the net force is acting towards mean position and is also proportional to x. So, the
particle will perform S.H.M. and its time period would be

m
T  2
k

Corporate Head Office : Motion Education Pvt. Ltd., 394 - Rajeev Gandhi Nagar, Kota-5 (Raj.)
S.H.M. Page # 21

Ex.16 Figure shows a system consisting of a massless pulley, a spring of force constant k
and ablock of mass m. If the block is slightly displaced vertically down from its
equillibrium position and then released, find the period of its vertical oscillation in
cases (a) & (b).

k
k

m
m

(a) (b)
Sol. Let us assume that in equillibrium condition spring is x0 elongate from its natural length

n.

kx 0 x kx 0
x0  x0
2

x/2
T0
T0
m
Case (a) x T
T
mg m
mg

When equillibrium When displaced block by 'x'


In equilibrium T0 = mg
and kx0 = 2T0
 kx0 = 2mg ...(1)
If the mass m moves down a distance x from its equilibrium position then pulley will move
x kx
down by . So the extra force in spring will be . From figure
2 2
k x
Fnet = mg – T = mg –  x0  
2 2
kx 0 kx
Fnet = mg – –
2 4
from eq. (1)
–kx
Fnet = ...(3)
4
Now compare eq. (3) with F = – KS.H.M x
K
then KS.H.M =
4
m 4m
 T  2 = 2
K S.H.M K

: 0744-2209671, 08003899588 | url : www.motioniitjee.com, : info@motioniitjee.com


Page # 22 S.H.M.

Case (b) :
In this situation if the mass m moves down distance x from its equilibrium position, then pulley
will also move by x and so the spring will stretch by 2x.

T0 mg
At equilibrium kx0 = =
2 2
When block is displaced
Fnet = mg – T
= mg – 2k (x0 + 2x) x0 x0+2x

= – 4 kx
2x
Now F = – KSHM x then
T0
KSHM = 4 K
m
x T
m mg m
So time period T  2
4k
mg

Ex.17 The left block in figure collides inelastically with the right block and sticks to it. Find
the amplitude of the resulting simple harmonic motion.

v
m m k

Sol. The collision is for a small interval only, we can apply the principal of conservation of momentum.
2
v 1 v 1
The common velocity after the collision is . The kinetic energy  ( 2m)   mv2. This is
2 2 2 4
also the total energy of vibration as the spring is unstretched at this moment. If the amplitude
1 2
is A, the total energy can also be written as kA . Thus,
2

1 2 1 m
kA  mv 2 , giving A  v
2 4 2k
Ex.18 The system is in equilibrium and at rest. Now mass m1 is removed from m2. Find the
time period and amplitude of resultant motion. (Given : spring constant is K.)
Sol. Initial extension in the spring
(m1  m2 )g
x
k
Now, if we remove m1. equillibrium position (E.P.)
m2g
of m2 will be below natural length of spring. N.L m 2g
K
(m1  m2 )g K
At the initial position, since velocity is zero i.e. E.P
K m1g
it is the extreme position. m1 K
m1g m2 m2
Hence Amplitude =
K

m2
Time period = 2
K

Corporate Head Office : Motion Education Pvt. Ltd., 394 - Rajeev Gandhi Nagar, Kota-5 (Raj.)
S.H.M. Page # 23

Ex.19 Block of mass m2 is in equilibrium and at rest. The mass m1 moving with velocity u
vertically downwards collides with m2 and sticks to it. Find the energy of oscillation.

N.L.

Sol. m1 u
x0

m1 +m2 v
m2

At equilibrium position m2g = kx0

m2g
 x0 =
K
After collision m2 sticks to m1.  By momentum conservation.
m1u = (m1 + m2) v

m1u
v = m m
1 2

Now both the blocks are executing S.H.M. which can be interpreted as follows:
N.L. v

m.p.

m 2g m1g
K K
Now, we know that v2 = 2(A2 – x2) ...(1)

k
2 =
m1  m 2

m1g
 x=
k
Put the values of v, 2 & x in eq. (1)

2
 m1u 
2
 k   2  m1g  
     A –   
 m  m  =  m1  m 2    k  
 1 2  

 m 2 u 2
1
  m 1g  2 
 
 kA =  m  m
2
  k  

 1 2  

 2 2
  m12 g2 
1 2 1  m1 u  
 Energy of oscillation = kA =    k 
2 2  m1  m2   

: 0744-2209671, 08003899588 | url : www.motioniitjee.com, : info@motioniitjee.com


Page # 24 S.H.M.

Ex.20 A body of mass m falls from a height h on to the pan of a spring balance. The masses
of the pan and spring are negligible. The spring constant of the spring is k. Having
stuck to the pan the body starts performing harmonic oscillations in the vertical di-
rection. Find the amplitude and energy of oscillation.
Sol. Suppose by falling down through a height h, the mass m compresses the spring balance by a
length x.

mg k
x= ,=
k m

velocity at Q v = 2gh

 v =  A2  x2
2
k  mg  mg 2 kh
2gh  A2     A 1
m  k  k mg

2 2
1 2 1  mg   2kh   mgh  (mg)
Energy of oscillation  kA  k   1  
2 2  k   mg  2k
2m
Ex.21 A body of mass 2m is connected to another
body of mass m as shown in figure. The mass
2m performs vertical S.H.M. Then find out the
maximum amplitude of 2m such that mass m
doesn't lift up from the ground.

m
Sol. In the given situation 2m mass is in equilibrium condition.
Let assume spring is compressed x0 distance from its natural length.
 kx0 = 2mg
2mg
 x0  N.L.
k
The lower block will be lift up, only in the case x0
when the spring force on it will be greater than 2m M.P.
equal to mg and in upward direction kx0
mg
 kx' = mg  x' 
k
Above situation arises when 2m block moves
upward mg/k from natural length as shown in figure
kx0
A
2m m
x'=mg/k
N.L.
kx'
2mg
k
M.P.

kx'
m

Block m doesn't lift up if the maximum amplitude of the 2m block is


2mg mg 3mg
  
k k k

Corporate Head Office : Motion Education Pvt. Ltd., 394 - Rajeev Gandhi Nagar, Kota-5 (Raj.)
S.H.M. Page # 25

Ex.22 A block of mass m is at rest on the another m


block of same mass as shown in figure. Lower m
block is attached to the spring then determine
the maximum amplitude of motion so that
K
both the block will remain in
contact.
Sol.

N.L of spring

2mg
 x0
m k
m eq. position (M.P.)
Kx0

The blocks will remain in contact till the blocks do not go above the natural length of the
spring, because after this condition the deceleration of lower block becomes more then upper
block due to spring force. So they will get seprated.
2mg
So maximum possible amplitude = x0 =
k

12.1 Two Block Systems:

Ex.23 Two blocks of mass m1 and m2 are connected with a spring of natural length  and
spring constant k. The system is lying on a smooth horizontal surface. Initially spring
is compressed by x0 as shown in figure.
Show that the two blocks will perform SHM about their equilibrium position. Also (a)
find the time period, (b) find amplitude of each block and (c) length of spring as a
function of time.
  x0
k
m1 m2

Sol. (a) Here both the blocks will be in equilibrium at the same time when spring is in its natural
length. Let EP1 and EP2 be equilibrium positions of block A and B as shown in figure.
EP1 EP2

k
m1 m2

Let at any time during oscillations, blocks are at a distance of x1 and x2 from their equilibrium
positions.

: 0744-2209671, 08003899588 | url : www.motioniitjee.com, : info@motioniitjee.com


Page # 26 S.H.M.

EP1 EP2
x1 x2
k
m1 m2

As no external force is acting on the spring block system


 (m1 + m2)xcm = m1x1 – m2x2 = 0 or m1x1 = m2x2
For 1st particle, force equation can be written as

d2 x1  m 
k(x1 + x2) = – m1 or, k x1  1 x1   –m1a1
2 m2 
dt 
k(m1  m 2 ) k(m1  m 2 )
or, a1  – x1  2 
m1m2 m1m2

m1m2  m1m2
Hence, T = 2 = 2 where   which is known as reduced mass
k(m1  m2 ) K (m1  m 2 )

(b) Let the amplitude of blocks be A1 and A2.


m1A1 = m2A2
By energy conservation ;

1 1
k ( A 1  A 2 ) 2  kx 20 or, A1 + A2 = x0
2 2

m1
or, A1 + A2 = x0 or, A1  A 1  x0
m2

m 2 x0 m1x0
or, A1  Similarly, A2 = m  m
m1  m 2 1 2

(c) Let equilibrium position of 1st particle be origin, i.e., x = 0. EP1 EP2
x co-ordinate of particles can be written as 
x1 = A1 cost and x2 =  – A2cost
Hence, length of spring can be written as : m1 m2
length = x2 – x1
x=0
=  – (A1 + A2)cost
13. COMBINATION OF SPRINGS :

13.1 Series Combination :


Total displacement x = x1 + x2
k1 k2
Tension in both springs = k1x1 = k2x2
m
 Equivalent constant in series combination Keq is given by :

m
1/keq = 1/k1 + 1/k2  T  2
k eq

In series combination, tension is same in all the springs & extension will be different. (If k is
same then deformation is also same)

Corporate Head Office : Motion Education Pvt. Ltd., 394 - Rajeev Gandhi Nagar, Kota-5 (Raj.)
S.H.M. Page # 27

13.2 Parallel combination :

Extension is same for both springs but force acting will be different.
Force acting on the system = F
k1
 F = – (k1x + k2x)  F = – (k1 + k2) x  F = – keqx
k2 m
m
 keq = k1 + k2  T  2 k
eq

Ex.24 Find the time period of the oscilltion of mass m in figure a and b. What is the equivalent
spring constant of the spring in each case. ?

k1 k2 k1 k2
k3 m m
m

(a) (b)

Sol. In figure (a)

k 1k 2
k1 k2 k1  k 2

Which gives

k 1k 2
k1  k 2
k1k 2
+ k3
 k1  k 2

k3

k1k 2 k 1k 2  k 2k 3  k 1k 3
keq = k  k  k 3 = k1  k 2
1 2

Now m m(k 1  k 2 )
T  2  2
k eq k 1k 2  k 2k 3  k1k 3

Case (b)
If the block is displaced slightly by an amount x then both the spring are displaced by x from
their natural length so it is parallel combination of springs.
which gives
keq = k1 + k2

m m
T  2 2
k eq = k1  k 2

: 0744-2209671, 08003899588 | url : www.motioniitjee.com, : info@motioniitjee.com


Page # 28 S.H.M.

Note :
• In series combination, extension of springs will be reciprocal of its spring constant.
 k  1/
 k11 = k22 = k33
• If a spring is cut in 'n' equal pieces then spring constant of one piece will be nk.

Ex.25 The friction coefficient between the two blocks


shown in figure is  and the horizontal plane
is smooth. (a) If the system is slightly
m
displaced and released, find the time period. k
(b) Find the magnitude of the frictional force M
between the blocks when the displacement
from the mean position is x. (c) What can be
the maximum amplitude if the upper block
does not slip relative to the lower block?

Sol. (a) For small amplitude, the two blocks oscillate together.
The angular frequency is

k Mm
 and so the time period T  2
Mm k
(b) The acceleration of the blocks at displacement x from the mean position is

 – kx 
a  – 2 x   
Mm

 – mkx 
The resultant force on the upper block is, therefore, ma =  
 Mm 
This force is provided by the friction of the lower block. Hence, the magnitude of the
 mk | x | 
frictional force is  
 Mm 
(c) Maximum force of friction required for simple harmonic motion of the upper block is
mkA
at the extreme positions. But the maximum frictional force can only be  mg.
Mm
Hence

mkA (M  m)g


 mg or, A
Mm k
14. ENERGY METHOD :

Another method of finding time period of SHM is energy method. To understand this method
we will consider the following example.

Ex.26 Figure shows a system consisting of pulley


having radius R, a spring of force constant k
and a block of mass m. Find the period of its
vertical oscillation. K
m

Corporate Head Office : Motion Education Pvt. Ltd., 394 - Rajeev Gandhi Nagar, Kota-5 (Raj.)
S.H.M. Page # 29

Sol. The following steps are usually followed in this


method:
Step 1. Find the mean position. In following figure point A shows mean position.
Step 2. Write down the mean position force relation from figure. T=mg
mg = kx0 T=kx0
Step 3. Assume that particle is performing SHM with T
amplitude A. Then displace the particle from its T
mean position. x0 A
m M.P.
N.L.
Step 4. Find the total mechanical energy (E) in the
displaced position since, mechanical energy in SHM mg
dE
remains constant 0
dt
1 1 1
* E= mv 2  I2  k ( x  x 0 )2 – mgx
2 2 2
1 1 v2 1
E= mv 2  I 2  k( x  x 0 ) 2 – mgx
2 2 R 2
dE 2mv dv 2Iv dv 2k ( x  x 0 ) dx dx
   – mg ...(1)
dt 2 dt 2R 2 dt 2 dt dt
dx dv d2 x
Put  v and  2
dt dt dt
in eq. (1) put
dE d2 x Iv d2 x
0  mv   kxv  kx0 v  mgv  0
dt dt 2 R2 dt 2 v

R
 I  d2 x
which gives  m  2  2  kx  0
 R  dt

d2 x k
 x0 x+x0
dt 2
 I  G.P.E=0
m  2  ...(2)
 R  x
n.
m v
compare eq. (2) with S.H.M eq. the

k (m  I / R2 )
2   T  2
 I  k
m  2 
 R 

15. ANGULAR S.H.M. :


If the restoring torque acting on the body in oscillatory motion is directly proportional to the
angular displacement of body from its equillibrium position i.e.,
=–k
k = S.H.M. constant
 = angular displacement from M.P.
S.H.M. equation is given by

d2  K
2
 2   0 Here 
dt I
Here I is moment of inertia of the body/particle about a given axis.

: 0744-2209671, 08003899588 | url : www.motioniitjee.com, : info@motioniitjee.com


Page # 30 S.H.M.

16. SIMPLE PENDULUM :

If a heavy point-mass is suspended by a weightless, inextensible and perfectly flexible string


from a rigid support, then this arrangement is called a simple penduluml.

Time period of a simple pendulum T  2  . O


g  

(some times we can take g = 2 for making calculation simple)


m n   mgcos
si mg
mg
Proof :
Now taking moment of forces acting on the bob about point O.
 = T + mg
T = 0
  = –(mg sin )

if  is very small then sin  ~– 


  = – mg   ...(1)
Now compare eq. (1) with
net = – KS.H.M 
which gives KS.H.M = mg 

I m 2 
 T  2 = 2 = 2 g
K S.H.M mg

Note :
• Time period of second pendulum is 2 seconds.
• Simple pendulum performs angular S.H.M. but due to small angular displacement, it is
considered as linear S.H.M.
• If time period of clock based upon simple pendulum increases then clock will become
slow but if time period decreases then clock will become fast.

17. TIME PERIOD OF SIMPLE PENDULUM IN ACCELERATING REFERENCE FRAME :


T  2 where
geff .
 
geff = Effective acceleration due to gravity in reference system = | g – a |

a = acceleration of the point of suspension w.r.t. ground.
 
Condition for applying this formula : | g – a | = constant


 
If the acceleration a is upwards, then | g eff | g  a and T  2
ga
Time lost or gained in time t is given by

T
T'  .t
T

Corporate Head Office : Motion Education Pvt. Ltd., 394 - Rajeev Gandhi Nagar, Kota-5 (Raj.)
S.H.M. Page # 31

Ex.27 If T = 2 sec Tnew = 3 sec. then T = 1 sec.


Since time lost by clock in 3 sec is = 1 sec

1
then time lost by clock in 1 sec = sec
3

1
 Time lost by the clock in an hour =  3600 = 1200 sec.
3

Ex.28 A simple pendulum is suspended from the ceiling of a car which is accelerating
uniformly on a horizontal road. The acceleration of car is a0 and the length of the
pendulum is 1. Then find the time period of small oscillations of pendulum about the
mean position.
Sol. We shall work in the car frame. As it is accelerated with respect to the road, we shall have to
apply a psuedo force ma0 on the bob of mass m.
For mean position, the acceleration of the bob with respect to the car should be zero. If 0 be
the angle made by the string with the vertical, the tension, weight and the peusdo force will
add to zero in this position.

Hence, resultant of mg and ma0 (say F = m g2  a02 ) has to be along the string.

ma 0 a 0
 tan  0  
mg g

Now, suppose the string is further deflected by an angle  as


shown in figure.
Now, restoring torque about point O can be given by   I O
(F sin )  = – m 2  
 
0
Substituting F and using sin  = , for small . ma 0

( m g2  a02 )   = – m2  F mg

g2  a 20 g2  a02
or, –  so ; 2 
 

This is an equation of simple harmonic motion with time period.

2 
T = 2 2
 (g  a02 )1/ 4

18. COMPOUND PENDULUM / PHYSICAL PENDULUM :

s s
× ×
When a rigid body is suspended from an axis and

made to oscillate about that then it is called  
c
compound pendulum. c
C = Position of centre of mass mg sin 
S = Point of suspension
mg mg cos 
 = Distance between point of suspension and centre of mass

: 0744-2209671, 08003899588 | url : www.motioniitjee.com, : info@motioniitjee.com


Page # 32 S.H.M.

(it remains constant during motion for small angular displacement "" from mean position
The restoring torque is given by
 = – mg  sin 
 = – mg    for small , sin  = 
or, I = – mg   where, I = Moment of inertia about point of suspension.
mg mg
or, a= –  or, 2 
I I

I
Time period, T  2
mg

Ex. 29 A ring is suspended at a point on its rim and it behaves as a second's pendulum when
it oscillates such that its centre move in its own plane. The radius of the ring would be
( g = 2)
Sol. Time period of second pendulum T = 2 cm.
×
I
T  2 R
Mgd
Moment of inertia with respect to axis O M
C.O.M
I = MR2 + MR2 = 2MR2
the distance between centre of mass and the axis O
d=R

2MR 2
2  2  R = 0.5 m
MgR

Ex.30 A circular disc has a tiny hole in it, at a distance z from its center. Its mass is M and
radius R(R > 2). Horizontal shaft is passed through the hole and held fixed so that the
disc can freely swing in the vertical plane. For small distrubance, the disc performs
SHM whose time period is minimum for z . Find the value of z.

I
Sol. The time period w.r.t the axis T  2 Mgd ×O
z
where I = moment of inertia w.r.t the axis O M
R C.O.M
d = distance between C.O.M and O

MR 2
 I=  Mz 2
2
d=z
MR 2
 Mz 2
2 R2 z
 T  2  2 
Mgz 2gz g

R2
the time period will be minimum when  z = minimum
2z
R2
Let say f  z
2z
df
f will be minimum when 0
dz

Corporate Head Office : Motion Education Pvt. Ltd., 394 - Rajeev Gandhi Nagar, Kota-5 (Raj.)
S.H.M. Page # 33

R2 R
 – 2
1 0  z 
2z 2
Ex.31 Find out the angular frequency of small oscillation about axis O

m l

k k

m l

(1) l
(2)
Sol. mg
k k
(k l ) (k l )
l
The compression in spring (1) = l 
and the extension in spring (2) = l 
l
Net torque opposite to the mean position = – (2 kl  ) l – mg sin  = net
2
 is small  sin   
l
net = – I 2  = – (2 kl  ) l – mg sin  = net
2

ml 2
I=
3

3 ( 4k l  mg)
 =
2 ml
19. TORSIONAL PENDULUM :
In torsional pendulum, an extended object is suspended at the centre by a light torsion wire.
A torsion wire is essentially inextensible, but is free to twist about its axis. When the lower
end of the wire is rotated by a slight amount, the wire applies a restoring torque causing the
body to oscillate rotationally when released.
The restoring torque produced is given by

A

C A C
X X
 = – C where, C = Torsional constant
or, I = – C where, I = Moment of inertia about the vertical axis.

C I
or, = –   Time Period, T  2
I C

: 0744-2209671, 08003899588 | url : www.motioniitjee.com, : info@motioniitjee.com


Page # 34 S.H.M.

: The above concept of torsional pendulum is used in inertia table to calculate the moment of
inertia of unknown body.

Ex.32 A uniform disc of radius 5.0 cm and mass 200 g is fixed at its centre to a metal wire,
the other end of which is fixed to a ceiling. The hanging disc is rotated about the wire
through an angle and is released. If the disc makes torsional oscillations with time
period 0.20 s, find the torsional constant of the wire.
Sol. The situation is shown in figure. The moment of inertia of the disc about the wire is

mr 2 (0.200kg)(5.0  10 –2 m)2
I  = 2.5 × 10–4 kg-m2.
2 2
The time period is given by

I 4 2I
T  2 or, C
C T2

4 2 (2.5  10 –4 kg  m 2 ) kg  m 2
= = 0.25
(0.20 s)2 s2

20. VECTOR METHOD OF COMBINING TWO OR MORE SIMPLE HARMONIC MOTIONS:


A simple harmonic motion is produced when a force (called restoring force) proportional to
the displacement acts on a particle. If a particle is acted upon by two such forces the
resultant motion of the particle is a combination of two simple harmonic motions.

20.1 In Same direction :


(a) Having same Frequencies:
Suppose the two individual motions are represented by,
x1 = A1 sin t and x2 = A2 sin (t + )
Both the simple harmonic motions have same angular frequency .
x = x1 + x2 = A1 sin t + A2 sin (t + ) 
= A sin (t + ) A2 
A

Here, A  A 12  A 22  2A 1A 2 cos  A 2 sin


A 2 sin  
and tan  =
A 1  A 2 cos   A 2 cos 
A1
Thus, we can see that this is similar to the vector addition. The same method of vector
addition can be applied to the combination of more than two simple harmonic motions.

Important points to remember before solving the questions:


1. Convert all the trignometric ratios into sine form and ensure that t term is with +ve sign.
2. Make the sign between two term +ve.
3. A1 is the amplitude of that S.H.M whose phase is small.
4. Then resultant x = Anet sin (phase of A1 + )

Corporate Head Office : Motion Education Pvt. Ltd., 394 - Rajeev Gandhi Nagar, Kota-5 (Raj.)
S.H.M. Page # 35

Where Anet is the vector sum of A1 & A2 with angle between them is the phase difference
between two S.H.M.

Ex.33 x1 = 3 sin t ; x2 = 4 cos t


Find (i) amplitude of resultant SHM. (ii) equation of the resultant SHM.
Sol. First right all SHM's in terms of sine functions with positive amplitude. Keep "t" with positive
sign.
 x1 = 3 sin t
x2 = 4 sin (t + /2)


A  3 2  4 2  2  3  4 cos = 9  16 = 25 = 5
2

4 sin
4
2
tan   
 3  = 53°
3  4 cos
2
equation x = 5 sin (t + 53°)

10
Ex.34 x1 = 5 sin (t + 30°) ; x2 = 10 cos (t)
Find amplitude of resultant SHM.
Sol. x1 = 5 sin (t + 30°) 60°
 5
x2 = 10 sin (t + ) Phasor Diagram
2

A  5 2  10 2  2  5  10 cos 60 = 25  100  50  175 = 5 7


Ex.35 A particle is subjected to two simple harmonic motions
x1 = A1 sin t
and x2 = A2 sin (t + /3)
Find (a) the displacment at t = 0, (b) the maximum speed of the particle and (c) the
maximum acceleration of the particle.
Sol. (a) At t = 0, x1 = A1 sin t = 0
A2 3
and x2 = A2 sin (t + /3) = A2 sin (/3) =
2
Thus, the resultant displacement at t = 0 is
3
x = x1 + x2 = A 2
2
(b) The resultant of the two motion is a simple harmonic motion of the same angular
frequency . The amplitude of the resultant motion is

A  A 12  A 22  2A 1A 2 cos(  / 3) = A 12  A 22  A 1A 2
The maximum speed is

umax = A  =  A 12  A 22  A 1A 2
(c) The maximum acceleration is

amax = A 2 = 2 A 12  A 22  A 1A 2

(b) Having different frequencies


x1 = A1 sin t

: 0744-2209671, 08003899588 | url : www.motioniitjee.com, : info@motioniitjee.com


Page # 36 S.H.M.

x2 = A2 sin 2t
then resultant displacement x = x1 + x2 = A1 sin 1t + A2 sin 2t This resultant motion is not
SHM.
20.2 In two perpendicular directions
x = A1 sin t ...(1)
y = A2 sin (t + ) ...(2)
The Amplitudes A1 and A2 may be different and Phase difference  and  is same.
So equation of the path may be obtained by eliminating t from (1) & (2)

x
sin t = A ...(3)
1

x2
cos t = 1– 2 ...(4)
A1

On rearranging we get

x2 y2 2xy cos 
2
 2
  sin 2  ...(5)
A 1 A2 A 1A 2

(general eq. of ellipse)


special case :
(1) If  = 0

x2 y2 2xy
 2
 2
 0
A 1 A2 A 1A 2

A2
 y = A .x (eq. of straight line)
1

(2) If  = 90° A2
2 2
x y
 2
 1 (Eq. of ellipse) A1
A 1
A 22

(3) If  = 90° & A1 = A2 = A

then x2 + y2 = A2 (Eq. of circle.) A

The above figures are called Lissajous figures.

Corporate Head Office : Motion Education Pvt. Ltd., 394 - Rajeev Gandhi Nagar, Kota-5 (Raj.)
S.H.M. Page # 37

MIND MAP

1. Equation of S.H.M
(i) Linear : a = –2x
(ii) Angular :  = – 2

2. Linear SHM 3. Angular S.H.M


(i) Displacement of particle :
x = A sin (t + ) (i) Displacement :
(ii) Velocity  = 0 sin(t + )
dx
= A cos(t + ) (ii) Angular velocity
dt
d
=  A 2 – x2 = 0 cos(t + )
dt
(iii) Acceleration :
(iii) Acceleration
2
d x d2
= – A2 sin(t + )   0  2 sin(t  )   2 
dt 2 dt 2
= – 2x (iv) Phase : t + 
(iv) Phase : t + 
(v) Phase constant :  (v) Phase constant : 

5. Time Period :
4. Energy in S.H.M Pendulums :
1 (a) Simple pendulum :
(i) K = m 2 ( A 2 – x 2 )
2 l
T  2
SHM g
1 (b) Physical pendulum :
(ii) U = m2x2
2
I
T  2
1 mg l
(iii) E = K + U = m2 A2 (c) Torsional pendulum :
2
= constant I
T  2
C

6. Mass-spring system Combination of springs : 7. Composition of 2 SHMs :


x1 = A1 sin t
(a) T  2 m 1 1 1
k (a) series :   x2 = A2 sin (t + )
K eff K 1 K 2
(b) Two bodies system : x = x1 + x2
(b) parallel : Keff = K1 + K2 x = A sin (t + )

T  2 ; (c) Spring cut into two
k where, A  A12  A 22  2A1A 2 cos 
parts m : n
m1 m 2 (m  n)K (m  n)K A 2 sin 
where   m  m K1  ,K 2  and tan  
1 2 m n A 1  A 2 cos 

: 0744-2209671, 08003899588 | url : www.motioniitjee.com, : info@motioniitjee.com


Page # 38 S.H.M.

Exercise - I OBJECTIVE PROBLEMS (JEE MAIN)

LINEAR S.H.M. 4. A particle is executing S.H.M. of amplitude


'a' and time period = 4 second. Then the time
taken by it to move from the extreme position
1. For a particle executing simple harmonic motion,
to half the amplitude is
the acceleration is proportional to.
(A) displacement from the mean position 1
(A) 1 sec (B) sec
(B) distance from the mean position 3
(C) distance travelled since t = 0 2 4
(D) speed (C) sec (D) sec
3 3
Sol.

2. The distance moved by a particle in simple


harmonic motion in one time period is
(A) A (B) 2A (C) 4A (D) zero
Sol.

5. Equations y = 2 A cos2t and y = A(sin t +


3 cost) represent the motion of two particles.
(A) Only one of these is S.H.M
(B) Ratio of maximum speeds is 2 : 1
(C) Ratio of maximum speeds is 1 : 1
(D) Ratio of maximum accelerations is 1 : 4
Sol.
3. The time period of a particle in simple harmonic
motion is equal to the time between consecutive
appearance of the particle at a particular point in
its motion. This point is
(A) the mean position
(B) an extreme position
(C) between the mean position and the positive
extreme.
(D) between the mean position and the negative
extreme.
Sol.

Corporate Head Office : Motion Education Pvt. Ltd., 394 - Rajeev Gandhi Nagar, Kota-5 (Raj.)
S.H.M. Page # 39

6. The displacement of a body executing SHM is 9. The time period of a particle in simple harmonic
given by x = A sin(2t + /3). The first time from motion is equal to the smallest time between the
t = 0 when the velocity is maximum is particle acquiring a particular velocity v . The
(A) 0.33 sec (B) 0.16 sec value of v is
(C) 0.25 sec (D) 0.5 sec (A) vmax (B) 0
Sol. (C) between 0 and vmax
(D) between 0 and –vmax
Sol.

7. A simple harmonic motion having an amplitude


A and time period T is represented by the equa-
tion :
y = 5 sin(t + 4) m
Then the values of A (in m) and T (in sec) are :
(A) A = 5; T = 2 (B) A = 10 ; T = 1
(C) A = 5 ; T = 1 (D) A = 10 ; T = 2 10. The average acceleration in one time period
Sol. in a simple harmonic motion is
(A) A 2 (B) A 2/2
(C) A 2/ 2 (D) zero
Sol.

8. Two particles are in SHM on same straight line


with amplitude A and 2A and with same angular
frequency . It is observed that when first par-
ticle is at a distance A / 2 from origin and going
toward mean position, other particle is at ex-
treme position on other side of mean position.
Find phase difference between the two particles.
(A) 45° (B) 90°
(C) 135° (D) 180° 11. A mass m is performing linear simple harmonic
Sol. motion, then correct graph for acceleration a and
corresponding linear velocity v is

v2 v2
(A) (B)

a2 a2

v2 v2
(C) (D)

a2 a2

: 0744-2209671, 08003899588 | url : www.motioniitjee.com, : info@motioniitjee.com


Page # 40 S.H.M.

Sol. 14. A particle executes S.H.M. given by the


equation y = 0.45 sin 2t where y is in meter
and t is in second. What is the speed of the
particle when its displacement is 7.5 cm?

(A) 0.075 3 ms–1 (B) 7.5 3 ms–1

(C) 0.15 3 ms–1 (D) 15 3 ms–1


Sol.

12. The time taken by a particle performing SHM


to pass from point A to B where its velocities are
same is 2 seconds. After another 2 seconds it
returns to B. The time period of oscillation is (in
seconds)
(A) 2 (B) 8 (C) 6 (D) 4
Sol.

15. The maximum displacement of a particle


executing S.H.M. is 1 cm and the maximum
acceleration is (1.57)2cm per sec 2. Then the
time period is
(A) 0.25 sec (B) 4.00 sec
(C) 1.57 sec (D) (1.57)2 sec
13. A simple pendulum performs S.H.M. about
x = 0 with an amplitude a and time period T.
The speed of the pendulum at x = a/2 will be–
a 3 a 3
(A) (B)
T 2T
a 3 2 a
(C) (D)
T T
Sol.

Corporate Head Office : Motion Education Pvt. Ltd., 394 - Rajeev Gandhi Nagar, Kota-5 (Raj.)
S.H.M. Page # 41

16. A particle performing SHM is found at its equi- 18. Two particles are in SHM in a straight line
librium at t = 1 sec. and it is found to have a about same equilibrium position. Amplitude A and
speed of 0.25 m/s at t = 2 sec. If the period of time period T of both the particles are equal. At
oscillation is 6 sec. Calculate amplitude of oscil- time t = 0, one particle is at displacement y1 =
lation +A and the other at y2 = – A/2, and they are
approaching towards each other. After what time
3 3
(A) m (B) m they cross each other ?
2 4 (A) T/3 (B) T/4
6 3 (C) 5T/6 (D) T/6
(C) m (D) Sol.
 8
Sol.

19. Two particles execute SHM of same ampli-


tude of 20 cm with same period along the same
line about the same equilibrium position. The maxi-
mum distance between the two is 20 cm. Their
phase difference in radians is
2   
(A) (B) (C) (D)
3 2 3 4
Sol.

17. The angular frequency of motion whose equa-


d2 y
tion is 4 + 9y = 0 is (y = displacement and
dt 2
t = time)
9 4 3 2
(A) (B) (C) (D)
4 9 2 3
Sol.

20. A particle of mass 1 kg is undergoing S.H.M.,


for which graph between force and displacement
(from mean position) as shown. Its time period,
in seconds, is. F(N)

13.5
1.5
–1.5 xm
–13.5

(A) /3 (B) 2/3 (C) /6 (D) 3/

: 0744-2209671, 08003899588 | url : www.motioniitjee.com, : info@motioniitjee.com


Page # 42 S.H.M.

Sol. 23. A particle performs SHM with a period T and


amplitude a. The mean velocity of the particle
over the time interval during which it travels a
distance a/2 from the extreme position is
(A) a/T (B) 2a/T
(C) 3a/T (D) a/2T
Sol.

21. A point particle of mass 0.1 kg is executing


S.H.M of amplitude of 0.1 m. When the particle
passes through the mean position, its kinetic
energy is 8 × 10–3 J. The equation of motion of
this particle when the initial phase of oscillation
is 45° can be given by
   
(A) 0.1cos  4 t   (B) 0.1sin 4 t   24. A toy car of mass m is having two similar
 4  4 rubber ribbons attached to it as shown in the
figure. The force constant of each rubber ribbon

(C) 0.4 sin t    (D) 0.2 sin  2t is k and surface is frictionless. The car is displaced
 4 2  from mean position by x cm and released. At the
Sol. mean position the ribbons are underformed.
Vibration period is

m(2k ) 1 m(2k )
(A) 2 2 (B)
k 2 k2

m m
(C) 2 (D) 2
k k k
22. A particle executes SHM of period 1.2 sec. Sol.
and amplitude 8 cm. Find the time it takes to
travel 3 cm from the positive extremely of its
oscillation.
(A) 0.28 sec. (B) 0.32 sec.
(C) 0.17 sec. (D) 0.42 sec.
Sol.

Corporate Head Office : Motion Education Pvt. Ltd., 394 - Rajeev Gandhi Nagar, Kota-5 (Raj.)
S.H.M. Page # 43

25. A spring mass system oscillates with a


y y
frequency . If it is taken in an elevator slowly
accelerating upward, the frequency will
(A) increase (B) decrease
(C) remain same (D) become zero
Sol. (III) (IV)

O d O d
Which graphs best represents the relationship
obtained when y is
27. The total energy of the system
(A) I (B) II (C) III (D) IV
Sol.

26. A hollow metal sphere is filled with water


and hung by a long thread. A small hole is
drilled at the bottom through which water
slowly flows out. Now the sphere is made to
oscillate, the period of oscillation of the
pendulum -
(A) remains constant 28. The time
(B) continuously decreases (A) I (B) II (C) III (D) IV
(C) continuously increases Sol.
(D) first increases and then decreases

29. The unbalanced force acting on the system.


(A) I (B) II
(C) III (D) None of these
Sol.

Question No. 27 to 29 (3 questions)


The graph in figure show that a quantity y varies
with displacement d in a system undergoing simple
harmonic motion.
y y

(I) (II)

O d O d

: 0744-2209671, 08003899588 | url : www.motioniitjee.com, : info@motioniitjee.com


Page # 44 S.H.M.

30. The potential energy of a simple harmonic Sol.


oscillator of mass 2 kg in its mean position is 5 J.
If its total energy is 9J and its amplitude is 0.01
m, its time period would be
 
(A) sec (B) sec
10 20
 
(C) sec (D) sec
50 100
Sol.

33. A spring mass system performs S.H.M. If the


mass is doubled keeping amplitude same, then
the total energy of S.H.M. will become :
(A) double (B) half
(C) unchanged (D) 4 times
Sol.

31. Find the ratio of time periods of two identical


springs if they are first joined in series & then in
parallel & a mass m is suspended from them
(A) 4 (B) 2 (C) 1 (D) 3
Sol.
34. A mass at the end of a spring executes har-
monic motion about an equilibrium position with
an amplitude A. Its speed as it passes through
the equilibrium position is V. If extended 2A and
released, the speed of the mass passing through
the equilibrium position will be
(A) 2V (B) 4V
V V
(C) (D)
2 4
Sol.

32. Two bodies P & Q of equal mass are


suspended from two separate massless springs
of force constants k1 & k2 respectively. If the
maximum velocity of them are equal during their
motion, the ratio of amplitude of P to Q is :
k1 k2 k2 k1
(A) k (B) k1 (C) k (D)
2 1 k2

Corporate Head Office : Motion Education Pvt. Ltd., 394 - Rajeev Gandhi Nagar, Kota-5 (Raj.)
S.H.M. Page # 45

35. A particle is subjected to two mutually per- Sol.


pendicular simple harmonic motions such that its
x and y coordinates are given by x = 2 sin t ; y
 
= 2 sin  t  4 
The path of the particle will be :
(A) an ellipse (B) a straight line
(C) a parabola (D) a circle
Sol.

38. Two springs of the same material but of


length L and 2L are suspended with masses M
and 2M attached at their lower ends. Their
time periods when they are allowed to oscillate
will be in the ratio

36. The amplitude of the vibrating particle due


 
to superposition of two SHMs, y1 = sin  t  3 
and y2 = sin t is :
(A) 1 (B) 2 (C) 3 (D) 2
Sol.

(A) 1 : 2 (B) 2 : 1
(C) 1 : 4 (D) 4 : 1
Sol.

37. Two simple harmonic motions y1 = A sin t


and y2 = A cos t are superimposed on a particle
of mass m. The total mechanical energy of the
particle is :
1
(A) m2A2 (B) m2A2
2
1
(C) m2A2 (D) zero
4

: 0744-2209671, 08003899588 | url : www.motioniitjee.com, : info@motioniitjee.com


Page # 46 S.H.M.

39. A body of mass 'm' hangs from three springs, Sol.


each of spring constant 'k' as shown in the figure.
If the mass is slightly displaced and let go, the
system will oscillate with time period–

K K

m
41. A particle of mass 4 kg moves between two
points A and B on a smooth horizontal surface
m 3m under the action of two forces such that when it
(A) 2 (B) 2  
3k 2k is at a point P, the forces are 2PA N and 2PB N.
2m 3k If the particle is released from rest at A, find the
(C) 2 (D) 2 time it takes to travel a quarter of the way from
3k m
Sol. A to B.
  
(A) s (B) s (C) s (D) s
2 3 4
Sol.

42. In an elevator, a spring clock of time period


TS (mass attached to a spring) and a pendulum
clock of time period TP are kept. If the elevator
accelerates upwards
(A) TS well as TP increases
(B) TS remain same, TP increases
40. A block of mass m is connected between (C) TS remains same, TP decreases
(D) TS as well as TP decreases
two springs (constants K1 and K 2) as shown Sol.
in the figure and is made to oscillate, the
frequency of oscillation of the system shall
be-

1/ 2 1/ 2
1  m  1  K 1K 2 
(A)   (B)  
2  K1  K 2  2  (K 1  K 2 )m 
1/ 2 1/ 2
1  K1  K 2  1  (K 1  K 2 )m 
(C)   (D)  
2  m  2  K 1K 2 

Corporate Head Office : Motion Education Pvt. Ltd., 394 - Rajeev Gandhi Nagar, Kota-5 (Raj.)
S.H.M. Page # 47

43. Two pendulums have time periods T and 5T/ Sol.


4. They start SHM at the same time from the
mean position. After how many oscillations of the
smaller pendulum they will be again in the same
phase
(A) 5 (B) 4 (C) 11 (D) 9
Sol.

46. A ring is suspended at a point on its rim and


it behaves as a second’s pendulum when it oscil-
lates such that its centre move in its own plane.
The radius of the ring would be (g = 2)
(A) 0.5 m (B) 1.0 m
(C) 0.67 m (D) 1.5 m
44. A simple pendulum is oscillating in a lift. If Sol.
the lift is going down with constant velocity, the
time period of the simple pendulum is T1. If the
lift is going down with some retardation its time
period is T2, then
(A) T1 > T2 (B) T1 < T2 (C) T1 = T2
(D) depends upon the mass of the pendulum bob
Sol.

45. A simple pendulum with length  and bob of


mass m executes SHM of small amplitude A. The
maximum tension in the string will be
(A) mg (1 + A/) (B) mg (1 + A/)2
(C) mg [1 + (A/)2] (D) 2 mg

: 0744-2209671, 08003899588 | url : www.motioniitjee.com, : info@motioniitjee.com


Page # 48 S.H.M.

Exercise - II

SINGLE CORRECT Sol.

1. The maximum acceleration of a particle in SHM


is made two times keeping the maximum speed to
be constant. It is possible when
(A) amplitude of oscillation is doubled while fre-
quency remains constant
(B) amplitude is doubled while frequency is halved
(C) frequency is doubled while amplitude is halved
(D) frequency is doubled while amplitude remains
constant.
Sol.

4. A particle executes SHM with time period T


and amplitude A. The maximum possible average
T
2. A small mass executes linear SHM about O velocity in time is :
4
with amplitude a and period T. Its displacement
from O at time T/8 after passing through O is : 2A 4A 8A 4 2A
(A) (B) (C) (D)
(A) a/8 (B) a/2 2 T T T T
Sol.
(C) a/2 (D) a / 2
Sol.

3. Two particles undergo SHM along parallel lines


with the same time period (T) and equal ampli-
tudes. At a particular instant, one particle is at
its extreme position while the other is at its mean
position. They move in the same direction. They
will cross each other after a further time
5. Time period of a particle executing SHM is 8
B O A sec. At t = 0 it is at the mean position. The ratio
of the distance covered by the particle in the 1
B’ O’ A’ st second to the 2nd second is

(A) T/8 (B) 3T/8 1 1


(A) (B) 2 (C) (D) 2 +1
(C) T/6 (D) 4T/3 2 1 2

Corporate Head Office : Motion Education Pvt. Ltd., 394 - Rajeev Gandhi Nagar, Kota-5 (Raj.)
S.H.M. Page # 49

Sol. 8. A body at the end of a spring executes S.H.M.


with a period t1, while the corresponding period
for another spring is t2. If the period of oscillation
with the two spring in series is T, then
(A) T = t1 + t2 (B) T 2 = t12 + t22
1 1 1 1 1 1
(C) T  t  t (D) 2
 
1 2 T t12 t 22
Sol.

6. Two particles P and Q describe simple har-


monic motions of same period, same amplitude,
along the same line about the same equilibrium
position O. When P and Q are on opposite sides
of O at the same distance from O they have the
same speed of 1.2 m/s in the same direction,
when their displacements are the same they have
the same speed of 1.6 m/s in opposite directions.
The maximum velocity in m/s of either particle is
(A) 2.8 (B) 2.5 (C) 2.4 (D) 2
Sol.

9. A particle moves along the x-axis according to


x = A. [1 + sin t]. What distance does it travel
between t = 0 and t = 2.5/?
(A) 4A (B) 6A (C) 5A (D) none
Sol.

7. A body performs simple harmonic oscillations


along the straight line ABCDE with C as the mid-
point of AE. Its kinetic energies at B
and D are each one fourth of its maximum value.
If AE = 2R, the distance between B and D is

A B C D E
3R R
(A) (B) (C) 3R (D) 2R
2 2
Sol.
10. A particle executes SHM on a straight line
path. The amplitude of oscillation is 2 cm. When
the displacement of the particle from the mean
position is 1 cm, the numerical value of magni-
tude of acceleration is equal to the numerical
value of magnitude of velocity. The frequency of
SHM (in second–1) is
2 3 1
(A) 2 3 (B) (C) (D)
3 2 2 3

: 0744-2209671, 08003899588 | url : www.motioniitjee.com, : info@motioniitjee.com


Page # 50 S.H.M.

Sol. Sol.

13. A particle starts oscillating simple harmoni-


11. Vertical displacement of a plank with a body cally from its equilibrium position then, the ratio
of mass ‘m’ on it is varying according to law y = of kinetic energy and potential energy of the par-
sin t + 3 cos t. The minimum value of  for ticle at the time T/12 is : (T = time period)
(A) 2 : 1 (B) 3 : 1
which the mass just breaks off the plank and the
(C) 4 : 1 (D) 1 : 4
moment it occurs first after t = 0 are given by (y
Sol.
is positive vertically upwards)

g 2  g 2 
(A) , (B) ,
2 6 g 2 3 g

g  2 2
(C) , (D) 2g ,
2 3 g 3g
Sol.
14. In the figure, the block of mass m, attached
to the spring of stiffness k is in contact with the
completely elastic wall, and the compression in
the spring is ‘e’. The spring is compressed further
by ‘e’ by displacing the blocktowards left and is
then released. If the collision between the block
and the wall is completely elastic then the time
period of oscillations of the block will be :

Wall

12. Two particles A and B perform SHM along the m


same straight line with the same amplitude ‘a’,
same frequency ‘f’ and same equilibrium position
‘O’. The greatest distance between them is found
2 m m
to be 3a/2. At some instant of time they have (A) (B) 2
the same displacement from mean position. What 3 k k
is the displacement?
(A) a / 2 (B) a 7 / 4  m  m
(C) (D)
3 k 6 k
(C) 3a/2 (D) 3 a /4

Corporate Head Office : Motion Education Pvt. Ltd., 394 - Rajeev Gandhi Nagar, Kota-5 (Raj.)
S.H.M. Page # 51

Sol. 17. The springs in fig. A and B are identical but


length in A is three times each of that in B. The
ratio of period TA/TB is
A B

m m
(A) 3 (B) 1/3 (C) 3 (D) 1/3
Sol.

15. A 2 Kg block moving with 10 m/s strikes a


spring of constant 2N/m attached to 2 Kg block
at rest kept on a smooth floor. The time for
which rear moving block remain in contact with
spring will be
10m/s
2kg 2kg

18. A particle of mass m moves in the potential


1
(A) energy U shown above. The period of the motion
2 sec (B)
2
sec
when the particle has total energy E is
1
(C) 1 sec (D) sec U(x)
2
Sol.

1
U = 2 kx2, x < 0 U = mgx, x > 0

(A) 2 m / k + 4 2E / mg 2

(B) 2 m / k
16. In the above question, the velocity of the
rear 2 kg block after it separates from the spring (C)  m / k + 2 2E / mg2
will be :
(A) 0 m/s (B) 5 m/s (D) 2 2E / mg2
(C) 10 m/s (D) 7.5 m/s
Sol. Sol.

: 0744-2209671, 08003899588 | url : www.motioniitjee.com, : info@motioniitjee.com


Page # 52 S.H.M.

19. A man is swinging on a swing made of 2 21. A small bob attached to a light inextensible
ropes of equal length L and in direction perpen- thread of length l has a periodic time T when
dicular to the plane of paper. The time period of allowed to vibrate as a simple pendulum. The
the small oscillations about the mean position is thread is now suspended from a fixed end O of a
3
vertical rigid rod of length (as in figure). If
4
L now the pendulum performs periodic oscillations
in this arrangement, the periodic time will be
L L
O

M 3l
4 l
L 3L
(A) 2 (B) 2
2g 2g

L L A
(C) 2 (D) 
2 3g g
Sol. 3T T
(A) (B) (C) T (D) 2T
4 2
Sol.

20. A ring of diameter 2m oscillates as a com-


pound pendulum about a horizontal axis passing
through a point at its rim. It oscillates such that
its centre move in a plane which is perpendicular
to the plane of the ring. The equivalent length of 22. A system of two identical rods (L-shaped) of
the simple pendulum is mass m and length l are resting on a peg P as
(A) 2m (B) 4m shown in the figure. If the system is displaced in
(C) 1.5 m (D) 3m its plane by a small angle , find the period of
Sol.
oscillations.

P
l l

2l 2 2l
(A) 2 (B) 2
3g 3g

2l l
(C) 2 3g (D) 3 3g

Corporate Head Office : Motion Education Pvt. Ltd., 394 - Rajeev Gandhi Nagar, Kota-5 (Raj.)
S.H.M. Page # 53

Sol. Sol.

25. A rod whose ends are A & B of length 25 cm


is hanged in vertical plane. When hanged from
23. In the figure shown, the spring are connected point A and point B the time periods calculated
to the rod at one end and at the midpoint. The are 3 sec & 4 sec respectively. Given the mo-
rod is hinged at its lower end. Rotational SHM of ment of inertia of rod about axis perpendicular to
the rod (Mass m, length L) will occur only if the rod is in ratio 9 : 4 at points A and B. Find the
distance of the centre of mass from point A.
k (A) 9 cm (B) 5 cm
(C) 25 cm (D) 20 cm
k Sol.

(A) k > mg / 3L (B) k > 2mg/3L


(C) k > 2mg/5L (D) k > 0
Sol.

MULTIPLE CORRECT

26. A spring has natural length 40 cm and spring


constant 500 N/m. A block of mass 1 kg is at-
tached at one end of the spring and other end of
the spring is attached to ceiling. The block re-
leased from the position, where the spring has
length 45 cm.
(A) the block will perform SHM of amplitude 5 cm.
(B) the block will have maximum velocity 30 5
cm/sec.
24. What is the angular frequency of oscillations (C) the block will have maximum acceleration 15
of the rod in the above problem if k = mg/L ? m/s2.
(A) (3/2).[k/m]1/2 (B) (3/4).[k/m]1/2 (D) the minimum potential energy of the spring
(C) [2k/5m] 1/2
(D) None
will be zero.

: 0744-2209671, 08003899588 | url : www.motioniitjee.com, : info@motioniitjee.com


Page # 54 S.H.M.

Sol. Sol.

27. A particle executing a simple harmonic mo-


tion of period 2s. When it is at its extreme dis-
placement from its mean position, it receives an 29. Two particles execute SHM with amplitude A
additional energy equal to what it had in its mean and 2A and angular frequency  and 2 respec-
position. Due to this, in its subsequent motion, tively. At t = 0 they starts with some initial phase
(A) its amplitude will change and become equal
2
to 2 times its previous amplitude difference. At, t = . They are in same phase.
3
(B) its periodic time will become doubled i.e. 4s
Their initial phase difference is :
(C) its potential energy will be decreased
(D) it will continue to execute simple harmonic  2 4
motion of the same amplitude and period as be- (A) (B) (C) (D) 
3 3 3
fore receiving the additional energy. Sol.
Sol.

30. Two particles are in SHM with same angular


frequency and amplitudes A and 2A respectively
28. Part of a simple harmonic motion is graphed
along same straight line with same mean posi-
in the figure, where y is the displacement from
tion. They cross each other at position A/2 dis-
the mean position. The correct equation describing
tance from mean position in opposite direction.
this S.H.M is
The phase between them is :
2
5  1   1
(A) – sin–1   (B) – sin–1  
6  4 6  4
(0.6) t(s)
O (0.3) 5  1   1
(C) – cos–1   (D) – cos –1 
6 4 6  4
–2
Sol.

 10 
(A) y = 4 cos (0.6t) (B) y = 2 sin  t – 
3 2

 10   10 
(C) y = 4 sin  t   (D) y = 2 cos  t  
3 2 3 2

Corporate Head Office : Motion Education Pvt. Ltd., 394 - Rajeev Gandhi Nagar, Kota-5 (Raj.)
S.H.M. Page # 55

31. The equation of motion for an oscillating par- Sol.


ticle is given by x = 3sin (4t) + 4 cos(4t), where
x is in mm and t is in second
(A) The motion is simple harmonic
(B) The period of oscillation is 0.5 s
(C) The amplitude of oscillation is 5 mm
(D) The particle starts its motion from the equi-
librium
Sol.

34. A block is placed on a horizontal plank. The


plank is performing SHM along a vertical line with
amplitude of 40 cm. The block just loses contact
with the plank when the plank is momentarily at
rest. Then
(A) the period of its oscillations is 2/5 sec.
(B) the block weights on the plank double its
weight, when the plank is at one of the positions
32. A particle is executing SHM of amplitude A,
of momentary rest.
about the mean position X = 0. Which of the
(C) the block weights 1.5 times its weight on the
following cannot be a possible phase difference
plank halfway down from the mean position.
betwe en the pos i ti ons of the parti c l e at
(D) the block weights its true weight on the plank,
x = + A/2 and x = – A/ 2 . when velocity of the plank is maximum.
(A) 75° (B) 165° (C) 135° (D) 195° Sol.
Sol.

33. Speed v of a particle moving along a straight


line, when it is at a distance x from a fixed point
on the line is given by v 2 = 108 – 9x 2 (all
quantities in S.I. unit). Then 35. The potential energy of a particle of mass
(A) The motion is uniformly accelerated along the 0.1 kg, moving along x-axis, is given by U = 5x
straight line (x – 4) J where x is in metres. It can be con-
(B) The magnitude of the acceleration at a cluded that
distance 3 cm from the fixed point is 0.27 m/s2. (A) the particle is acted upon by a constant force
(C) The motion is simple harmonic about x = 12 m. (B) the speed of the particle is maximum at x = 2 m
(D) The maximum displacement from the fixed (C) the particle executes simple harmonic motion
point is 4 cm. (D) the period of oscillation of the particle is /5 s

: 0744-2209671, 08003899588 | url : www.motioniitjee.com, : info@motioniitjee.com


Page # 56 S.H.M.

Sol. Sol.

38. The displacement of a particle varies ac-


36. A particle is executing SHM with amplitude A, cording to the relation x = 3 sin 100t + 8 cos2
time period T, maximum acceleration a0 and maxi- 50t. Which of the following is/are correct about
mum velocity v0. Its starts from mean position at this motion.
t = 0 and at time t, it has the displacement A/2, (A) the motion of the particle is not S.H.M.
acceleration a and velocity v then (B) theamplitude of the S.H.M. of the particle is
(A) t = T/12 (B) a = a0/2 5 units
(C) v = v0/2 (D) t = T/8 (C) the amplitude of the resultant S.H.M. is 73
Sol.
units
(D) the maximum displacement of the particle from
the origin is 9 units.
Sol.

37. The amplitude of a particle executing SHM


about O is 10 cm. Then 39. In SHM, acceleration versus displacement
(A) When the K.E. is 0.64 of its max. K.E. its (from mean position) graph :
displacement is 6cm from O. (A) is always a straight line passing through
(B) When the displacement is 5 cm from O its origin and slope –2
K.E. is 0.75 of its max. P.E. (B) is always a straight line passing through
(C) Its total energy at any point is equal to its origin and slope +2
maximum K.E. (C) is a straight line not necessarily passing through
(D) Its velocity is half the maximum velocity when its origin
displacement is half the maximum displacement. (D) none of the above

Corporate Head Office : Motion Education Pvt. Ltd., 394 - Rajeev Gandhi Nagar, Kota-5 (Raj.)
S.H.M. Page # 57

Sol. 42. The figure shows a graph between velocity


and displacement (from mean position) of a par-
ticle performing SHM
v(in cm/s)
10
2.5
x
(in cm)

(A) the time period of the particle is 1.57 s


(B) the maximum acceleration will be 40cm/s2
(C) the velocity of particle is 2 21 cm/s when it
is at a distance 1 cm from the mean position.
(D) none of these
40. A particle moves in xy plane according to the
Sol.
law x = a sin t and y = a(1 – cos t) where a
and  are constants. The particle traces
(A) a parabola
(B) a straight line equallyinclined to x and y axes
(C) a circle
(D) a distance proportional to time
Sol.

43. Two blocks of masses 3 kg and 6 kg rest on a


horizontal smooth surface. The 3 kg block is at-
tached to a spring with a force constant k = 900
Nm–1 which is compressed 2 m from beyond the
equilibrium position. The 6 kg mass is at rest at
1m from mean position. 3kg mass strikes the 6 kg
mass and the two stick together.
1m

41. For a particle executing S.H.M., x = displace- 3kg


2m
6kg
ment from equilibrium position, v = velocity at
any instant and a = acceleration at any instant, equilibrium
then position
(A) v-x graph is a circle (A) velocity of the combined masses immediately
(B) v-x graph is an ellipse after the collision is 10 ms–1
(C) a-x graph is a straight line (B) velocity of the combined masses immediately
(D) a-v graph is an ellipse after thecollision is 5 ms–1
Sol. (C) amplitude of the resulting oscillation is 2 m
(D) amplitude of the resulting oscillation is 5/2 m
Sol.

: 0744-2209671, 08003899588 | url : www.motioniitjee.com, : info@motioniitjee.com


Page # 58 S.H.M.

44. A particle starts from a point P at a distance Sol.


of A/2 from the mean position O & travels towards
left as shown in the figure. If the time period of
SHM, executed about O is T and amplitude A
then the equation of motion of particle is :
A/2

O P
A
 2   2 5 
(A) x = A sin  t   (B) x = A sin  t 
T 6  T 6 

 2   2 
(C) x = A cos  T t  6  (D) x = A cos  T t  3 

Sol. 46. A disc of mass 3m and a disc of mass m are


connected by a massless spring of stiffness k.
The heavier disc is placed on the ground with the
spring vertical and lighter disc on top. From its
equilibrium position, the upper disc is pushed down
by a distance  and released. Then
(A) if  > 3mg/k, the lower disc will bounce up
(B) if  = 2mg/k, maximum normal reaction from
ground on lower disc = 6 mg.
(C) if  = 2mg/k, maximum normal reaction from
ground on lower disc = 4 mg.
(D) if  > 4 mg/k, the lower disc will bounce up
Sol.

45. The angular frequency of a spring block sys-


tem is 0. This system is suspended from the
ceiling of an elevator moving downwards with a
constant speed v0. The block is at rest relative
to the elevator. Lift is suddenly stopped. Assum- 47. A system is oscillating with undamped simple
ing the downwards as a positive direction, choose harmonic motion. Then the
the wrong statement. (A) average total energy per cycle of the motion
v0 is its maximum kinetic energy.
(A) The amplitude of the block is  (B) average total energy per cycle of the motion
0
(B) The initial phase of the block is . 1
is times its maximum kinetic energy..
2
v0 1
(C) The equation of motion for the block is  (C) root mean square velocity is times its
0 2
maximum velocity
sin 0t.
(D) The maximum speed of the block is v0. (D) mean velocity is 1/2 of maximum velocity.

Corporate Head Office : Motion Education Pvt. Ltd., 394 - Rajeev Gandhi Nagar, Kota-5 (Raj.)
S.H.M. Page # 59

Sol. 50. The graph plotted between phase angle ()


and displacement of a particle from equilibrium
position (y) is a sinusoidal curve as shown below.
Then the best matching is
y

3 /2 2
O /2 P t

Column A Column B
48. A particle of mass m performs SHM along a
(A) K. E. versus
straight line with frequency f and amplitude A.
phase angle curve (i)
(A) The average kinetic energy of the particle is
zero.
(B) P.E. versus phase
(B) The average potential energy is m 2 f2 A2.
(C) The frequency of ocillation of kinetic energy
is 2f. angle curve (ii)
(D) Velocity function leads acceleration by /2.
Sol. (C) T.e. versus phase
angle curve (iii)

(D) Velocity versus

phase angle curve (iv)

(A) (a) - (i), (b) - (ii), (c) - (iii) & (d) - (iv)
(B) (a) - (ii), (b) - (i), (c) - (iii) & (d) - (iv)
49. A linear harmonic oscillator of force constant (C) (a) - (ii), (b) - (i), (c) - (iv) & (d) - (iii)
2 × 106Nm–1 and amplitude 0.01 m has a total (D) (a) - (ii), (b) - (iii), (c) - (iv) & (d) - (i)
mechanical energy of 160 J. Its Sol.
(A) maximum potential energy is 100 J
(B) maximum kinetic energy is 100J
(C) maximum potential energy is 160 J
(D) minimum potential energy is zero.
Sol.

: 0744-2209671, 08003899588 | url : www.motioniitjee.com, : info@motioniitjee.com


Page # 60 S.H.M.

Exercise - III (JEE ADVANCED)

LINEAR S.H.M

1. The equation of a particle executing SHM is


 
x  (5m)sin( s –1 )t   . Write down the amplitude,
 6
phase constant, time period and maximum speed.
Sol.

3. The equation of motion of a particle started at


t = 0 is given by x = 5 sin (20 t + /3) where x is
in centimetre and t in second. When does the
particle.
(a) first come to rest ?
(b) first have zero acceleration ?
(c) first have maximum speed ?
Sol.

2. A particle having mass 10 g oscillates according


to the equation x = (2.0 cm) sin [100 s–1] t +

]. Find (a) the amplitude, the time period and
6
the force constant (b) the position, the velocity
and the acceleration at t = 0.
Sol.

4. A body is in SHM with period T when oscillated


from a freely suspended spring. If this spring is
cut in two parts of length ratio 1 : 3 & again
oscillated from the two parts separately, then
the periods are T1 & T2 then find T1/T2.
Sol.

Corporate Head Office : Motion Education Pvt. Ltd., 394 - Rajeev Gandhi Nagar, Kota-5 (Raj.)
S.H.M. Page # 61

Sol.

5. The system shown in the figure can move on a


smooth surface. The spring is initially compressed
by 6 cm and then released. Find
k = 800 N/m 6 kg
3 kg

(a) Time period (b) Amplitude of 3 kg block


(c) Maximum momentum of 6 kg block
Sol. 7. The acceleration-displacement (a – x) graph
of a particle executing simple harmonic motion is
shown in the figure. Find the frequency of oscil-
a
lation.

– O x

Sol.

6. A body undergoing SHM about the origin has


its equation is given by x = 0.2 cos 5t. Find its
average speed from t = 0 to t = 0.7 sec.

: 0744-2209671, 08003899588 | url : www.motioniitjee.com, : info@motioniitjee.com


Page # 62 S.H.M.

8. A block of mass 0.9 kg attached to a spring of 10. Potential energy (U) of a body of unit mass
force constant k is lying on a frictionless floor. moving in a one-dimension conservative force fileld
is given by, U = (x2 – 4x + 3). All units are in S.I.
The spring is compressed to 2 cm and the block
(i) Find the equilibrium position of the body.
is at a distance 1/ 2 cm from the wall as shown (ii) Show that oscillations of the body about this
in the figure. When the block is released, it make equilibrium position is simple harmonic motion &
elastic collision with the wall and its period of find its timeperiod.
motion is 0.2 sec. Find the approximate value of k. (iii) Find the amplitude of oscillations if speed of
the body at equilibrium position is 2 6 m/s.
Wall
Sol.

1/ 2 cm
2 cm
Sol.

11. The resulting amplitude A and the phase of


A  
the vibrations  S = A cos (t) + cos  t  2 
2
9. A force f = –10x + 2 acts on a particle of mass A A  3 
0.1 kg, where ‘k’ is in m and F in newton. If it is + cos (t + ) + cos  t  2  = A cos (t
4 8
released from rest at x = –2 m, find : + ) are ....... and ......... respectively.
(A) Amplitude (b) Time period Sol.
(c) Equation of motion
Sol.

Corporate Head Office : Motion Education Pvt. Ltd., 394 - Rajeev Gandhi Nagar, Kota-5 (Raj.)
S.H.M. Page # 63

12. A body is executing SHM under the action of 14. The springs shown in the figure are all
force whose maximum magnitude is 50N. Find the unstretched in the beginning when a man starts
magnitude of force acting on the particle at the pulling the block. The man exerts a constant force
time when its energy is half kinetic and half po- F on the block. Find the amplitude and the
tential. frequency of the motion of the block.
Sol. M
k1

k2 k3 F

Sol.

13. A 1kg block is executing simple harmonic


motion of amplitude 0.1 m on a smooth horizontal
surface under the restoring force of a spring of
spring constant 100 N/m. A block of mass 3 kg is 15. Two identical springs are attached to a small
gently placed on it at the instant it passes through block P. The other ends of the springs are fixed
the mean position. Assuming that the two blocks at A and B. When P is in equilibrium the extension
move together, find the frequency and the of top spring is 20 cm and extension of bottom
amplitude of the motion. spring is 10 cm. Find the period of small vertical
oscillations of P about its equilibrium position. (use
g = 9.8 m/s2)
3kg
A
1kg

Sol. P

B
Sol.

: 0744-2209671, 08003899588 | url : www.motioniitjee.com, : info@motioniitjee.com


Page # 64 S.H.M.

18. Two blocks A (5kg) and B(2kg) attached to


16. The figure shows the displacement - time
the ends of a spring constant 1120 N/m are placed
graph of a particle executing SHM. If the time on a smooth horizontal plane with the spring
undeformed. Simultaneously velocities of 3m/s and
period of oscillation is 2s, then the equation of 10m/s along the line of the spring in the same
direction are imparted to A and B then
motion is given by x = ................. .
3m/s 10m/s
x
A 5 2 B
10 (a) Find the maximum extension of the spring.
mm (b) When does the first maximum compression
5 occurs after start.
0 Sol.
t(s)

Sol.

19. The motion of a particle is described by x =


30 sin (t + /6), where x is in cm and t in sec.
Potential energy of the particle is twice of kinetic
energy for the first time after t = 0 when the
particle is at position ............. after .......... time.
17. Two particles A and B execute SHM along Sol.
the same line with the same amplitude a, same
frequency and same equilibrium position O. If the
phase difference between them is  = 2 sin–1 (0.9),
then find the maximum distance between the two.
Sol.

20. A particle is performing SHM with accleration


a = 8 2 – 4 2 x where x is coordinate of the
particle w.r.t. the origin. The parameters are in
S.I. units. The particle is at rest at x = –2 at t =
0. Find coordinate of the particle w.r.t. origin at
any time.

Corporate Head Office : Motion Education Pvt. Ltd., 394 - Rajeev Gandhi Nagar, Kota-5 (Raj.)
S.H.M. Page # 65

Sol. Sol.

23. A body of mass 1 kg is suspended from a


21. (a) Find the time period of oscillations of a weightless spring having force constant 600 N/
torsional pendulum, if the torsional constant of m. Another body of mass 0.5 kg moving vertically
the wire is K = 102J/rad. The moment of inertia upwards hits the suspended body with a velocity
of rigid body is 10 kg m2 about the axis of rota- of 3.0 m/s and get embedded in it. Find the fre-
tion. quency of oscillations and amplitude of motion.
(b) A simple pendulum of length l = 0.5 m is hanging Sol.
from ceiling of a car. The car is kept on a hori-
zontal plane. The car starts accelerating on the
horizontal road with acceleration of 5 m/s2. Find
the time period of oscillations of the pendulum
for small amplitudes about the mean position.
Sol.

24. A block is kept on a horizontal table. The


table is undergoing simple harmonic motion of fre-
22. An object of mass 0.2 kg executes SHM along quency 3 Hz in a horizontal plane. The coeffi-
the x-axis with frequency of (25/) Hz. At the cient of static friction between block and the
point x = 0.04m the object has KE 0.5 J and PE table surface is 0.72. Find the maximum ampli-
0.4 J. The am pl i t ud e of osc i l l ati on i s tude of the table at which the block does not slip
_____________. on the surface.

: 0744-2209671, 08003899588 | url : www.motioniitjee.com, : info@motioniitjee.com


Page # 66 S.H.M.

Sol. 27. The angle made by the string of a simple


pendulum with the vertical depends on time as 

= sin[s–1)t]. Find the length of the pendulum
90
if g = 2 m/s2.
Sol.

25. A particle of mass m moves in a one-dimen-


sional potential energy U(x) = –ax2 + bx4, where
‘a’ and ‘b’ are positive constants. Then what is
the angular frequency of small oscillations about
the minima of the potential energy.
Sol.

28. A pendulum is suspended in a lift and its period


of oscillation is T0 when the lift is stationary.
(i) What will the period T of oscillation of pendulum
be, if the lift begins to accelerate downwards
3g
with an acceleration equal to ?
4
(ii) What must be the acceleration of the lift for
the period of oscillation of the pendulum to be
T0
?
2
Sol.
26. A pendulum having time period equal to two
seconds is called a seconds pendulum. Those used
in pendulum clocks are of this type. Find the length
of a seconds pendulum at a place where g = 2
m/s2
Sol.

Corporate Head Office : Motion Education Pvt. Ltd., 394 - Rajeev Gandhi Nagar, Kota-5 (Raj.)
S.H.M. Page # 67

29. A simple pendulum of length  is suspended 31. Two identical rods each of mass m and length
through the ceiling of an elevator. Find the time L, are rigidly joined and then suspended in a ver-
period of small oscillations if the elevator (a) is tical plane so as to oscillate freely about an axis
going up with an acceleration a0 (b) is going down normal to the plane of paper passing through ‘S’
with an acceleration a0 and (c) is moving with a (point of supension). Find the time period of such
uniform velocity. small oscillations.
Sol.
S

Sol.

32. A simple pendulum has a time period T = 2


30. A simple pendulum fixed in a car has a time
sec when it swings freely. The pendulum is hung
period of 4 seconds when the car is moving
as shown in figure. so that only one-fourth of its
uniformly on a horizontal road. When the
total length is free to swing to the left of ob-
accelerator is pressed, the time period changes
stacle. It is displaced to position A and released.
to 3.99 seconds. Making an approximate analysis,
How long does it take to swing to extreme dis-
find the acceleration of the car.
placement B and return to A? Assume that dis-
Sol.
placement angle is always small.
cle
sta
ob

B A
Sol.

: 0744-2209671, 08003899588 | url : www.motioniitjee.com, : info@motioniitjee.com


Page # 68 S.H.M.

1. A point particle of mass 0.1 kg is executing


SHM with amplitude of 0.1 m. When the particle
passes through the mean position, its K.E. is 8 ×
10–3J. Obtain the equation of motion of this par-
ticle if the initial phase of oscillation is 45°.
2. The particle executing SHM in a straight line
has velocities 8 m/s, 7 m/s, 4 m/s at three points
distant one metre from each other. What will be (a) Calculate the frequency of oscillation of the
the maximum velocity of the particle? ball B.
3. One end of an ideal spring is fixed to a wall at (b) What is the total energy of the system.
origin O and the axis of spring is parallel to a x- (c) Find the speed of the ball A when A and B are
axis. A block of mass m = 1 kg is attached to free at the two ends of the diameter PQ.
end of the spring and it is performing SHM. Equa- 6. An ideal gas is enclosed in a vertical cylinderical
tion of position of block in coordinate system container and supports a freely moving piston of
shown is x = 10 + 3 sin10t, t is in second and x in mass m. The piston and the cylinder have equal
cm. Another block of mass M = 3kg, moving to- cross-sectional area A, atmospheric pressure is
wards the origin with velocity 30 cm/s collides P0 and when the piston is in equilibrium position.
with the block performing SHM at t = 0 and gets Show that the piston executes SHM and find
struck to it, calculate : the frequency of oscillation (system is completely
(i) new amplitude of oscillations. isolated from the surrounding).  = Cp/Cv. Height
(ii) new equation for position of the combined of the gas in equilibrium position is h.
body. 7. A massless rod is hinged at O. A string carry-
(iii) loss of energy during collision. Neglect fric- ing a mass m at one end is attached to point A
tion. on the rod so that OA = a. At another point B
1kg 3kg (OB = b) of the rod, a horizontal spring of force
constant k is attached as shown. Find the period
of small vertical oscilla tions of mass m around its
4. A mass M is in static equilibrium on a massless equilibrium position.
vertical spring as shown in the figure. A ball of
mass m dropped from certain height sticks to the k B
mass M after colliding with it. The oscillations
they perform reach to height ‘a’ above the origi- A
nal level of scales & depth ‘b’ below it.
(a) Find the force constant of the spring.; m
O
8. Two blocks A (2kg) and B(3kg) rest up on a
M a smooth horizontal surface are connected by a
b spring of stiffness 120 N/m. Initially the spring is
underformed. A is imparted a velocity of 2m/s
along the line of the spring away from B. Find the
displacement of A t second later.
3kg 2kg 2m/s
(b) Find the oscillation frequency. B A
(c) What is the height above the initial level from 9. Consider a fixed ring shaped uniform body of
which the mass m was droped ? linear mass density  and radius R. A particle at
5. Two identical balls A and B each of mass 0.1 the centre of ring is displaced along the axis by a
kg are attached to two identical massless springs. small distance, show that the particle will ex-
The spring mass system is constrained to move ecute SHM under gravitation of ring & find its
inside a rigid smooth pipe in the form of a circle time period neglecting other forces.
as in fig. The pipe is fixed in a horizontal plane.
The centres of the ball can move in a circle of
radius 0.06 m. Each spring has a natural length
0.06 m and force constant 0.1 N/m. Initially both
t he b al l s are d i s pl ac ed b y an ang l e of
 =/6 radian with respect to diameter PQ of the
circle and released from rest

Corporate Head Office : Motion Education Pvt. Ltd., 394 - Rajeev Gandhi Nagar, Kota-5 (Raj.)
S.H.M. Page # 69

Exercise - IV PREVIOUS YEAR QUESTIONS

LEVEL - I JEE MAIN

1. If a spring has time period T, and is cut into n Sol.


equal parts, then the time period of each part
will be [AIEEE 2002]
T
(A) T n (B) (C) nT (D) T
n
Sol.

4. A body executes simple harmonic motion. The


potential energy (PE), the kinetic energy (KE)
and total energy (TE) are measured as function
of displacement x. Which of the following
statements is true ? [AIEEE 2003]
(A) KE is maximum when x = 0
(B) TE is zero when x = 0
2. In a simple harmonic oscillator, at the mean (C) KE is maximum when x is maximum
position [AIEEE 2002] (D) PE is maximum when x = 0
(A) kinetic energy is minimum, potential energy Sol.
is maximum
(B) both kinetic and potential energies are
maximum
(C) kinetic energy is maximum, potential energy
is minimum
(D) both kinetic and potential energies are
minimum
Sol.

5. The length of a simple pendulum executing


simple harmonic motion is increased by 21%. The
percentage increase in the time period of the
pendulum of increased length is [AIEEE 2003]
(A) 11% (B) 21% (C) 42% (D) 10.5%
Sol.

3. A child swinging on a swing in sitting position,


stands up, then the time period of the swing will
[AIEEE 2003]
(A) increase (B) decrease
(C) remain same
(D) increase if the child is long and decrease if the
child is short

: 0744-2209671, 08003899588 | url : www.motioniitjee.com, : info@motioniitjee.com


Page # 70 S.H.M.

6. Two particles A and B of equal masses are 8. The total energy of a particle, executing simple
suspended from two massless springs of spring harmonic motion is [AIEEE 2004]
constants k1 and k2, respectively. If the maximum (A)  x (B)  x2
velocities, during oscillations are equal, the ratio
of amplitudes of A and B is [AIEEE 2003] (C) independent of x (D)  x1/ 2
where x is displacement from the mean position
k1 k1 k2 k2 Sol.
(A) (B) (C) (D)
k2 k2 k1 k1
Sol.

9. A particle at the end of a spring executes


simple harmonic motion with a period t1, while
the corresponding period for another spring is t2.
7. A mass M is suspended from a spring of If the period of oscillation with the two springs in
negligible mass. The spring is pulled a little and series is T, then [AIEEE 2004]
then released so that the mass executes SHM of
time period T. If the mass is increased by m, the (A) T  t1  t 2 (B) T 2  t12  t 22
m (C) T 1  t11  t 21 (D) T  2  t1 2  t 2 2
time period becomes 5T/3, then the ratio of is
M Sol.
[AIEEE 2003]

3 25 16 5
(A) (B) (C) (D)
5 9 9 3
Sol.

10. The bob of a simple pendulum executes simple


harmonic motion in water with a period t, while
the period of oscillation of the bob is t0 in air.
Neglecting frictional force of water and given that
the density of the bob is (4/3) × 1000 kgm–3 .
What relationship between t and t0 is true ?
[AIEEE 2004]

t0
(A) t  t0 (B) t 
2
(C) t  2 t0 (D) t  4 t0

Corporate Head Office : Motion Education Pvt. Ltd., 394 - Rajeev Gandhi Nagar, Kota-5 (Raj.)
S.H.M. Page # 71

Sol.

13. The function sin 2


ωt  represents
[AIEEE 2005]
(A) a periodic, but not simple harmonic, motion
with a period 2 π/ω
(B) a periodic, but not s7imple harmonic
with a period π/ω
(C) a si mple harmonic motion with a period
2 π/ω
11. If a simple harmonic motion is represented (D) a simple harmonic motion with a period
d2x π/ω
by  αx  0 , its time period is Sol.
dt 2
[AIEEE 2005]

2π 2π
(A) (B) (C) 2 πα (D) 2 π α
α α
Sol.

14. Starting from the origin a body oxcillates


simple harmonically with a period of 2 s. After
what time will its kinetic energy be 75% of the
total energy ? [AIEEE 2006]

1 1 1 1
(A) s (B) s (C) s (D) s
6 4 3 12
Sol.

12. Two simple harmonic motions are represented

 π
by the equations y1 = 0.1 sin 100 πt   and y2
 3
= 0.1 cos πt . The phase difference of the
velocity of particle1, with respect to the velocity
of particle 2 is [AIEEE 2005]

π π π π
(A) (B) (C) (D)
6 3 3 3
Sol.

: 0744-2209671, 08003899588 | url : www.motioniitjee.com, : info@motioniitjee.com


Page # 72 S.H.M.

15. The maximum velocity of a particle, executing 17. A particle of mass m executes simple harmonic
simple harmonic motion with an amplitude 7 mm, mot ion with amplitude a and frequency v. The
is 4.4 ms–1. The period of oscillation is average kinetic energy during its motion from the
[AIEEE 2006] position of equilibrium to the end is
(A) 0.01 s (B) 10 s (C) 0.1 s (D) 100 s [AIEEE 2007]
Sol.
1
(A) 2 ma2 v2 (B) ma2 v2
4
(C) 42 ma2 v2 (D) 22 ma2 v2
Sol.

18. Two springs, of force constants k1 and k2,


are connected to a mass m as shown. The
16. The displacement of an object attached to a frequency of oscillation of the mass is f. If both
spring and executing simple harmonic motion is k1 and k2 are made four times their original values,
given by x  2 10  2 cos πt metre. The time at the frequency of oscillation becomes
[AIEEE 2007]
which the maximum speed first occurs is
[AIEEE 2007] K1 K2
(A) 0.5 s (B) 0.75 s m
(C) 0.125 s (D) 0.25 s
Sol. f f
(A) (B) (C) 4f (D) 2f
2 4
Sol.

Corporate Head Office : Motion Education Pvt. Ltd., 394 - Rajeev Gandhi Nagar, Kota-5 (Raj.)
S.H.M. Page # 73

19. A point masss oscillates along the x-axis 21. A mass , attached to a horizontal spring,
according to the law x  x0 cos  ωt  / 4 . If the executes SHM with amplitude A1. When the mass
M passes through its mean position then a smaller
accel eration of the particl e i s wri tten as
mass m is placed over it and both of them move
a  Acos ωt  δ  , then [AIEEE 2007]
 A1 
π 2 π together with amplitude A2. The ratio of   is
(A) A  x 0 , δ   (B) A  x 0 ω , δ   A2 
4 4
[AIEEE 2011]
2 π 2 3π
(C) A  x 0 ω , δ   (D) A  x 0 ω , δ  M m  M 
1/ 2
4 4 (A) (B)  
Sol. M  M m 
1/ 2
 M m  M
(C)   (D)
 M  M m
Sol.

20. If x, v and a denote the displacement, the


velocity and the acceleration of a particle
executing, simple harmonic motion of time period 22. Two particles are executing simple harmonic
of T, then, which of the following does not change motion of the same amplitude A and frequency
with time? [AIEEE 2009] ω along the x-axis. Their mean position is
separated by distance X0 (X0 > A). If the maximum
aT
(A) a 2 T 2  4 π 2 v 2 (B) separation between them is (X0+A), the phase
x difference between their motion is
[AIEEE 2011]
aT
(C) aT 2 πv (D)
v π π π π
(A) (B) (C) (D)
Sol. 3 4 6 2
Sol.

: 0744-2209671, 08003899588 | url : www.motioniitjee.com, : info@motioniitjee.com


Page # 74 S.H.M.

23. If a spring of stiffness k is cut into two parts 24. T hi s q ues t i on has s t at e me nt 1 and
A and B of length lA : lB = 2 : 3, then the stiffness statement2. Of the four choices given after the
of spring A is given by [AIEEE 2011] statement, choose the one that best describes
the two statements.
5 3k 2k If two springs S1 and S2 of force contants K1 and
(A) k (B) (C) (D) k
2 5 5 K2, respectively are stretched by the same force,
Sol. it is found that more work is done on spring S1
than on spring S2. [AIEEE 2012]
Statement 1 If stretched by the same amount,
work done on S1, will be more than that on S2.
Statement 2 k1 < k2
(A) Statement 1 is false, Statement 2 is true.
(B) Statement 1 is true, Statement 2 is false.
(C) Statement 1 is true, Statement 2 is true
Statement 2 is the correct explanation for
Statement 1
(D) Statement 1 is true, Statement 2 is true,
Statement 2 is not the correct explanation of
Statement 1
Sol.

Corporate Head Office : Motion Education Pvt. Ltd., 394 - Rajeev Gandhi Nagar, Kota-5 (Raj.)
S.H.M. Page # 75

LEVEL - II JEE ADVANCED


1. A particle is executing SHM according to y = a 3. A block P of mass m is placed on a frictionless
cos t. Then which of the graphs represents varia- horizontal surface. Another block Q of same mass
tions of potential energy : [JEE (Scr)’ 2003] is kept on P and connected to the wall with the
help of a spring of spring constant k as shown in
P.E. the figure. s is the coefficient of friction be-
I II
tween P and Q. The blocks move together per-
forming SHM of amplitude A. The maximum value
of the friction force between P and Q is

Q s
k
t
P smooth
P.E.
(A) (I) & (III) III IV
(B) (II) & (IV) kA
(A) kA (B)
(C) (I) & (IV) 2
(D) (II) & (III) (C) zero (D) smg [JEE’ 2004]
Sol. x Sol.

2. Two masses m1 and m2 connected by a light 4. A simple pendulum has time period T1. When
spring of natural length l0 is compressed com- the point of suspension moves vertically up ac-
pletely and tied by a string. This system while cording to the equation y = kt2 where k = 1 m/s2
moving with a velocity v0 along +ve x-axis pass and ‘t’ is time then the time period of the pendu-
through the origin at t = 0. At this position the
string snaps. Position of mass m1 at time is given 2
 T1 
by the equation.x1(t) = v0 t – A (1 – cos t) lum is T2 then   is [JEE’ 2005(Scr)]
Calculate :  T2 
(a) Position of the particle m2 as a function of
time. 5 11 6 5
(b) l0 is terms of A. (A) (B) (C) (D)
6 10 5 4
[JEE’ 2003]
Sol.
Sol.

: 0744-2209671, 08003899588 | url : www.motioniitjee.com, : info@motioniitjee.com


Page # 76 S.H.M.

5. A small body attached to one end of a vertically Sol.


hanging spring is performing SHM about
it’s mean position with angular frequency  and
amplitude a. If at a height y*
from the mean position the body
gets detached from the spring,
calculate the value of y* so
that the height H attained by y0
the mass is maximum. The
m
body does not interact with the
spring during it’s subsequent motion after
detachment. (aw2>g). [JEE’ 2005]
Sol.

8. Colum n I describes some situations in which a


small objact moves. Column II describes some
characteristics of these motions. Match the situ-
ations in Column I with the characteristics in
Column II and indicate your answer by darken-
ing appropriate bubbles in the 4 × 4 matrix given
in the ORS.
Column I Column II
6. Function x = A sin2 t + B cos2 t + C sin t cos (A) The object moves (P) The object
t represents SHM [JEE’ 2006] on the x-axis under a executes a SHM
(A) for any value of A, B and C (except C = 0) conservative force
(B) if A = – B ; C = 2B, amplitude = |B 2 | in such a way that
(C) if A = B; C = 0 its “speed” and “po
(D) if A = B; C = 2B, amplitude = |B| sition” satisfy v =
Sol.
c1 c 2  x2 , where
c1 and c2 are positive
constants.
(B) The object moves (Q) The object does
on the x-axis in such a not change its
way that its velocity and direction
its displacement from the
origin satisfyv = –kx,
7. A student performs an experiment for determi- where k is a positive
constant.
 4 2 l 
(C) The object is attached (R) The kinetic en
nation of g    l  1 m and he commits an
 T 2  to one end of a massless ergy of the object
spring of a given spring keeps on decreasing
error of l.
constant. The other end
For The takes the time of n oscillations with the
of the spring is attached
stop watch of least count T and he commits a
to the ceiling of an elevator.
human error of 0.1 sec. For which of the
Initially everything is at rest.
following data, the measurement of g will be most
The elevator starts going
accurate? [JEE’ 2006]
upwards with a constant
l T nAmplitude of oscillation
acceleration a. The motion
(A)5 mm 0.2 sec 10 5 mm
of the object is observed
(B) 5 mm 0.2 sec 20 5 mm
from the elevator during the
(C) 5 mm 0.1 sec 20 1 mm
period it maintains this
(D) 1 mm 0.1 sec 50 1 mm
acceleration.

Corporate Head Office : Motion Education Pvt. Ltd., 394 - Rajeev Gandhi Nagar, Kota-5 (Raj.)
S.H.M. Page # 77

(D) The object is projected (S) The object 10. The x–t graph of particle undergoing simple
from the earth’s surface can change its harmonic motion is shown below. The accelera-
vertically upwards with direction only once tion of the particle at t = 4 / 3 s is[JEE’ 2009]

a speed 2 GMe / R e , where


Me is the mass of the earth
and Re is the radius of the
earth. Neglect forces from
objects other than the earth.
[JEE’ 2007]
Sol.

3 2 – 2
(A)  cm / s 2 (B) cm / s 2
32 32

2 3 2
(C) cm / s 2 (D) –  cm / s 2
32 32
Sol.

9. A block (B) is attached to two unstretched


springs S1 and S2 with spring constants k and
4k, respectively (see figure I). The other ends
are attached to identical supports M1 and M2
not attached to the walls. The springs and
supports have negligible mass. There is no friction
anywhere. The block B is displaced towards wall
1 by a small distance x (figure II) and released.
The block returns and moves a maximum distance
11. The mass M shown in the figure oscillates in
y towards wall 2. Displacement x and y are
measured with respect to the equilibrium position simple harmonic motion with amplitude A. The
amplitude of the point P is [JEE’ 2009]
y
of the block B. The ratio in Figure
x
K1 K2
P M

k2A k2A
(A) k (B) k
2 2

[JEE’2008] k 1A k 2A
1 1 (C) k  k (D) k  k
(A) 4 (B) 2 (C) (D) 1 2 1 2
2 4 Sol.
Sol.

: 0744-2209671, 08003899588 | url : www.motioniitjee.com, : info@motioniitjee.com


Page # 78 S.H.M.

12. A uniform rod of length L and mass M is pivoted Sol.


at the centre. Its two ends are attached to two
springs of equal spring constants k. The springs
are fixed to rigid supports as shown in the figure,
and the rod is free to oscillate in the horizontal
plane. The rod is gently pushed through a small
angle  in one direction and released. The
frequency of oscillation is [JEE’ 2009]

Paragraph for Question Nos. 14 to 16


When a particle of mass m moves on the
1 2k 1 k x-axis in a potential of the from V(x) = kx2,
(A) (B) it performs simple harmonic motion. The
2 M 2 M
corresponding time period is proportional to
1 6k 1 24k
(C) (D) m
2 M 2 M , as can be seen easily using dimensional
k
Sol.
analysis. However, the motion of a particle
can be periodic even when its potential energy
increases on both sides of x = 0 in a way
different from kx2 and its total energy is such
that the particle does not escape to infinity.
Consider a particle of mass m moving on the
x-axis. Its potential energy is V(x) = x 4
( > 0) for |x| near the origin and becomes
a constant equal to V0 for |x|  X0 (see
figure). [JEE 2010]
13. A metal rod of length 'L' and mass 'm' is piv-
oted at one end. A thin disk of mass 'M' and V(x)
radius 'R' (<L) is attached at its center to the
free end of the rod. Consider two ways the disc V0
is attached : (case A) The disc is not free to
x
rotate about its center and (case B) the disc is X0
free to rotate about its center. The rod-disc sys-
tem performs SHM in vertical plane after being 14. If the total energy of the particle is E, it
released from the same displaced position. which will perform periodic motion only if :
of the following statement(s) is (are) true? (A) E < 0 (B) E > 0
(C) V0 > E > 0 (D) E > V0
Sol.

(A) Restoring torque in case A = Restoring torque


in case B 15. For periodic motion of small amplitude A,
(B) Restoring torque in case A < Restoring torque the time period T of this particle is proportional
in case B to :
(C) Angular frequency for case A > Angular fre-
m 1 m  1 
quency for case B (A) A (B) (C) A (D)
(D) Angular frequency for case A < Angular fre-  A  m A m
quency for case B [JEE’ 2011]
Corporate Head Office : Motion Education Pvt. Ltd., 394 - Rajeev Gandhi Nagar, Kota-5 (Raj.)
S.H.M. Page # 79

Sol. is a plane in which position is plotted along


horizontal axis and momentum is plotted along
vertical axis. The phase space diagram is x(t) vs.
p(t) curve in this plane. The arrow on the curve
idicates the time flow. For example, the phase
space diagram for a particle moving with constant
velocity is a straight line as shown in the figure.
We use the sign convention in which position of
16. The acceleration of this parti cle for momentum upwards (or to right) is positive and
|x| > X0 is : downwards (or to left) is negative.[JEE’ 2012]
(A) proportional to V0
V0
(B) proportional to
mX0

Momentum
V0
(C) proportional to
mX0

(D) Zero
Sol.
Position

18. The phase space diagram for a ball thrown


vertically up from ground is

Momentum Momentum
17. A poi nt mass i s subje cte d t o t wo
simultaneous sinusoidal displacements in
x-direction, x 1(t) = A sin t and x 2(t) = A sin
 2  (A) (B)
Position Position
 t  3  . Add i ng a thi rd s i nus oi dal
 
displacement x 3(t) = B sin (t + ) brings the
mass to a complete rest. The values of B and Momentum
Momentum
 are : [JEE 2011]

3 4
(A) 2A, (B) A,
4 3
(C) (D)
Position Position
5 
(C) 3A, (D) A,
6 3
Sol.
Sol.

Paragraph for Question Nos. 18 to 20


Phase space diagrams are useful tools in analyzing
all kinds of dynamical problems. They are
especially useful in studying the changes in motion
as initial position and momentum are changed.
Here we consider some simple dynamical systems
in one-dimension. For such systems, phase space
: 0744-2209671, 08003899588 | url : www.motioniitjee.com, : info@motioniitjee.com
Page # 80 S.H.M.

19. The phase space diagram for simple harmonic


motion is a circle centered at the origin. In the
figure, the two circles represent the same
oscillator but for different initial conditions, and
E1 and E 2 are the total Momentum

me chani c al e ne rg i e s
respectively. Then E2
E1

(A) E1 = 2 E2 2a

(B) E1 =2E2 a position

(C) E1 = 4E2
(D) E1 = 15E2
Sol.

21. A particle of mass m is attached to one end


of a mass-less spring of force constant k, lying
on a frictionless horizontal plane. The other end
of the spring is fixed. The particle starts moving
horizontally from its equilibrium position at time
t= 0 with an initial velocity u0. When the speed
of the particle of 0.5 u0, it collides elastically
with a rigid wall. After this collision.
[JEE Advance-2013]
(A) the speed of the particle when it returns to
its equilibrium position is u0.
(B) the time at which the particle passes through
the equilibrium position for the first time is t =

20. Consider the spring-mass system, with the m



mass submerged in water, as shown in the figure. k
The phase space diagram for one cycle of this (C) the time at which the maximum compression
system is
4 m
of the spring occurs is t 
3 k
(D) the time at which the particle passes through
the equilibrium position for the second time is
5 m
t .
3 k
Sol.
Momentum
Momentum

(A) Position
(B)
Position

Momentum
Momentum

(C) Position
(D) Position

Sol.
Corporate Head Office : Motion Education Pvt. Ltd., 394 - Rajeev Gandhi Nagar, Kota-5 (Raj.)
S.H.M. Page # 81

:: ANSWER KEY ::

Exercise - I OBJECTIVE PROBLEMS (JEE MAIN)


1. A 2. C 3. B 4. C 5. C 6. A 7. A
8. C 9. A 10. D 11. B 12. B 13. A 14. C
15. B 16. A 17. C 18. D 19. C 20. B 21. B
22. C 23. C 24. C 25. C 26. D 27. A 28. D
29. D 30. D 31. B 32. B 33. C 34. A 35. A
36. C 37. B 38. A 39. B 40. C 41. B 42. C
43. A 44. A 45. C 46. A

Exercise - II
1. C 2. D 3. B 4. D 5. D 6. D 7. C
8. B 9. C 10. C 11. A 12. B 13. B 14. A
15. C 16. A 17. C 18. C 19. B 20. C 21. A
22. B 23. C 24. A 25. D 26. B,C,D 27. A 28. B
29. B,C 30. A 31. A,B,C 32. C 33. B 34. A,B,C,D 35. B,C,D
36. A,B 37. A,B,C 38. B,D 39. A 40. C,D 41. B,C,D 42. A,B,C
43. A,C 44. B,D 45. B 46. B,D 47. A,C 48. B,C 49. B,C
50. B

Exercise - III (JEE ADVANCED)

LINEAR S.H.M

1. Amplitude = 5 m
Initial Phase = /6
Maximum speed = 5m/sec
2. (a) 2.0 cm, /50 sec, 100 N/m

(b) 1 cm, 3 m / sec , 100 m/sec–1

   1 
3. (a) sec. , (b) sec. (c) sec. 4. 5. (a) sec , (b) 4 cm, (c) 2.40 kg m/sec
120 30 30 3 10

1  11  11
6. 2m/sec 7. 8. 100 Nm–2 9. (a) m (b) sec (c) x = 0.2 – cos t
2  5 5 5

3 5 –1  1 
10. (i) x0 = 2m (ii) T = 2 sec (iii) 2 3 11. A , tan   12. 25 2 N
8 2

5 F(K 2  K 3 ) 1 K 1K 2  K 2K 3  K 3K 1 
13. Hz , 5 cm 14. K K  K K  K K , 2 15.
2 1 2 2 3 3 1 M (K 2  K 3 ) 7
3
16. X = 10 sin (t + /6 ) 17. 1.8 a 18. (a) 25 cm, (b) seconds
56

: 0744-2209671, 08003899588 | url : www.motioniitjee.com, : info@motioniitjee.com


Page # 82 S.H.M.

1 2 1 2
19. 10 6 cm , sin –1 – sec 20. 2 – 4 cos 2t 21. (a) 2 sec. (b) T = sec.
 3 6 51/ 4

10 5 37 a
22. 0.06 m 23. Hz , cm 24. 2 cm 25. 2
 6 m

ANGULAR S.H.M

  
26 1m 27. 1m 28. (i) 2T0 , (ii) 3 g up wards 29. (a) 2 a  g , (b) 2 g – a , (c) 2
0 0 g

g 17L 3
30. 31. 2 18 g 32. sec
10 2

1. y = 0.1 sin (4t +/4) 2. 65 m / s 3. 3cm, x = 10 – 3 sin 5t; E = 0.135 J

2mg  M  m  ab 1 2mg 1
4. (a) K = ; (c)  m  b – a , 5. f= ; E = 42 × 10–5 J; v =2 × 10–2 m/s
b–a 2 (b – a)(M  m) 

1  (P0  mg / A )A 2R 2
6. f 7. (2a/b)(m/k)1/2 8. 0.8t + 0.12 sin 10t 9.
2 mh Gp

Exercise - IV PREVIOUS YEAR QUESTIONS

LEVEL - I JEE MAIN


1. B 2. C 3. B 4. A 5. D 6. C 7. C
8. C 9. B 10. C 11. B 12. A 13. B 14. A
15. A 16. A 17. A 18. D 19. D 20. B 21. C
22. A 23. A 24. A

LEVEL - II JEE ADVANCED

m1  m1 
1. A 2. (a) x2 = v0t + A (1 – cos t), (b) l0 =  m  1 A 3. B 4. C
m2  2 

mg g
5. y* =  2 <a 6. A,B,D 7. D
k 
8. (A) P; (B) Q, R ; (C) P ; (D) Q, R or (A) P ; (B) Q, R ; (C) P ; (D) R
9. C 10. D 11. D 12. C 13. A,D
14. C 15. B 16. D 17. B
18. D 19. C 20. B 21. A,D

Corporate Head Office : Motion Education Pvt. Ltd., 394 - Rajeev Gandhi Nagar, Kota-5 (Raj.)
WAVES Page # 83

WAVES
1. WAVES :

Waves is distributed energy or distributed "disturbance (force)"

• Following points regarding waves :


1. The disturbance (force) is transmitted from one point to another.
2. The energy is transmitted from one point to another.
3. The energy or distrubance passes in the form of wave without any net displacement of
medium.
4. The oscillatory motion of preceding particle is imparted to the adjacent particle following it.
5. We need to keep creating disturbance in order to propagate wave (energy or disturbance)
continuously.

(a) Waves classification


The waves are classified under two high level headings :
1. Mechanical waves : The motion of the particle constituting the medium follows mechanical
laws i.e. Newton's laws of motion. Mechanical waves originate from a distrubance in the
medium (such as a stone dropping in a pond) and the disturbance propagates through the
medium. The force between the atoms in the medium are responsible for the propagation of
mechanical waves. Each atom exerts a force on the atoms near it, and through this force the
motion of the atom is transmitted to the others. The atoms in the medium do not experience
any net displacement.
Mechanical waves is further classified in two categories such that
1. Transverse waves (waves on a string)
2. Longitudnal waves (sound waves)

2. Non Mechanical waves : These are electro magnetic waves. The electromagnetic waves do
not require a medium for propagation. Its speed in vacuum is a universal constant. The
motion of the electromagnetic waves in a medium depends on the electromagnetic properties
of the medium.

2.1 Transverse waves


If the disturbance travels in the x direction but the particles move in a direction, perpendicular
to the x axis as the wave passes it is called a transverse waves.

v
y T
T

2Tsin

y x
V

T T
O x

figure - I

: 0744-2209671, 08003899588 | url : www.motioniitjee.com, : info@motioniitjee.com


Page # 84 WAVES

Consider a sinusoidal harmonic wave travelling through a string and the motion of a particle
as shown in the figure Ist (only one unit of wave shown for illustration purpose). Since the
particle is displaced from its natural (mean) position, the tension in the string arising from
the deformation tends to restore the position of the particle. On the other hand, velocity of
the particle (kinetic energy) move the particle farther is zero. Therefore, the particle is pulled
down due to tension towards mean position. In the process, it acquires kinetic energy (greater
speed) and overshoots the mean position in the downward direction. The cycle of restoration
of position continues as vibration (oscillation) of particle takes place.

2.2 Longitudinal waves


Longitudinal waves are characterized by the direction of vibration (disturbance) and wave
motion. They are along the same direction. It is clear that vibration in the same direction
needs to be associated with a "restoring" mechanism in the longitudinal direction.

(b) Mathematical description of waves


We shall attempt here to evolve a mathematical model of a travelling transverse wave. For
this, we choose a specific set up of string and associated transverse wave travelling through
it. The string is tied to a fixed end, while disturbance is imparted at the free end by up and
down motion. For our purpose, we consider that pulse is small in dimension; the string is
light, elastic and homogeneous. The assumptions are required as we visualize a small travelling
pulse which remains undiminished when it moves through the strings. We also assume that
the string is long enough so that our observation is not subjected to pulse reflected at the
fixed end.
For understanding purpose, we first consider a single pulse as shown in the figure (irrespective
of whether we can realize such pulse in practice or not). Our objective here is to determine
the nature of a mathematical description which will enable us to determine displacement
(disturbance) of string as pulse passes through it. We visualize two snap shots of the travelling
pulse at two close time instants "t" and "t + t". The single pulse is moving towards right in
the positive x-direction.

Y 12 3

t=t
O x

t  t  t
O x

The vibration and wave motion are at right angle to each other.
Three position along x-axis named "1", "2" and "3" are marked with three vertical dotted
lines. At either of two instants as shown, the positions of string particles have different
displacements from the undisturbed position on horizontal x-axis. We can conclude from this
observation that displacement in y-direction is a function of positions of particle in x-direction.
As such, the displacement of a particle constituting the string is a function of "x".
Let us now observe the positions of a given particle, say "1". It has certain positive displacement
at time t = t, At the next snapshot at t = t + t, the displacement has reduced to zero. The
particle at "2" has maximum displacement at t = t, but the same has reduced at t = t + t.
The third particle at "3' has certain positive displacement at t = t, At t = t + t, it acquires
additional positive displacement and reaches the position of maximum displacement. From
these observation, we conclude that displacement of a particle at any position along the
string is a function of "t".

Corporate Head Office : Motion Education Pvt. Ltd., 394 - Rajeev Gandhi Nagar, Kota-5
WAVES Page # 85

Combining two observations, we conclude that displacment of a particle is a function of both


position of the particle along the string and time.
y = f (x, t)
We can further specify the nature of the mathematical function by association the speed of
the wave in our consideration. Let "v" be the constant speed with which wave travels from
the left end to the right end. We notice that wave function at a given position of the string is
a function of time only as we are considering displacement at a particular value of "x". Let us
consider left hand end of the string as the origin of reference (x = 0 and t = 0). The displacement
in y-direction (disturbance) at x = 0 is a function of time, "t" only :
y = f(t) = A sin t
The disturbance travels to the right at constant speed "v'. Let it reaches a point specified as
x = x after time "t". If we visualize to describe the origin of this disturbance at x = 0, then
time elapsed for the distrubance to move from the origin (x = 0) to the point (x = x) is "x/v".
Therefore, if we want to use the function of displacement at x = 0 as given above, then we
need to subtract the time elapsed and set the equation is :

 x  x
y  f  t –   A sin   t – 
 v  v

This can also be expressed as

 vt – x   x – vt 
 f   –f  
 v   v 

y(x, t) = g(x – vt)


using any fixed value of t (i.e. at any instant), this shows shape of the string.
If the wave is travelling in –x direction, the wave equation is written as

x
y (x, t) = f ( t  )
v
The quantity x – vt is called phase of the wave function. As phase of the pulse has fixed value
x – vt = const.

dx
Taking the derivative w.r.t. time v
dt
where v is the phase velocity although often called wave velocity. It is the velocity at which
a particular phase of the distrubance travels through space.
In order for the function to represent a wave travelling at speed v, the quantities x, v and t
must appear in the combination (x + vt) or (x – vt). Thus (x – vt)2 is acceptable but x2 – v2
t2 is not.

(c) Describing Waves :


Two kinds of graph may be drawn displacement - distance and displacement-time.
A displacement-distance graph for a transverse mechanical waves shows the displacement y
of the vibrating particles of the transmitting medium at different distance x from the source
at a certain instant i.e. it is like a photograph showing shape of the wave at that particular
instant.
The maximum displacement of each particle from its undisturbed position is the amplutude
of the wave.
In the figure 1, it OA or OB.

: 0744-2209671, 08003899588 | url : www.motioniitjee.com, : info@motioniitjee.com


Page # 86 WAVES

One wavelength

A

Displacement
O
Distance x

Crest Trough Crest Trough


The wavelength  of a wave is generally taken as the distance between two successive
crests or two successive trough. To be more specific, it is the distance between two
consecutive points on the wave which have same phase.
A displacement-time graph may also be drawn for a wave motion, showing how the displacement
of one particle at a particular distance from the source varies with time. If this is simple
harmonic variation then the graph is a sine curve.

• Wave Length, Frequency, Speed


If the source of a wave makes f vibrations per second, so they will the particles of the
transmitting medium. That is, the frequency of the waves equals frequency of the source.
When the source makes one complete vibration, one wave is generated and the disturbance
spreads out a distance  from the source. If the source continues to vibrate with constant
frequency f, then f waves will be produced per second and the wave advances a distance f 
in one second. If v is the wave speed then
v=f
This relationship holds for all wave motions.

Frequency depends on source (not on medium), v depends on medium (not on source


frequency), but wavelength depend on both medium and source.

(d) Initial Phase :


At x = 0 and t = 0, the sine function evaluates to zero and as such y-displacement is zero.
However, a wave form can be such that y-displacement is not zero at x =0 and t = 0. In such
case, we need to account for the displacement by introducting an angle like :
y(x,t) = Asin (kx – t + )
where "" is initial phase. At x = 0 and t = 0.
y(0, 0) = A sin ()
The measurement of angle determines following two aspects of wave form at x = 0, t = 0 :
(i) whether the displacement is positive or negative and (ii) whether wave form has positive
or negative slope.
For a harmonic wave represented by sine function, there are two values of initial
phase angle for which displacement at reference origin (x = 0, t = 0) is positive and has
equal magnitude. We know that the sine values of angles in first and second quadrants are
positive. A pair of initial phase angles, say  = /3 and 2/3, correspond to equal positive
sine values are :
sin = sin ( – )

    2  1
sin = sin  – 3  = sin 3  =
3 2

Corporate Head Office : Motion Education Pvt. Ltd., 394 - Rajeev Gandhi Nagar, Kota-5
WAVES Page # 87

2
To choose the initial phase in between the two values /3 & . We can look at a wave e
3
motion in yet another way. A wave form at an instant is displaced by a distance x in very
small time interval t then then speed to the particle at t = 0 & x = 0 is in upward +ve
direction in further time t
v

(0,0)

Ex.1 Find out the expression of wave equation which is moving is +ve x direction and at x = 0,
A
t=0y=
2
Sol. Let y = A sin (t – kx + )
at t = 0 and x = 0

A 1
 A sin   sin =
2 2

 3
= ,
4 4
To choose the correct phase angle  we displaced to wave. Slightly in +ve x direction such
that

A
In above figure Paticle at a is move downward towards point b i.e. particle at x = 0 & y =
2
have negative velocity which gives

y
 A cos(  – kx  ) at
t

t = 0, x = 0
is cos = – ve (from figure) ...(2)
from above discussion 3/4 gives sin + ve and cos negative i.e.

3

4

: 0744-2209671, 08003899588 | url : www.motioniitjee.com, : info@motioniitjee.com


Page # 88 WAVES

Note : Equation of wave which is moving –ve x direction.


v
 x
at time t y  A sin t  
 v

y  A sint
x
y = A sin ( t + kx + )
Ex.2 If ( t) & (kx) terms have same sign then the wave move toward –ve x direction and
vice versa and with diffierent initial phase.
y = A sin (t – kx) Wave move toward +ve x direction
y = A sin (–kx + t)

y = A sin (–kx – t) Wave move toward –ve x direction.


= A sin (kx + t + )
y = A sin (kx + t)

2. PARTICLE VELOCITY AND ACCELERATION :

Particle velocity at a given position x = x is obtained by differentiating wave function with


respect to time "t". We need to differentiate equation by treating "x" as constant. The partial
differentiation yields particle velocity as :

 
vp = y( x, t) = A sin(kx – t) = –A cos (kx – t)
t t
We can use the property of cosine function to find the maximum velocity. We obtain maximum
speed when cosine function evaluates to "–1" :
 vpmax = A
The acceleration of the particle is obtained by differentiating expression of velocity partially
with respect to time :

 
 ap = vp = {–A cos(kx – t)} = –2 A sin (kx – t) = –2y
t t
Again the maximum value of the acceleration can be obtained using property of sine function
:
 apmax = 2A

3. DIFFERENT FORMS OF WAVE FUNCTION :


Different forms give rise to bit of confusion about the form of wave function. The forms used
for describing wave are :
y (x, t) = A sin (kx – t)
y(x, t) = A sin (t – kx + )
Which of the two forms is correct ? In fact, both are correct so long we are in a position to
accurately interpret the equation. Starting with the first equation and using trigonometric
identity :

Corporate Head Office : Motion Education Pvt. Ltd., 394 - Rajeev Gandhi Nagar, Kota-5
WAVES Page # 89

We have,
 A sin (kx – t) = A sin ( – kx + t) = A sin (t – kx + )
 
Thus we see that two forms represent waves along at the same speed  v   . They differ,,
k
however, in phase. There is phase difference of "". This has implication on the waveform
and the manner particle oscillates at any given time instant and position. Let us consider two
waveforms at x = 0, t = 0. The slopes of the waveforms are :


y( x, t)  kA cos(kx – t) = kA = a positive number
x


and y( x, t)  –kA cos(t – kx) = –kA = a negative number
x

Forms of wave functions


y
v

y  A sin[kx – t]

x
O
vp

y  A sin[ t – kx]
vp

x
O

Exchange of terms in the argument of sine function results in a phase difference of .

In the first case, the slope is positive and hence particle velocity is negative. It means
particle is moving from reference origin or mean position to negative extreme position. In
the second case, the slope is negative and hence particle velocity is positive. It means
particle is moving from positive extreme position to reference origin or mean position. Thus
two forms represent waves which differ in direction in which particle is moving at a given
position.
Once we select the appropriate wave form, we can write wave equation in other forms as
given here :

 t  2
y(x, t) = A sin (kx – t) = A sin k  x –  = A sin ( x – vt)
 k  
Further, substituting for "k" and "" in wave equation, we have :

 2 2  x t
y (x, t) = A sin   x – T t  A sin 2   – T 

If we want to represent waveform moving in negative "x" direction, then we need to replace
"t" by "–t".

: 0744-2209671, 08003899588 | url : www.motioniitjee.com, : info@motioniitjee.com


Page # 90 WAVES

4. THE LINEAR WAVE EQUATION :


By using wave function y = A sin (t – kx + ), we can describe the motion of any point on
the string. Any point on the string moves only vertically, and so its x coordinate remains
constant. The transverse velocity vy of the point and its transverse acceleration ay are therefore.
 dy  y
vy    
 dt  x cons tan t t = A cos (t – kx + ) ...(1)

 dv y  v y 2y
ay    
 t =  t 2 = – A sin (t – kx + )..(2)
2
 dt  x cons tan t
and hence
vy. max
= A
ay.max = 2A
The transverse velocity and transverse acceleration of any point on the string do not reach
their maximum value simultaneously. Infact, the transverse velocity reaches its maximum
value (A) when the displacement y = 0, whereas the transverse acceleration reaches its
maximum magnitudes (2A) when y = ± A
further

 dy  y
 dx   = – kA cos (wt – kx + ) ...(3)
  t  cons tan t x

2y
= = – k2A sin (t – kx + ) ...(4)
x 2

y  y
From (1) and (3) – y
t k x
 vp = – vw × slope A
B
i.e. if the slope at any point is negative, particle
velocity and vice-versa, for a wave moving along positive x
x1 x2
x axis i.e. vw is positive.
For example, consider two points A and B on the y-curve
for a wave, as shown. The wave is moving along positive
x-axis.
Slope at A is positive therefore at the given moment, its velocity is negative. That means it
is coming downward. Reverse is the situation for particle at point B.
Now using equation (2) and (4)

2y k2 2y 2y 1 2 y


2
 2 2  2

x  t x v 2 t 2
This is known as the linear wave equation or diffential equation representation of the travelling
wave model. We have developed the linear wave equation from a sinusoidal mechanical
wave travelling through a medium. But it is much more general. The linear wave equation
successfully describes waves on strings, sound waves and also electromagnetic waves.

Thus, the above equation can be written as,

2y 2 y
2
 v2 ...(i)
t x 2

Corporate Head Office : Motion Education Pvt. Ltd., 394 - Rajeev Gandhi Nagar, Kota-5
WAVES Page # 91

The general solution of this equation is of the form


y(x, t) = f (ax ± bt) ...(ii)
Thus, any function of x and t which satisfies Eq. (i) or which can be written as Eq. (ii)
represents a wave. The only condition is that it should be finite everywhere and at all times.
Further, if these conditions are satisfied, then speed of wave (v) is given by,
coefficient of t b
v 
coefficient of x a
Thus plus (+) sign between ax and bt implies that the wave is travelling along negative x-
direction and minus (–) sign shows that it is travelling along positive x-direction.

Ex.3 Verify that wave function


2
y
(x – 3t)2  1
is a solution to the linear wave equation x and y are in cm.
Sol. By taking partial derivatives of this function w.r.t x and to t.
2y 12( x – 3 t) 2 – 4
2
 , and
x [( x – 3 t) 2  1]3

2y 108( x – 3 t)2 – 36


2

t [( x – 3 t) 2  1]3

2y 1 2 x
or 
 x2 9  t2
Comparing with linear wave equation, we see that the wave function is a solution to the
linear wave equation if the speed at which the pulse moves is 3 cm/s. It is apparent from
wave function therefore it is a solution to the linear wave equation.

Ex 4. A wave pulse is travelling on a string at 2 m/s. displacement y of the particle at x =


0 at any time t is given by
2
y 2
t 1
Find
(i) Expression of the function y = (x, t) i.e., displacement of a particle position x
and time t.
(ii) Shape of the pulse at t = 0 and t = 1s.
 x
Sol. (i) By replacing t by  t – v  , we can get the desired wave function i.e.,,

2
y 2
 x
t –  1
 2
(ii) We can use wave function at a particular instant, say t = 0, to find shape of the wave
pulse using different values of x.

: 0744-2209671, 08003899588 | url : www.motioniitjee.com, : info@motioniitjee.com


Page # 92 WAVES

2
y 2
Y
at t=0 x 2
1
4
at x=0 y=2
x=2 y=1 1
x=–2 y=1
x=4 y = 0.4
x = –4 y = 0.4 –2 0 –2 –4
–4
Using these value, shape is drawn.
Similarly for t = 1s, shape can drawn. What do you conclude about direction of motion of the
wave from the graphs? Also check how much the pulse has move in 1s time interval. This is
equal to wave speed. Here is the procedure.
2
y 2
 x at t = 1s
1 –   1
 2
at x=2 y = 2(maximum value)
at x=0 y=1
at x=4 y=1
Y
2

t=1
t=0
1

x
0
–2 2 4 6

The pulse has moved to the right by 2 units in 1 s interval.


x
Also as t – = constt.
2
Differentiating w.r.t time
1 dx dx
1– . 0  2
2 dt dt

Ex.5 A sinusoidal wave travelling in the positive x direction has an amplitude of 15 cm,
wavelength 40 cm and frequency 8 Hz. The vertical displacement of the medium at t
=0 and x = 0 is also 15 cm, as shown
y(cm)

40

15

x(cm)

(a) Find the angular wave number, period angular frquency and speed of the wave.
(b) Determine the phase constant , and write a general expression for the wave function.

Corporate Head Office : Motion Education Pvt. Ltd., 394 - Rajeev Gandhi Nagar, Kota-5
WAVES Page # 93

2  2rad 
Sol. (a) k    40cm  20 rad / cm

1 1
T  s  = 2 f = 16 s–1
f 8
v = f  = 320 cm/s
(b) It is given that A = 15 cm
and also y = 15 cm at x = 0 and t = 0
then using y = A sin (t – kx + )
15 = 15 sin   sin  = 1
Therefore, the wave function is
    rad 
y = A sin (t – kx + )  (15 cm)sin(16 s – ) t –  . x  
2   20 cm  2

5. SPEED OF A TRANSVERSE WAVE ON A STRING


Consider a pulse travelling along a string with a speed v to the right. If the amplitude of the
pulse is small compared to the length of the string, the tension T will be approximately
constant along the string. In the reference frame moving with speed v to the right, the pulse
in stationary and the string moves with a speed v to the left. Figure shows a small segment
of the string of length l. This segment forms part of a circular arc of radius R. Instantaneously
the segment is moving with speed v in a circular path, so it has centripetal acceleration v2/R.
The forces acting on the segment are the tension T at each end. The horizontal component of
these forces are equal and opposite and thus cancel. The vertical component of these forces
point radially inward towards the centre of the circular. arc. These radial forces provide
centripetal acceleration. Let the angle substended by the segment at centre be 2. The net
radial force acting on the segment is
v
l v2
ar  l
R v
 

R T T

R  
O

v O
(b)
(a)
Fig. (a) To obtain the speed v of a wave on a stretched string. It is convenient to describe the
motion of a small segment of the string in a moving frame of reference.
Fig. (b) In the moving frame of reference, the small segment of length l moves to the left
with speed v. The net force on the segment is in the radial direction because the
horizontal components of the tension force cancel.

F r  2T sin  2T
Where we have used the approximation sin    for small .
If  is the mass per unit length of the string, the mass of the segment of length l is
m =  l = 2R (as  l = 2R)
mv 2
From Newton's second law  Fr = ma =
R
 v2  T
or 2T = (2R)  R   v
  

: 0744-2209671, 08003899588 | url : www.motioniitjee.com, : info@motioniitjee.com


Page # 94 WAVES

Ex.6 Find speed of the wave generated in the string


as in the situation shown. Assume that the
tension in not
affected by the mass of the cord. 500 gm/m

Sol. T = 20 × 10 = 200 N

200
v  20m / s 20 kg
0.5

Ex.7 A taut string having tension 100 N and linear mass density
0.25 kg/m is used inside a cart to generate a wave pulse
starting at the left end, as shown. What should be the
velocity of the cart so that pulse remains stationary w.r.t
ground.
T
Sol. Velocity of pulse =  20m / s

  
Now vPG = vPC  v CG

0 = 20 i + v CG

VCG  –20î m/ s

Ex.8 One end of 12.0 m long rubber tube with a total mass of 0.9 kg is fastened to a fixed
support. A cord attached to the other and passes over a pulley and supports an
object with a mass of 5.0 kg. The tube is struck a transverse blow at one end. Find
the time required for the pulse to reach the other end (g = 9.8 m/s2)
Sol. Tension in the rubber tube AB, T = mg
T = (5.0) (9.8) = 49 N
or
Mass per unit length of rubber tube,

0.9
= = 0.075 kg/m
12
B
m
 Speed of wave on the tube,
A
T 49
v   25.56 m / s
 0.075
 The required time is,
AB 12
t   0.47 s
v 25.56

Ex.9 A uniform rope of mass 0.1 kg and length 2.45 m hangs from a ceiling
(a) Find the speed of transverse wave in the rope at a point 0.5 m distant from the
lower end.
(b) Calculate the time taken by a transverse wave to travel the full length of the
rope.
Sol. (a) As the string has mass and it is suspended vertically, tension in it will be different at
different points. For a point at a distance x from the free end, tension will be due to the
weight of the string below it. So, if m is the mass of string of length l, the mass of length x

Corporate Head Office : Motion Education Pvt. Ltd., 394 - Rajeev Gandhi Nagar, Kota-5
WAVES Page # 95

 m
of the string will be,   x.
l

 m m 
 T    xg  xg   
 l  l 

T
  xg

T 
or v  xg ...(i)
 x
At x = 0.5 m, v  0.5  9.8 = 2.21 m/s
(b) From Eq. (i) we see that velocity of the wave is different at different points. So, if at
point x the wave travels a distance dx in time dt, then

dx dx
dt  
v gx

t l
dx
  dt   gx
0 0

l 2.45
or t2 2 = 1.0 s Ans.
g 9.8

6. ENERGY CALCULATION IN WAVES :


,

(a) Kinetic energy per unit length


The velocity of string element in transverse direction is greatest at mean position and zero at
the extreme positions of waveform. We can find expression of transverse velocity by differ-
entiating displacement with respect to time. Now, the y-displacement is given by :
y = A sin (kx – t)
Differentiating partially with respect to time, the expression of particle velocity is :

y
vp  = – A cos (kx – t)
t
In order to calculate kinetic energy, we consider a small string element of length "dx" having
mass per unit length "". The kinetic energy of the element is given by :

1 1
dK  dmvp2  dx 2 A 2 cos 2 (kx – t)
2 2
This is the kinetic energy associated with the element in motion. Since it involves squared of
cosine function, its value is greatest for a phase of zero (mean position) and zero for a phase

of (maximum displacement).
2
Now, we get kinetic energy per unit length, "KL", by dividing this expression with the length
of small string considered :

dK 1
KL    2 A 2 cos 2 (kx – t)
dx 2

: 0744-2209671, 08003899588 | url : www.motioniitjee.com, : info@motioniitjee.com


Page # 96 WAVES

• Rate of transmission of kinetic energy


The rate, at which kinetic energy is transmitted, is obtained by dividing expression of kinetic
energy by small time element, "dt" :

dK 1 dx 2 2
   A cos 2 (kx – t)
dt 2 dt

dx
But, wave or phase speed, v, is time rate of position i.e. . Hence,
dt

dK 1
 v 2 A 2 cos 2 (kx – t)
dt 2
Here kinetic energy is a periodic function. We can obtain average rate of transmission of
kinetic energy by integrating the expression for integral wavelengths. Since only cos2(kx –
t) is the varying entity, we need to find average of this quantity only. Its integration over
1
intergal wavelengths give a value of " " . Hence, average rate of transmission of kinetic
2
energy is :

dK 1 1 1
|avg   v 2 A 2  v 2 A 2
dt 2 2 4

(b) Elastic potential energy


The elastic potential energy of the string element results as string element is stretched
during its oscillation. The extension or stretching is maximum at mean position. We can see
in the figure that the length of string element of equal x-length "dx" is greater at mean
position than at the extreme. As a matter of fact, the elongation depends on the slope of the
curve. Greater the slope, greater is the elongation. The string has the least length when
slope is zero. For illustration purpose, the curve is purposely drawn in such a manner that the
elongation of string element at mean position is highlighted.

y
V

t=t

x
O

t  t  t
vp
x
O

fig : The string element stretched most at equilibrium position

Greater extension of string element corresponds to greater elastic energy. As such, it is


greatest at mean position and zero at extreme position. This deduction in contrary to the
case of SHM in which potential energy is greatest at extreme position and zero at mean
position.

Corporate Head Office : Motion Education Pvt. Ltd., 394 - Rajeev Gandhi Nagar, Kota-5
WAVES Page # 97

• Potential energy per unit length


When the string segment is stretched from the length dx
to the length ds an amount of work = T (ds – dx) is ds
dy
done. This is equal to the potential energy stored in the
stretched string segment. So the potential energy in this
case is :
U = T (ds – dx)
Now ds  (dx2  dy 2 )
x x + dx
  dy  2 
 dx 1    
  dx  
from the binomial expansion
2
1  dy 
so ds  dx +   dx
2  dx 
2
1   y
U = T (ds – dx)  T  dx
2   x
or the potential energy density
2
dU 1   y 
 T  ...(i)
dx 2   x 

dy
= kAcos (kx – t)
dx
and T = v2 
Put above value in equation (i) then we get
dU 1
  2 A 2 cos 2 (kx – t)
dx 2

• Rate of transmission of elastic potential energy


The rate, at which elastic potential energy is transmitted, is obtained by dividing expression
of kinetic energy by small time element, "dt". This expression is same as that for kinetic
enegy.
dU 1
 v 2 A 2 cos 2 (kx – t)
dt 2
and average rate of transmission of elastic potential energy is :
dU 1 1 1
|avg   v 2 A 2  v 2 A 2
dt 2 2 4

(c) Mechanical energy per unit length


Since the expression elastic potential energy is same as that of kinetic energy, we get me-
chanical energy expression by multiplying expression of kinetic energy by "2". The mechani-
cal energy associated with small string element, "dx", is :
1 2
dE = 2xdK = 2x dmvp = dx2A2cos2 (kx – t)
2
Similarly, the mechanical energy per unit length is :

dE 1
EL   2x  2 A 2 cos 2 (kx – t ) = 2 A2 cos2 (kx – t)
dx 2

: 0744-2209671, 08003899588 | url : www.motioniitjee.com, : info@motioniitjee.com


Page # 98 WAVES

(d) Average power transmitted


The average power transmitted by wave is equal to time rate of transmission of mechanical
energy over integral wavelengths. It is equal to :
dE 1 1
Pavg  |avg  2  v 2 A 2  v 2 A 2
dt 4 2
If mass of the string is given in terms of mass per unit volume, "", then we make appropri-
ate change in the derivation. We exchange "" by "s" where "s" is the cross section of the
string :
1
Pavg  sv 2 A 2
2
(e) Energy density
Since there is no loss of energy involved, it is expected that energy per unit length is uniform
throughout the string. As much energy enters that much energy goes out for a given length
of string. This average value along unit length of the string length is equal to the average
rate at which energy is being transferred.
The average mechanical energy per unit length is equal to integration of expression
over integral wavelength
1 1 2 2
EL|avg = 2xv2 A2 = v A
4 2
We have derived this expression for harmonic wave along a string. The concept, however,
can be extended to two or three dimensional transverse waves. In the case of three dimen-
sional transverse waves, we consider small volumetric element. We, then, use density, , in
place of mass per unit length, . The corresponding average energy per unit volume is
referred as energy density (u) :
1
u vw 2 A 2
2
(f) Intensity
Intensity of wave (I) is defined as power transmitted per unit cross section area of the
medium :

A2 1
I   sv  2  vw 2 A 2
2s 2
Intensity of wave (I) is a very useful concept for three dimensional waves radiating in all
direction from the source. This quantity is usually referred in the context of light waves,
which is transverse harmonic wave in three dimensions. Intensity is defined as the power
transmitted per unit cross sectional area. Since light spreads uniformly all around, intensity
is equal to power transmitted, divided by spherical surface drawn at that point with source at
its center.

Phase difference between two particles in the same wave :


The general expression for a sinusoidal wave travelling in the positive x direction is
y(x, t) = A sin (t – kx)
Eq of Particle at x1 is given by y1 = A sin (t – kx1)
n

Eqn of particle which is at x2 from the origin


y2 = Asin (t – kx2)
Phase difference between particles is k(x2 – x1) = 


Kx =  x 
k

Corporate Head Office : Motion Education Pvt. Ltd., 394 - Rajeev Gandhi Nagar, Kota-5
WAVES Page # 99

7. PRINCIPLE OF SUPERPOSITION :

This principle defines the displacement of a medium particle when it is oscillating under the
influence of two or more than two waves. The principle of superposition is stated as :
"When two or more waves superpose on a medium particle than the resultant
displacement of that medium particle is given by the vector sum of the individual displacements
produced by the component waves at that medium particle independently."
  
Let y 1, y 2 ,....... y N are the displacements produced by N independent waves at a
medium particle in absence of others then the displacemnt of that medium, when all the
waves are superposed at that point, is given as
    
y  y 1  y 2  y 3  .......  y N
If all the waves are producing oscillations at that point are collinear then the displacement of
the medium particle where superposition is taking place can be simply given by the algebric
sum of the individual displacement. Thus we have
y = y1 + y2 + ..............+yN
The above equation is valid only if all individual displacements y1, y2 ........... yN are along
same straight line.
A simple example of superposition can be understood by figure shown. Suppose two wave
pulses are travelling simultaneously in opposite directions as shown. When they overlap
each other the displacement of particle on string is the algebric sum of the two displacement
as the displacements of the two pulses are in same direction. Figure shown (b) also shows
the similar situation when the wave pulses are in opposite side.
y
y v
v
y2 v y2
y1 y2
y1 y1 x
x
v
y
y v
v
y2
v y2
y1 y2
y1 y1 x
x
v

y1 + y2
v v

v
(a) Applications of Principle of Superposition of Waves
There are several different phenomenon which takes place during superposition of two or
more wave depending on the wave characteristics which are being superposed. We'll discuss
some standard phenomenons, and these are :
(1) Interference of Wave
(2) Stationary Waves
(3) Beats
(4) Lissajou's Figures (Not discussed here in detail.)
Lets discuss these in detail.

: 0744-2209671, 08003899588 | url : www.motioniitjee.com, : info@motioniitjee.com


Page # 100 WAVES

(b) Interference of Waves


Suppose two sinusoidal waves of same wavelength and amplitude travel in same direction
along the same straight line (may be on a stretched string) then superposition principle can
be used to define the resultant displacement of every medium particle. The resultant wave in
the medium depends on the extent to which the waves are in phase with respect to each
other, that is, how much one wave form is shifted from the other waveform. If the two waves
are exactly in same phase, that is the shape of one wave exactly fits on to the other wave
then they combine to double the displacement of every medium particle as shown in figure
(a). This phenomenon we call as constructive interference. If the superposing waves are
exactly out of phase or in opposite phase then they combine to cancel all the displacements
at every medium particle and medium remains in the form of a straight line as shown in
figure (b)

y y

A A

Wave I
x x

–A –A

y y

A +A

Wave II
x x

–A

y
y
2A

Resultant
Wave x
x

–2A (a)
(b)

This phenomenon we call destructive interference. Thus we can state that when waves meet,
they interfere constructively if they meet in same phase and destructively if they meet in
opposite phase. In either case the wave patterns do not shift relative to each other as they
propagates. Such superposing waves which have same form and wavelength and have a
fixed phase relation to each other, are called coherent waves. Sources of coherent waves are
called coherent source. Two indepedent sources can never be coherent in nature due to
practical limitations of manufacturing process. Generally all coherent sources are made either
by spliting of the wave forms of a single source or the different sources are fed by a single
main energy source.

Corporate Head Office : Motion Education Pvt. Ltd., 394 - Rajeev Gandhi Nagar, Kota-5
WAVES Page # 101

In simple words interference is the phenomenon of superposition of two coherent


waves travelling in same direction.
We've discussed that the resultant displacement of a medium particle when two
coherent waves interfere at that point, as sum or difference of the individual displacements
by the two waves if they are in same phase (phase difference = 0, 2, .....) or opposite phase
(phase difference = , 3,.....) respectively. But the two waves can also meet at a medium
particle with phase difference other then 0 or 2, say if phase difference  is such that 0 < 
< 2, then how is the displacement of the point of superposition given ? Now we discuss the
interference of waves in details analytically.

(c) Analytical Treatment of Interference of Waves


S1
x1
A1 sin(t  kx) y1  A1 sin(t  kx1 )

y2  A 2 sin(t  kx2 )
x2
S2
A 2 sin(t  kx)
Interference implies super position of waves. Whenever two or more than two waves
superimpose each other they give sum of their individual diplacement.
Let the two waves coming from sources S1 & S2 be
y1 = A1 sin (t + kx1 )
y2 = A2 sin (t + kx2) respectively.
Due to superposition
ynet = y1 + y2
ynet = A1 sin (t + kx1) + A2 sin (t + kx2)
Phase difference between y1 & y2 = k(x2 – x1)
i.e.,  = k(x2 – x1)

2
As  = x (where x = path difference &  = phase difference)

Anet = A 12  A 22  2A 1A 2 cos 

2
 A net  A 12  A 22  2A 1A 2 cos 

 Inet = I1 + I2 + 2 I1I2 cos  (as I  A2)


When the two displacements are in phase, then the resultant amplitude will be sum of the
two amplitude & Inet will be maximum, this is known of constructive interference.
For Inet to be maximum
cos = 1   = 2n where n = {0,1,2,3,4,5...........}
2
x  2n  x = n

For constructive interference
Inet = ( I1  I2 )2
When I1 = I2 = I
Inet = 4 I
Anet = A1 + A2

: 0744-2209671, 08003899588 | url : www.motioniitjee.com, : info@motioniitjee.com


Page # 102 WAVES

When superposing waves are in opposite phase, the resultant amplitude is the difference of
two amplitudes & Inet is minimum; this is known as destructive interference.
For Inet to be minimum,
cos  = – 1
 = (2n + 1)  where n = {0,1,2,3,4,5...........}

2 λ
x = (2n + 1)  x = (2n  1)
 2
For destructive interfence
Inet = ( I1 – I2 )2
If I1 = I2
Inet = 0
Anet = A1 – A2
( I1  I2 ) 2
Ratio of Imax & Imin =
( I1 – I2 )2
Generally,
Inet = I1 + I2 + 2 I1I2 cos 
If I1 = I2 = I
Inet = 2I + 2Icos

Inet = 2I(1 + cos ) = 4Icos2
2

Ex.10 Wave from two source, each of same frequency and travelling in same direction,
but with intensity in the ratio 4 : 1 interfere. Find ratio of maximum to minimum
intensity.
2
 I1 
2
  1
Imax  I1  I2   I2  2
   2  1
Sol.  = =   =9:1
Imin  I1 – I2 
   I1 – 1  2 – 1
 I 
 2 

Ex.11 A triangular pulse moving at 2 cm/s on a rope approaches an end at which it is free
to slide on a vertical pole.

2 cm/s

1 cm
2 cm 1cm 1cm

1
(a) Draw the pulse at s interval until it is completely reflected.
2
(b) What is the particle speed on the trailing edge at the instant depicted ?

Corporate Head Office : Motion Education Pvt. Ltd., 394 - Rajeev Gandhi Nagar, Kota-5
WAVES Page # 103

Sol. (a) Reflection of a pulse from a free boundary is really the superposition of two identical
waves travelling in opposite direction. This can be shown as under.

1cm 1cm 1cm


+ =
2cm 1cm 1cm 2cm 2cm 1cm

At t = ½ S
(a) (b) (c)

2cm
1cm 1cm 1cm
+ =
2cm 1cm 1cm 2cm 2cm 1cm

At t = 1 s
(d) (e) (f)

0.5cm 1cm

0.5cm 1cm 1cm


+ =
1cm 1cm 1cm 1cm 1cm 1cm 2cm

At t = ½ s
(g) (h) (i)

1cm 1cm
1cm
2cm 1cm
+ =
1cm 2cm 1cm 2cm

At t = 2s
(j) (k) (i)

1
In every s , each pulse (one real moving towards right and one imaginary moving towards
2
left travels a distance of 1 cm, as the wave speed is 2 cm/s.)
(b) Particle speed, vp = |– v (slope)|

1
Here, v = wave speed = 2 cm/s and slope =
2
 Particle speed = 1 cm/s Ans.

: 0744-2209671, 08003899588 | url : www.motioniitjee.com, : info@motioniitjee.com


Page # 104 WAVES

Ex.12 Figure shows a rectanglar pulse and triangular pulse approaching each other. The pulse
speed is 0.5 cm/s. Sketch the resultant pulse at t = 2 s

2cm

–2 –1 0 1 2 3
x(cm)

Sol. In 2 s each pulse will travel a distance of 1 cm.


The two pulses overlap between 0 and 1 cm as shown in figure. So, A1 and A2 can be added
as shown in figure (c).

(a) A1 2cm

–1 0 1
+ (c) 2cm
A1
(b)
2cm
A2 A2 2cm

0 1 2 –1 0 1 2
Resultant pluse
at t = 2s

8. REFLECTION AND TRANSMISSION IN WAVES :

1. When a pulse travelling along a string reaches the end, it is reflected. If the end is fixed as
shown in figure (a), the pulse returns inverted. This is bacause as the leading edge reaches
the wall, the string pulls up the wall. According to Newton's third law, the wall will exert an
equal and opposite force on the string as all instants. This force is therefore, directed first
down and then up. It produces a pulse that is inverted but otherwise identical to the original.

The motion of free end can be studied by letting a ring at the end of string sliding smoothly
on the rod. The ring and rod maintain the tension but exert no transverse force.

Corporate Head Office : Motion Education Pvt. Ltd., 394 - Rajeev Gandhi Nagar, Kota-5
WAVES Page # 105

(a) (b)

Reflection of wave pulse (a) at a fixed end


of a string and (b) at a free end. Time
increases from top to bottom in each
figure.

When a wave arrives at this free end, the ring slides the rod. The ring reaches a maximum
displacement. At this position the ring and string come momentarily to rest as in the fourth
drawing from the top in figure (b). But the string is stretched in this position, giving in-
creased tension, so the free end of the string is pulled back down, and again a reflected pulse
is produced, but now the direction of the displacement is the same as for the initial pulse.

2. The formation of the reflected pulse is similar to the overlap of two pulses travelling in
opposite directions. The net displacement at any point is given by the principle of superpo-
sition.

: 0744-2209671, 08003899588 | url : www.motioniitjee.com, : info@motioniitjee.com


Page # 106 WAVES

(a) (b)

Fig (a) : shows two pulses with the same shape, one inverted with respect to the other,
travelling in opposite directions. Because these two pulses have the same shape the net
displacement of the point where the string is attached to the wall is zero at all times.
Fig (b) : shows two pulses with the same shape, travelling in oppoiste directions but not
inverted relative to each other. Note that at one instant, the displacement of the free end is
double the pulse height.

9. REFLECTION AND TRANSMISSION BETWEEN TWO STRING :

Here we are dealing with the case where the end point is neither completely fixed nor
completely free to move As we consider an example where a light string is attached to a
heavy string as shown is figure a.
If a wave pulse is produced on a light string moving towards the friction a part of the wave is
reflected and a part is transmitted on the heavier string the reflected wave is inverted with
respect to the original one.

T
v1  v2
1 y  At sin(t – k 2 x)
T
v2 
2 At
(v2 ,  2 ) v1> v2
( v1, 1 ) Ar

v1 y  Ar sin(t  k 1x  )
y  Ai sin(t – k 1x)

figure (a)

On the other hand if the wave is produced on the heavier string which moves toward the
junction a part will the reflected and a part transmitted, no inversion in waves shape will take
place.

The wave velocity is smaller for the heavier string lighter string
v1
y  Ai sin( t – k 1x)
v2
P
1 2

Corporate Head Office : Motion Education Pvt. Ltd., 394 - Rajeev Gandhi Nagar, Kota-5
WAVES Page # 107

v2
v1
Ar At
P

y  Ar sin(t  k1x) y  At sin(t – k 2 x)


figure : (b)
Now to find the relation between Ai, Ar, At we consider the figure (b)
Incident Power = Reflected Power + Transmitted Power
Pi = Pr + Pt

22 f 2 A i 2 1v 1  2 2 f 2 A r 21v1  2 2 f 2 A t 2 2 v 2 ...(i)


T T
Put 1 = 2 and 2 = 2
v1 v2
in equation (i) their
2 2 2
Ai A A
 r  t
v1 v1 v2

2 2 v1 2
Ai  Ar  At .......(ii)
v2
Maximum displacement of joint particle P (as shown in figure) due to left string
= Ai + Ar
Maximum displacement of joint particle due to right string = At
At the boundary (at point P) the wave must be continuous, that is there are no kinks in it.
Then we must have Ai + Ar = At ...(iii)
from equation (ii) & (iii)

v1
Ai – Ar = v A t ...(iv)
2

from eq. (iii) & (iv)

 2v 2 
At =  v  v  A i
 1 2

 v 2 – v1 
Ar =  v  v  A i
 1 2

10. STANDING WAVES :


In previous section we've discussed that when two coherent waves superpose on a medium
particle, phenomenon of interference takes place. Similarly when two coherent waves travelling
in opposite direction superpose then simultaneous interference if all the medium particles
takes place. These waves interfere to produce a pattern of all the medium particles what we
call, a stationary wave. If the two interfering waves which travel in opposite direction carry
equal energies then no net flow of energy takes place in the region of superposition. Within
this region redistribution of energy takes place between medium particles. There are some
medium particles where constructive interference takes place and hence energy increases
and on the other hand there are some medium particles where destructive interference takes
place and energy decreases. Now we'll discuss the stationary waves analytically.

: 0744-2209671, 08003899588 | url : www.motioniitjee.com, : info@motioniitjee.com


Page # 108 WAVES

Let two waves of equal amplitude are travelling in opposite direction along x-axis. The
wave equation of the two waves can be given as
y1 = A sin (t – kx) [Wave travelling in +x direction] ...(1)
and y2 = A sin (t + kx) [Wave travelling in –x direction] ...(2)
When the two waves superpose on medium particles, the resultant displacement of the
medium particles can be given as
y = y1 + y2
or y = A sin (t – kx) + A sin (t + kx)
or y = A [sint cos kx – cos t sin kx + sin t cos kx + cos t sin kx]
or y = 2A cos kx sin t ...(3)
Equation (3) can be rewritten as
y = R sin t ...(4)
Where R = 2 A cos kx ...(5)
Here equation (4) is an equation of SHM. It implies that after superposition of the two waves
the medium particles executes SHM with same frequency  and amplitude R which is given
by equation (5) Here we can see that the oscillation amplitude of medium particles depends
on x i.e. the position of medium particles. Thus on superposition of two coherent waves
travelling in opposite direction the resulting interference pattern, we call stationary waves,
the oscillation amplitude of the medium particle at different positions is different.
At some point of medium the resultant amplitude is maximum which are given as
R is maximum when cos kx = ± 1

2
or x  N [N  I]

N
or x=
2

 3
or x = 0, , , .....
2 2
and the maximum value of R is given as
Rmax= ± 2 A ...(6)

 3
Thus in the medium at position x = 0, , , , ........... the waves interfere constructively
2 2
and the amplitude of oscillations becomes 2A. Similarly at some points of the medium, the
waves interfere destructively, the oscillation amplitude become minimum i.e. zero in this
case. These are the points where R is minimum, when
cos kx = 0
2x 
or  ( 2N  1)
 2

or x = (2N + 1) [N  I]
4
 3 5
or x , , ...........
4 4 4
and the minimum value of R is given as
Rmin = 0 [7]
 3 5
Thus in the medium at position x = , , ......... the waves interfere destructively and
4 4 4
the amplitude of oscillation becomes zero. These points always remain at rest. Figure (a)
shows the oscillation amplitude of different medium particles in a stationary waves.

Corporate Head Office : Motion Education Pvt. Ltd., 394 - Rajeev Gandhi Nagar, Kota-5
WAVES Page # 109

figure (a)
In figure (a) we can see that the medium particles at which constructive interference takes
place are called antinodes of stationary wave and the points of destructive interference are
called nodes of stationary waves which always remain at rest.
Figure (b) explain the movement of medium particles with time in the region where stationary
waves are formed. Let us assume that at an instant t = 0 all the medium particles are at their
extreme positions as shown in figure - (b - 1). Here points ABCD are the nodes of stationary
waves where medium particles remains at rest. All other starts moving towards their mean
positions and t = T / 4 all particles cross their mean position as shown in figure (b – 3), you
can see in the figure that the particles at nodes are not moving. Now the medium crosses
their mean position and starts moving on other side of mean position toward the other
extreme position. At time t = T/2, all the particles reach their other extreme position as
shown in figure (b - 5) and at time t = 3T/4 again all these particles cross their mean position
in opposite direction as shown in figure (b - 7).

(1) (2) (3)

(4) (5) (6)

(7) (8) (9)

figure (b)
Based on the above analysis of one complete oscillations of the medium particles, we can
make some interference for a stationary waves. These are :
(i) In oscillations of stationary wave in a region, some points are always at rest (nodes) and
some oscillates with maximum amplitudes (antinodes). All other medium particles oscillate
with amplitudes less then those of antinodes.
(ii) All medium particles between two successive nodes oscillate in same phase and all
medium particles on one side of a node oscillate in opposite phase with those on the other
side of the same node.
(iii) In the region of a stationary wave during one complete oscillation all the medium particles
come in the form of a straight line twice.
(iv) If the component wave amplitudes are equal, then in the region where stationary wave
is formed, no net flow of energy takes place, only redistribution of energy takes place in the
medium.

: 0744-2209671, 08003899588 | url : www.motioniitjee.com, : info@motioniitjee.com


Page # 110 WAVES

(a) Different Equation for a Stationary Wave


Consider two equal amplitude waves travelling in opposite direction as
y1 = A sin (t – kx) ...(11)
and y2 = A sin (t + kx) ...(12)
The result of superposition of these two waves is
y = 2A cos kx sin t ...(13)
Which is the equation of stationary wave where 2A cos kx represents the amplitude of
medium particle situated at position x and sin t is the time sinusoidal factor. This equation
(13) can be written in several ways depending on initial phase differences in the component
waves given by equation (11)) can (12). If the superposing waves are having an initial phase
difference , then the component waves can be expressed as
y1 = A sin (t – kx) ...(14)
y2 = – A sin (t – kx) ...(15)
Superposition of the above two waves will result
y = 2A sin kx cos t ...(16)
Equation (16) is also an equation of stationary wave but here amplitude of different medium
particles in the region of interference is given by
R = 2A sin kx ...(17)
Similarly the possible equations of a stationary wave can be written as
y = A0 sin kx cos ( t + ) ....(18)
y = A0 cos kx sin (t + ) ...(19)
y = A0 sin kx sin (t + ) ...(20)
y = A0 cos kx cos (t + ) ...(21)
Here A0 is the amplitude of antinodes. In a pure stationary wave it is given as
A0 = 2A
Where A is the amplitude of component waves. If we care fully look at equation (18) to (21),
we can see that in equation (18) and (20), the particle amplitude is given by
R = A0 sin kx ...(22)
Here at x = 0, there is nodes as R = 0 and in equation (19) and (21) the particle amplitude
is given as
R = A0 cos kx ...(23)
Here at x = 0, there is an antinode as R = A0. Thus we can state that in a given system of co-
ordinates when origin of system is at a node we use either equation (18) or (20) for analytical
representation of a stationary wave and we use equation (19) or (21) for the same when an
antinode is located at the origin of system.
Ex.13 Find out the equation of the standing waves for the following standing wave pat-
tern.

x=L
x=0

2 x x x
(A) A sin x cos t (B) A sin cos  t (C) A cos cos t (D) A cos cos t
L L 2L L

Corporate Head Office : Motion Education Pvt. Ltd., 394 - Rajeev Gandhi Nagar, Kota-5
WAVES Page # 111

Sol. General Equation of standing wave


y = A cos t
where
A = A sin (kx + )
here  = L
2
 k=
L
 2 
A = A sin (kx + ) = A sin  x  
 L 
at x = 0 node
 A = 0 at x = 0
 =0
2
eq. of standing wave = A sin x cos t y
L
2A
Ex.14 Figure shows the standing waves pattern
in a string at t = 0. Find out the equation of A
the standing wave where the amplitude of
antinode is 2A. x
Sol. Let we assume the equation of standing waves
is = A sin (t + ) –2A
where A = 2A sin (kx + )
 x = 0 is node  A = 0, at x = 0
2A sin  = 0   = 0
at t = 0 Particle at is at y = A and going towards mean position.
  5
 = + =
2 3 6
so eq. of standing waves is
 5 
y = 2Asin kx sin  t  
 6 

Ex.15 A string 120 cm in length sustains standing wave with the points of the string at
which the displacement amplitude is equal to 3.5 mm being separated by 15.0 cm.
The maximum displacement amplitude is X. 95 mm then find out the value of X.
Sol. In this problem two cases are possible :

A x=0 B
B x=0
15cm
15cm
Case - I is that A and B have the same displacement amplitude and case - 2 is that C and
D have the same amplitude viz 3.5 mm. In case 1, if x = 0 is taken at antinode then
A = a cos kx
In case -2, if x = 0 is taken at node, then
A = a sin kx
But since nothing is given in the question.
Hence from both the cases, result should be same. This is possible only when
a cos kx = a sin kx


or kx = or a  A  3.5  4.95mm
4 cos kx cos  / 4

: 0744-2209671, 08003899588 | url : www.motioniitjee.com, : info@motioniitjee.com


Page # 112 WAVES

(b) Energy of standing wave in one loop


When all the particles of one loop are at extreme position then total energy in the loop is in
the form of potential energy only when the particles reaches its mean position then total
potential energy converts into kinetic energy of the particles so we can say total energy of
the loop remains constant
Total kinetic energy at mean position is equal to total energy of the loop because potential
energy at mean position is zero.
Small kinetic energy of the particle
which is in element dx is x dx

 /2
1
d (KE) = dmv 2
2
dm =  dx
Velocity of particle at mean position
= 2A sin kx 

1
then d (KE) = dx . 4A2 2 sin2kx  d (KE) = 2A22 . sin2kx dx
2
/2

 d(K.E )  2A 2  2  sin 2 kxdx



0

 /2 /2
2 2  sin 2kx  1 2 2
Total K.E = A 2  2  (1 – cos 2kx)dx  A    x – = A  
  2k  0 2
0
11. STATIONARY WAVES IN STRINGS :

(a) When both end of string is fixed :


A string of length L is stretched between two points. When the string is set into vibrations, a
transverse progressive wave begins to travel along the string. It is reflected at the other
fixed end. The incident and the reflected waves interfere to produce a stationary transverse
wave in which the ends are always nodes, if both ends of string are fixed.
Fundamental Mode
(a) In the simplest form, the string vibrates in one loop in which the ends are the nodes and
the centre is the antinode. This mode of vibration is known as the fundamental mode and
frequency of vibration is known as the fundamental frequency or first harmonic.


Since the distance between consecutive nodes is
2
1
 L  1 = 2L
2
If f1 is the fundamental frequency of vibration, then the velocity of transverse waves is given
as,

v
v   1f1 or f1  ...(i)
2L

Corporate Head Office : Motion Education Pvt. Ltd., 394 - Rajeev Gandhi Nagar, Kota-5
WAVES Page # 113

First Overtone
(b) The same string under the same conditions
may also vibrate in two loops, such that the cen-
tre is also the node

2 2
 L  2 = L
2  
If f2 is frequency of vibrations 2 2

v v
 f2  
2 L

v
 f2  ...(ii)
L
The frequency f2 is known as second harmonic or first overtone.
Second Overtone
(c) The same string under the same conditions
may also vibrate in three segments.
3 3
 L
2
2
 3  L
3
If f3 is the frequency in this mode of vibration, then,
3v
f3  ...(iii)
2L
The frequency f3 is known as third harmonic or second overtone.
Thus a stretched string vibrates with frequencies, which are integral multiples of the funda-
mental frequencies. These frequencies are known as harmonics.

T
The velocity of transverse wave in stretched string is given as v  . Where T = tension in

the string.
 = linear density or mass per unit length of string. If the string fixed at two ends, vibrates
in its fundamental mode, then

1 T
f ....(17)
2L 

n T
In general f = nth harmonic
2 
(n – 1)th overtone
In general, any integral multiple of the fundamental frequency is an allowed frequency.
These higher frequenceis are called overtones. Thus, v1 = 2v0 is the first overtone, v2 = 3v0
is the second overtone etc. An integral multiple of a frequency is called its harmonic. Thus,
for a string fixed at both the ends, all the overtones are harmonics of the fundamental
frequency and all the harmonics of the fundamental frequency are overtones.

: 0744-2209671, 08003899588 | url : www.motioniitjee.com, : info@motioniitjee.com


Page # 114 WAVES

(b) When one end of the string is fixed and other is free :
free end acts as antinode

/4 1 T
1. f fundamental or Ist harmonic
4 

3 T
2.   3 / 4 f IIIrd harmonic or Ist overtone
4 

(2n  1) T
In general : f  ((2n + 1)th harmonic, nth overtone)
4 

S.No. Tra ve lling w a ve s Sta tiona ry w a ve s


1 These waves advance in a medium with a These waves remain stationary between two
definite velocity boundaries in the m edium .

2 In these waves, all particles of the In these waves, all particles except nodes
medium oscillate with same frequency oscillate with same frequency but different
and amplitude. amplitudes. A mplitude is zero at nodes and
maximum at antinodes.
3 At any instant phase of vibration varies At any instant the phase of all particles
continuosly from one particle to the other between two successive nodes is the
i.e., phase difference between two sam e, but phase of particles on one side of
particles can have any value between a node is opposite to the phase of particles
0 and 2  on the other side of the node, i.e, phase
difference between any two particles can be
either 0 or 

4 In these wave, at no instant all the In these waves all particles of the medium
particles of the medium pass through pass through their mean position
their m ean positions simultaneously. simultaneously twice in each tim e period.
5 These waves transmit energy in the These waves do not transm it energy in the
medium. medium.

Corporate Head Office : Motion Education Pvt. Ltd., 394 - Rajeev Gandhi Nagar, Kota-5
WAVES Page # 115

Exercise - I OBJECTIVE PROBLEMS (JEE MAIN)


1. A transverse wave is described by the equation Sol.
Y = Y0 sin 2 (ft – x/). The maximum particle
velocity is equal to four times the wave velocity if
(A)  =  Y0/4 (B)  =  Y0/2
(C)  =  Y0 (D)  = 2 Y0
Sol.

4. A wave pulse is generated in a string that lies


along x-axis. At the points A and B, as shown in
figure, if RA and RB are ratio of wave speed to the
particle speed respectively then :
y
B V

2. Both the strings, show in figure, are made of A


same material and have x
same cross section. The A (A) RA > RB (B) RB > RA (C) RA = RB
pulleys are light. The wave (D) Information is not sufficient to decide.
speed of a transverse wave Sol.
in the string AB is v1 and in
CD it is v2. The v1/v2 is B C
(A) 1 (B) 2
(C) 2 (D) 1/ 2 D
Sol.

5. A wave is propagating along x-axis. The


displacement of particles of the medium in Z-
di re ct i on at t =0 is gi ve n by :
z  exp[ ( x  2)2 ], Where ‘x’ is in meters. At t=1s,
3. A transverse wave of amplitude 0.50 m,
wavel ength 1 m and frequency 2 hertz is the same wave di sturbance i s given by:
propagating in a string in the negative x-direction. z  exp[ (2  x ) 2 ], Then the wave propagation
The expression form of the wave is velocity is
(A) y(x,t) = 0.5 sin (2x – 4t) (A) 4 m/s in + x direction
(B) y(x,t) = 0.5 cos (2x + 4t) (B) 4 m/s in –x direction
(C) y(x,t) = 0.5 sin (x – 2t) (C) 2 m/s in + x direction
(D) y(x,t) = 0.5 cos (2x – 2t) (D) 2 m/s in –x direction

: 0744-2209671, 08003899588 | url : www.motioniitjee.com, : info@motioniitjee.com


Page # 116 WAVES

Sol. Sol.

8. If the speed of the wave shown in the


figure is 330 m/s in the given medium, then
the equation of the wave propagating in the
positive x-direction will be - (all quantities are
in MKS units)
6. The equation of a wave travelling along the positive
x-axis, as shown in figure at t=0 is given by
y
1

0
x
–0.5
–0.1
(A) y = 0.05 sin 2  (4000 t – 12.5 x)
    (B) y = 0.05 sin 2  (4000 t – 122.5 x)
(A) sin kx – t  6  (B) sin kx – t – 6 
(C) y = 0.05 sin 2  (3300 t – 10 x)
(D) y = 0.05 sin 2  (3300 x – 10 t)
    Sol.
(C) sin t – kx   (D) sin t – kx – 
 6  6
Sol.

9. The displacement produced by a simple


harmonic wave is :
10 FG x IJ
7. The velocity of a wave propagating along
y =
 H
sin 2000  t 
17 K
cm. The time
a stretched string is 10m/s and its frequency period and maximum velocity of the particle
is 100 Hz. The phase difference between the will be respectively -
particles situated at a distance of 2.5 cm on (A) 10–3 second and 200 m/s
the string will be - (B) 10–2 second and 2000 m/s
(A) /8 (B) /4 (C) 10–3 second and 330 m/s
(C) 3/8 (D) /2 (D) 10–4 second and 20 m/s

Corporate Head Office : Motion Education Pvt. Ltd., 394 - Rajeev Gandhi Nagar, Kota-5
WAVES Page # 117

Sol. Sol.

12. A uniform rope of length 10 m and mass


15 kg hangs vertically from a rigid support. A
block of mass 5 kg is attached
to the free end of the rope.
A transv erse p ul se of
10. Figure shown the shape of part of a long w av el engt h 0. 08 m i s
string in which transverse waves are produced produced at the lower end
by attaching one end of the string to tuning fork of the rope. The wavelength
of frequency 250Hz. What is the velocity of the of the pulse when it reaches
waves? the top of the rope will be-
5cm (A) 0.08 m (B) 0.04 m
(C) 0.16 m (D) 0 m
0.3cm Sol.
O
0.1cm 0.5cm
–5cm
(A) 1.0 ms–1 (B) 1.5 ms–1
(C) 2.0 ms–1 (D) 2.5 ms–1
Sol.

13. A uniform rope having some mass hanges


vertically from a rigid support. A transverse wave
pulse is produced at the lower end. The speed
(v) of the wave pulse varies with height (h) from
the lower end as:
v v

11. A block of mass 1 kg is hanging vertically


from a string of length 1 m and Mass/length =0.001 (A) (B)
kg/m. A small pulse is generated at its lower end.
h h
The Pulse reaches the top end in approximately.
v
v
1m

(C) (D)
(A) 0.2 sec (B) 0.1 sec h
(C) 0.02 sec (D) 0.01 sec h

: 0744-2209671, 08003899588 | url : www.motioniitjee.com, : info@motioniitjee.com


Page # 118 WAVES

Sol. Sol.

14. A wire of 102 kg m1 passes over a frictionless


light pulley fixed on the top of a frictionless
inclined plane, which makes an angle of 300 with
the horizontal. Masses m and M are tied at two
ends of wire such that m rests on the plane and
M hangs freely vertically downwards. The entire 16. A Wave pulse on a string has the dimension
system is in equilibrium and a transverse wave shown in figure. The waves speed is v=1 cm/s.
propagates along the wire with a velocity of If point O is a free end. The shape of wave at time
t=3 s is:
100 ms 1 .
v=1cm/s
m 1 1 cm
(A) M=5 kg (B) 
M 4
O
m
(C) m=20 kg (D) 4
M 1cm 1cm 2cm
Sol. O
1cm
(A) O (B)
1cm

1cm
2cm
(C) 1cm (D)

O 1cm
15. A pulse shown here is reflected from the rigid Sol.
wall A and then from free end B. The shape of
the string after these 2 Reflection will be.

B A

(A) A
(B) A
B B

(C) A
(D) A
B B

Corporate Head Office : Motion Education Pvt. Ltd., 394 - Rajeev Gandhi Nagar, Kota-5
WAVES Page # 119

17. A string 1m long is drawn by a 300Hz vibrator 20. When two waves of the same amplitude and
attached to its end. The string vibrates in 3 frequency but having a phase difference of ,
segments. The speed of transverse waves in the travelling with the same speed in the same
string is equal to direction (positive x), interfere, then
(A) 100 m/s (B) 200 m/s (A) their resultant amplitude will be twice that of
(C) 300 m/s (D) 400 m/s a single wave but the frequency will be same
Sol. (B) their resultant amplitude and frequency will
both be twice that of a single wave
(C) their resultant amplitude will depend on the
phase angle while the frequency will be the same
(D) the frequency and amplitude of the resultant
wave will depend upon the phase angle.
Sol.

18. For a wave displacement amplitude is 10–8 m,


density of air 1.3 kg m–3, velocity in air 340 ms–1
and frequency is 2000 Hz. The intensity of wave
is -
(A) 5.3 × 10–4 Wm–2 (B) 5.3 × 10–6 Wm–2
(C) 3.5 × 10 Wm
–8 –2
(D) 3.5 × 10–6 Wm–2
Sol.

21. A wave pulse, travelling on a two piece string,


gets partially reflected and partially transmitted
at the junction. The reflected wave is inverted in
shape as compared to the incident one. If the
i nci dent wave has wavel ength  and the
transmitted wave .
(A)  >  (B)  =  (C)  < 
(D) nothing can be said about the relation of 
and .
Sol.
19. Two waves of equal amplitude A, and equal
frequency travels in the same direction in a
medium. The amplitude of the resultant wave is
(A) 0 (B) A (C) 2A
(D) between 0 and 2A
Sol.

: 0744-2209671, 08003899588 | url : www.motioniitjee.com, : info@motioniitjee.com


Page # 120 WAVES

22. The rate of transfer of energy in a wave 24. A wave is represented by the equation
depends y= 1 0sin 2 (100t  0.02 X )  10sin 2 (100t  0.02 X ). The
(A) directly on the square of the wave amplitude
maxi mum ampli tude and l oop length are
and square of the wave frequency
respectively
(B) directly on the square of the wave amplitude
(A) 20 units and 30 units
and square root of the wave frequency
(B) 20 units and 25 units
(C) directly on the wave frequency and square
(C) 30 units and 20 units
of the wave amplitude
(D) 25 units and 20 units
(D) directly on the wave amplitude and square of
Sol.
the wave frequency.
Sol.

25. A wire of linear mass density 9 x 10–3 kg


23. Two wave pulses travel in opposite directions
/m is stretched between two rigid supports
on a string and approach each other. The shape
under a tension of 360 N. The wire resonates
of the one pulse in inverted with respect to the
at freq uency 210 Hz. The next hi gher
other.
frequency at which the same wire resonates
(A) the pulses will collide with each other and
is 280 Hz. The number of loops produced in
vanish after collision.
first case will be -
(B) the pulses will reflect from each other i.e.,
(A) 1 (B) 2 (C) 3 (D) 4
the pulse going towards right will finally move
Sol.
towards left and vice versa.
(C) the pulses will pass through each other but
their shapes will be modified
(D) the pulses will pass through each other without
any change in their shape.
Sol.

26. The resultant amplitude due to superposition


of two waves
Y1  5sin ( wt  kx ) and y2  5cos ( wt  kx  1500 )
(A) 5 (B) 5 3
(C) 5 2 – 3 (D) 5 2  3

Corporate Head Office : Motion Education Pvt. Ltd., 394 - Rajeev Gandhi Nagar, Kota-5
WAVES Page # 121

Sol. 28. In a stationary wave represented by y = a sin


t cos kx, amplitude of the component progressive
wave is :
a
(A) (B) a (C) 2a (D) None
2
Sol.

27. A stretched sonometer wire resonates at a


frequency of 350 Hz and at the next higher
frequency of 420 Hz. The fundamental frequency
of this wire is :
(A) 350 Hz (B) 5 Hz
(C) 70 Hz (D) 170 Hz
Sol.

: 0744-2209671, 08003899588 | url : www.motioniitjee.com, : info@motioniitjee.com


Page # 122 WAVES

Exercise - II
SINGLE CORRECT 3. A composition String is made up by joining two
1. Two stretched wires A and B of the same strings of different masses per unit length
lengths vibrate independently. If the radius,   and 4. the composite string is under the same
density and tension of wire A are respectively tension. A transverse wave pulse: Y = (6mm) sin
twice those of wire B, then the fundamental (5t+40x), Where ‘t’ is in seconds and ‘x’ in meters,
frequency of vibration of A relative to that of B is is sent along the lighter string towards the joint.
(A) 1 : 1 (B) 1 : 2 The joint is at x=0. The equation of the wave
(C) 1 : 4 (D) 1 : 8 pulse reflected from the joint is
Sol. (A) (2mm) sin (5t-40x)
(B) (4 mm) sin (40x-5t)
(C) –(2 mm) sin (5t-40x)
(D) (2 mm) sin (5t-10x)
Sol.

2. A copper wire is held at the two ends by rigid


supports. At 30°C the wire is just taut, with
negligible tension, The speed of transverse waves
in this wire at 10°C is : 4. In the previous question, the percentage of
(  =1.7 × 10–5/°C, Y = 1.3 × 1011 N/m2, d = 9 × power transmitted to the heavier string through
103 kg/m3). the joint is approximately
(A) 80 m/sec (B) 90 m/sec (A) 33% (B) 89% (C) 67% (D) 75%
(C) 100 m/sec (D) 70 m/sec Sol.
Sol.

Corporate Head Office : Motion Education Pvt. Ltd., 394 - Rajeev Gandhi Nagar, Kota-5
WAVES Page # 123

5. The frequency of a sonometer wire is f, but 7. A harmonic wave is travelling on string 1. At a


when the weights producing the tensions are junction with string 2 it is partly reflected and
completely immersed in water the frequency partly transmitted. The linear mass density of the
becomes f/2 and on immersing the weights in a second string is four times that of the first string,
certain liquid the frequency becomes f/3. The and that the boundary between the two strings
specific gravity of the liquid is: is at x =0. If the expression for the incident wave
4 16 15 32 is, yi = Ai cos (k1x – 1t)
(A) (B) (C) (D) Then findout the expression for the transmitted
3 9 12 27
Sol. wave.
1 3
(A) A i cos(2k 1x –  1t) (B) A i cos(2k 1x –  1t)
3 2
2
(C) A i cos(2k 1x –  1t) (D) None
3
Sol.

6. A wave moving with constant speed on a 8. A taut string at both ends vibrates in its n
th

uniform string passes the point x = 0 with overtone. The distance between adjacent Node
amplitude A0, angular frequency 0 and average and antinode is found to be ‘d’ If the length of
rate of energy transfer P0. As the wave travels the string is L, then
down the string it gradually loses energy and at (A) L=2d(n+1) (B) L=d(n+1)
the point x = , the average rate of energy (C) L=2dn (D) L = 2d(n – 1)
P Sol.
transfer becomes 0 . At the point x = , angular
2
frequency and amplitude are respectively.
(A) 0 and A 0 / 2 (B) 0/ 2 and A0
(C) less than 0 and A0 (D) 0/ 2 and A0 / 2
Sol.

9. A metallic Wire of length L is fixed between


two rigid supports. If the wire is cooled through a
temperature difference T (Y = young’s modulus,
 = density,  = coefficient of linear expansion)
then the frequency of transverse vibration is
proportional to:
 Y  
(A) (B)  (C) (D)
Y Y Y

: 0744-2209671, 08003899588 | url : www.motioniitjee.com, : info@motioniitjee.com


Page # 124 WAVES

Sol. Sol.

20
10. A Standing Wave y  A sin(  x ) cos (1000 t ) is
3
maintained in a taut string where y and x are
expressed in meters. The distance between the
successive points oscillating with the amplitude A/
2 across a node is equal to
(A) 2.5 cm (B) 25 cm (C) 5 cm (D) 10 cm 12. A wave represented by the equation y = a
Sol. cos (kx – t) is superposed with another wave to
form a stationary wave such that the point x = 0
is a node. The equation for other wave is :
(A) a sin (kx + t) (B) – a cos (kx + t)
(C) – a cos (kx – t) (D) – a sin (kx – t)
Sol.

11. A string of length 1m and linear mass density


0.01 kgm 1 is stretched to a tension of 100N.when
MULTIPLE CORRECT
both ends of the string are fixed, the three lowest 13. A wave equation which gives the displacement
frequencies for standing wave are f1, f 2 and f3 . along the Y direction is given by
when only one end of the string is fixed, the Y = 10–4 sin (60t + 2x)
three lowest frequencies for standing wave are where x and y are in metres and t is time in
seconds. This represents a wave
n1, n2 and n3 . Then
(A) travelling with a velocity of 30 m/s in the
(A) n3 = 5n1 = f3 = 125 Hz negative x direction.
(B) f3 = 5f1 = n2 = 125 Hz (B) of wavelength  metre
(C) f3 = n2 = 3f1 = 150 Hz (C) of frequency 30/ hertz
f f (D) of amplitude 10–4 metre travelling along the
(D) n 2  1 2  75 Hz negative x direction.
2

Corporate Head Office : Motion Education Pvt. Ltd., 394 - Rajeev Gandhi Nagar, Kota-5
WAVES Page # 125

Sol. 16. A transverse wave is described by the


equation y = A sin [2(ft – x/)]. The maximum
particle velocity is equal to four times the wave
velocity if :
(A)  = A/4 (B) =A/2 (C)  = A (D)  = 2A
Sol.

14. The displacement of a particle in a medium


due to a wave travelling in the x-direction through
the medium is given by y = A sin (  t – x), where
t = time, and  and  are constants :
(A) the frequency of the wave is 
(B) the frequency of the wave is /2
(C) the wavelength is 2/ 17. A wave equation is given as y = cos(500t –
(D) the velocity of the wave is / 70x), where y is in mm, x in m and t is in sec.
Sol. (A) the wave must be a transverse propagating
wave.
(B) The speed of the wave is 50/7 m/s
(C) The frequency of oscillations 1000 Hz
(D) Two closest points which are in same phase
have separation 20/7 cm.
Sol.

15. A sinusoidal progressive wave is generated in a


string. It’s equation is given by y = (2 mm) sin (2x
– 100 t + /3). The time when particle at x = 4 m
first passes through mean position, will be
1 1 1 1
(A) sec (B) sec (C) sec (D) sec
150 12 300 100
Sol.
18. At a certain moment,
the photograph of a string y
on which a harmonic wave
Q
is travelling to the right is
shown. Then, which of the
P x
following is true regarding R
the velocities of the points
P, Q and R on the string.
(A) vP is upwards (B) vQ = – vR
(C) |vP| > |vQ| = |vR| (D) vQ = vR

: 0744-2209671, 08003899588 | url : www.motioniitjee.com, : info@motioniitjee.com


Page # 126 WAVES

Sol. 24. One end of a string of length L is tied to the


ceiling of a lift accelerating upwards with an
acceleration 2g. The other end of the string is
free. The linear mass density of the string varies
linearly from 0 to  from bottom to top.
(A) The velocity of the wave in the string will be 0.
(B) The acceleration of the wave on the string
will be 3g/4 every where.
(C) The time taken by a pulse to reach from
bottom to top will be 8L / 3g .
(D) The time taken by a pulse to reach from
Question No. 19 to 22 (4 questions) bottom to top will be 4L / 3g .
The figure represents the instantaneous picture Sol.
of a transverse harmonic wave traveling along
the negative x-axis. Choose y b
the correct alternati ve(s)
related to the movement of the a c h x
o
nine points shown in the figure. d
e g
19. The points moving upward is/are f
(A) a (B) c (C) f (D) g
20. The points moving downwards is/are
(A) o (B) b (C) d (D) h
21. The stationary points is/are
(A) o (B) b (C) f (D) h
22. The points moving with maximum speed is/are
(A) b (B) c (C) d (D) h
Sol.
 x
25. A plane wave y = A sin   t   undergo a
 v
normal incidence on a plane boundary separating
medium M1 and M2 and splits into a reflected and
transmitted wave having speeds v1 and v2 then
(A) for all values of v 1 and v 2 the phase of
transmitted wave is same as that of incident wave
(B) for all values of v1 and v2 the phase of reflected
wave is same as that of incident wave
23. A perfectly elastic uniform string is suspended
(C) the phase of transmitted wave depends upon
vertically with its upper end fixed to the ceiling
v1 and v2
and the lower end loaded with the weight. If a
(D) the phase of reflected wave depends upon v1
transverse wave is imparted to the lower end of
and v2
the string, the pulse will
Sol.
(A) not travel along the length of the string
(B) travel upwards with increasing speed
(C) travel upwards with decreasing speed
(D) travelled upwards with constant acceleration
Sol.

Corporate Head Office : Motion Education Pvt. Ltd., 394 - Rajeev Gandhi Nagar, Kota-5
WAVES Page # 127

26. Two waves of equal frequency f and velocity Sol.


v travel in opposite directions along the same
path. The waves have amplitudes A and 3A. Then :
(A) the amplitude of the resulting wave varies
with position between maxima of amplitude 4A
and minima of zero amplitude
(B) the distance between a maxima and adjacent
minima of amplitude is V/2f
(C) at point on the path the average displacement
is zero
(D) the position of a maxima or minima of amplitude
does not change with time
Sol.
29. A standing wave pattern of amplitude A in a
string of length L shows 2 nodes (plus those at
two ends). If one end of the string corresponds
to the origin and v is the speed of progressive
wave, the disturbance in the string, could be
represented (with appropriate phase) as :
 2 x   2vt 
(A) y(x, t) = A sin  cos  
 L   L 
 3 x   2vt 
(B) y(x, t) = A cos  sin 
 L   L 
 4x   4vt 
(C) y(x, t) = A cos  cos 
 L   L 
27. The vibration of a string fixed at both ends
are described by Y = 2 sin(x)sin (100t) where Y  3 x   3vt 
(D) y(x, t) = A sin  cos 
is in mm, x is in cm, t in sec then  L   L 
(A) Maximum displacement of the particle at x = Sol.
1/6 cm would be 1mm.
(B) velocity of the particle at x = 1/6 cm at time
t = 1/600 sec will be 1573 mm/s
(C) If the length of the string be 10 cm, number
of loop in it would be 5
(D) None of these
Sol.

30. The length, tension, diameter and density of


a wire B are double than the corresponding
quantities for another stretched wire A. Then.
1
(A) Fundamental frequency of B is times
28. In a standing wave on a string. 2 2
(A) In one time period all the particles are that of A.
simultaneously at rest twice. 1
(B) All the particles must be at their positive (B) The velocity of wave in B is times that of
2
extremes simultaneously once in one time period. velocity in A.
(C) All the particles may be at their positive (C) The fundamental frequency of A is equal to
extremes simultaneously once in a time period. the third overtone of B.
(D ) Al l the part i c l e s are ne ve r at res t (D) The velocity of wave in B is half that of
simultaneously. velocity in A.

: 0744-2209671, 08003899588 | url : www.motioniitjee.com, : info@motioniitjee.com


Page # 128 WAVES

Sol. 33. Figure, shows a stationary wave between


two fixed points P and Q.
Which point(s) of 1, 2 and 3 are in phase with
the point x?
P X 1 23 Q
(A) 1, 2 and 3 (B) 1 and 2 only
(C) 2 and 3 only (D) 3 only
Sol.

31. A string is fixed at both ends vibrates in a


resonant mode with a separation 2.0 cm between
the consecutive nodes. For the next higher
resonant frequency, this separation is reduced to 34. The equation of a wave disturbance is given
1.6 cm. The length of the string is  
(A) 4.0 cm (B) 8.0 cm (C) 12.0 cm (D) 16.0 cm as : y  0.02 cos  2  50 t cos(10x) , where x and
Sol. y are in meters and t in seconds. Choose the
wrong statement
(A) Antinode occurs at x = 0.3 m
(B) The wavelength is 0.2 m
(C) The speed of the constituent waves is 4m/s
(D) Node occurs at x = 0.15 m
Sol.

32. A clamped string is oscillating in nth harmonic,


then 35. In a stationary wave,
(A) total energy of oscillations will be n2 times (A) all the particles of the medium vibrate in phase
that of fundamental frequency (B) all the antinodes vibrate in phase
(B) total energy of oscillations will be (n – 1)2 (C) the alternate antinodes vibrate in phase
times that of fundamental frequency (D) all the particles between consecutive nodes
(C) average kinetic energy of the string over a vibrate in phase
complete oscillations is half of that of the total Sol.
energy of the string.
(D) none of these.
Sol.

Corporate Head Office : Motion Education Pvt. Ltd., 394 - Rajeev Gandhi Nagar, Kota-5
WAVES Page # 129

Exercise - III (JEE ADVANCED)

1. Consider the wave y = (5 mm) sin (1 cm–1) 3. The string shown in figure is driven at a
x – (60 s–1)t] Find (a) the amplitude (b) the wave frequecy of 5.00 Hz. The amplitude of the motion
number, (c) the wavelength, (d) the frequency, is 12.0 cm, and the wave speed is 20.0 m/s.
(e) the time period and (f) the wave velocity. Furthermore, the wave is such that y = 0 at x =
Sol. 0 and t = 0. Determine (a) the angular frequency
and (b) wave number for this wave. (c) Write an
expression for the wave function. Calculate (d)
the maximum transverse speed and (e) the
maximum transverse acceleration of a point on
the string.

x=0

2. The wave function for a traveling wave on a Sol.


taut string is (in SI unit)
y(x, t) = (0.350 m) sin (10 t – 3x + /4)
(a) What are the speed and direction of travel of
the wave ?
(b) What is the vertical displacement of the string
at t = 0, x = 0.100 m ?
(c) What are wavelength and frequency of the
wave ?
(d) What is the maximum magnitude of the
transverse speed of a particle of the string ?
Sol. 4. Two strings A and B with  = 2 kg/m and  = 8
kg/m respectively are joined in series and kept
on a horizontal table with both the ends fixed.
The tension in the string is 200 N. If a pulse of
amplitude 1 cm travels in A towards the junction,
then fi nd the ampli tude of refl ected and
transmitted pulse.
Sol.

: 0744-2209671, 08003899588 | url : www.motioniitjee.com, : info@motioniitjee.com


Page # 130 WAVES

5. A parabolic pulse given by equation y (in cm) 7. A uniform rope of length 12 m and mass 6 kg
= 0.3 – 0.1 (x – 5t)2 ( y  0) x in meter and t in hange vertically from a rigid support. A block of
second travelling in a uniform string. The pulse mass 2 kg is attached to the free end of the
passes through a boundary beyond which its rope. A transverse pulse of wavelength 0.06 m is
velocity becomes 2.5 m/s. What will be the produced at the lower end of the rope. What is
ampl i tude of pul se i n thi s medi um after the wavelength of the pulse when it reaches the
transmission ? top of the rope ?
Sol. Sol.

6. In the arrangement shown in figure, the string


has mass of 4.5 g. How much time will it take for
a transverse disturbance produced at the floor
to reach the pulley ? Take g = 10 m/s2

25cm

2.0m
2kg
8. A particle on stretched string supporting a
travelling wave, takes 5.0 ms to move from its
mean position to the extreme position. The
distance between two consecutive particles,
Sol. which are at their mean position, is 2.0 cm. Find
the frequency, the wavelength and the wave
speed.
Sol.

Corporate Head Office : Motion Education Pvt. Ltd., 394 - Rajeev Gandhi Nagar, Kota-5
WAVES Page # 131

9. A 6.00 m segment of a long string has a mass 11. The equation of a plane wave travelling along
of 180 g. A high-speed photograph shows the at 2
segment contains four complete cycles of wave. positive direction of x-axis is y = a sin (vt – x)

The string is vibrating sinusoidally with a frequency When this wave is reflected at a rigid surface
of 50.0 Hz and a peak=to-valley displacement of and its amplitude becomes 80%, then find the
15.0 cm. (The "peak-to-valley" displacement is equation of the reflected wave
the vertical distance from the farthest positive Sol.
di sp l ace me nt t o the fart he st neg at i v e
di splacement. (a) Write the function that
describes this wave traveling in the positive x
direction. (b) Determine the power being supplied
to the string.
Sol.

12. A travelling wave of amplitude 5 A is partially


reflected from a boundary with the amplitude 3
A. Due to superposition of two waves with different
amplitudes in opposite directions a standing wave
pattern is formed. Determine the amplitude at
10. A 200 Hz wave with amplitude 1 mm travels node and antinodes.
on a long string of linear mass density 6 g/m kept Sol.
under a tension of 60 N. (a) Find the average
power transmitted across a given point on the
string. (b) Find the total energy associated with
the wave in a 2.0m long portion of the string.
Sol.

: 0744-2209671, 08003899588 | url : www.motioniitjee.com, : info@motioniitjee.com


Page # 132 WAVES

13. Two waves are described by 15. A nylon guitar string has a linear density of
y1 = 0.30 sin [(5x – 200)t] 7.20 g/m and is under a tension of 150 N. The
and y2 = 0.30 sin [(5x – 200t) + /3] fixed supports are distance D = 90.0 cm apart.
where y1, y2 and x are in meters and t is The string is oscillating in the standing wave
in seconds. When these two waves are combined, pattern shown in figure. Calculate the (a) speed
a traveling wave is produced. What are the (a) wavelength, and (c) frequency of the traveling
amplitude, (b) wave speed, and (c) wave length waves whose superposition gives this standing
of that traveling wave ? wave.
Sol.
D

Sol.

14.What are (a) the lowest frequency, (b) the


second lowest frequency, and (c) the third lowest
frequency for standing waves on a wire that is
10.0 m long has a mass of 100 g. and is stretched
under a tension of 250 N which is fixed at both
ends ?
Sol.
16.A string that is stretched between fixed
supports separated by 75.0 cm has resonant
frequencies of 420 and 315 Hz with no intermediate
resonant frquencies. What are
(a) the lowest resonant frequencies and (b) the
wave speed ?
Sol.

Corporate Head Office : Motion Education Pvt. Ltd., 394 - Rajeev Gandhi Nagar, Kota-5
WAVES Page # 133

17. A string oscillates according to the equation 19. A string vibrates in 4 loops with a frequency
of 400 Hz.
  –1  (a) What is its fundamental frequency ?
y = (0.50 cm) sin  3 cm  x cos [(40  s–1)t] (b) What is frequency will cause it to vibrate into
 
7 loops.
What are the (a) amplitude and (b) speed of the Sol.
two waves (identical except for direction of
travel) whose superposition gives this oscillation
? (c) What is the distance between nodes ? (d)
What is the transverse speed of a particle of the
string at the position x = 1.5 cm when t = 9/8 s ?
Sol.

20.A string fixed at both ends is vibrating in the


lowest mode of vibration for which a point at
quarter of its lengths from one end is a point of
maximum displacement. The frequency of vibration
18. In an experiment of standing waves, a string in this mode is 100 Hz. What will be the frequency
90 cm long is attached to the prong of an emitted when it vibrates in the next mode such
electrically driven tuning fork that oscillates that this point is again a point of maximum
perpendicular to the length of the string at a displacement.
frequency of 60 Hz. The mass of the string is Sol.
0.044 kg. What tension must the string be under
(weights are attached to the other end) if it is to
oscillate in four loops ?
Sol.

: 0744-2209671, 08003899588 | url : www.motioniitjee.com, : info@motioniitjee.com


Page # 134 WAVES

21. A guitar string is 90 cm long and has a 23. A stretched uniform wire of a sonometer
fundamental frequency of 124 Hz. Where should between two fixed knife edges, when vibrates in
it be pressed to produced a fundamental its second harmonic gives 1 beat per second with
frequency of 186 Hz ? a vibrating tuning fork of frequency 200 Hz. Find
Sol. the percentage change in the tension of the wire
to be in unison with the tuning fork.
Sol.

22. A 2.00 m long rope, having a mass of 80 g, is


fixed at one end and is tied to a light string at 24. A sonometer wires resonates with a given
the other end. The tension in the string is 256 N. tuning fork forming standing waves with five
(a) Find the frequencies of the fundamental and antinodes between the two bridges when a mass
the first two overtones. (b) Find the wavelength of 9 kg is suspended from the wire. When this
in the fundamental and the first two overtones. mass is replaced by M, the wire resonates with
Sol. the same tuning fork forming three antinodes for
the same position of bridges. Find the value of M.
Sol.

Corporate Head Office : Motion Education Pvt. Ltd., 394 - Rajeev Gandhi Nagar, Kota-5
WAVES Page # 135

25. A 40 cm long wire having a mass 3.2 gm and at its middle. Assuming young’s modulus to be 2
area of c.s. 1 mm2 is stretched between the × 1011 Pa, and density to be 8 gm/cm3 find the
support 40.05 cm apart. In its fundamental mode. fundamental frequency of the longitudinal
It vibrate with a frequency 1000/64 Hz. Find the vibration and frequency of first overtone.
young’s modulus of the wire. Sol.
Sol.

26. A steel rod having a length of 1m is fastened

: 0744-2209671, 08003899588 | url : www.motioniitjee.com, : info@motioniitjee.com


Page # 136 WAVES

1. The figure shows a snap photograph of a


4. In a stationary wave pattern that forms as a
vibrating string at t = 0. The particle P is
result of reflection of waves from an obstacle
observed moving up with velocity 20 cm/s. The
the ratio of the amplitude at an antinode and a
angle made by string with x-axis at P is 6°.
node is  = 1.5. What percentage of the energy
passes across the obstacle?
y
(in10–3m)
4 P 5. A string, 25 cm long, having a mass of 0.25
x gm/cm, is under tension. A pipe closed at one
0 1.5 3.5 5.5 7.5
–2
(in10 m) end is 40 cm long. When the string is set vibrat-
ing in its first overtone, and the air in the pipe in
its fundamental frequency, 8 beats/sec are heard.
(a) Find the direction in which the wave is mov-
It is observed that decreasing the tension in the
ing
string, decreases the beat frequency. If the speed
(b) the equation of the wave of sound in air is 320 m/s, find the tension in the
(c) the total energy carried by the wave per cycle string.
of the string, assuming that , the mass per unit
length of the string = 50 gm/m. 6. A metal rod of length l = 100 cm is clamped at
two points. Distance of each clamp from nearer
2. A uniform rope of length L and mass m is held end is a = 30cm. If density and Young’s modulus
at one end and whirled in a horizontal circle with of elasticity of rod material are  = 9000 kgm–3
angular velocity . Ignore gravity. Find the time and Y = 144 GPa respectively, calculate minimum
required for a transverse wave to travel from one and next higher frequency of natural longitudinal
end of the rope to the other. oscillations of the rod.

3. A symmetrical triangular pulse of maximum


height 0.4m and total length 1 m is moving in the
positive x-direction on a string on which the wave
speed is 24 m/s. At t = 0 the pulse is entirely
located between x = 0 and x = 1 m. Draw a graph
of the transverse velocity of particle of string
versus time at x = +1m.

Corporate Head Office : Motion Education Pvt. Ltd., 394 - Rajeev Gandhi Nagar, Kota-5
WAVES Page # 137

Exercise - IV PREVIOUS YEAR QUESTIONS

LEVEL - I JEE MAIN

Sol.
1. A wave y = a sin ωt  kx  on a string meets
with another wave producing a node at x= 0.
Then the equation of the unknown wave is
` [AIEEE 2002]
(A) y  a sin ωt  kx  (B) y   a sin ωt kx 
(C) y  a sin ωt kx  (D) y   a sin ωt kx 
Sol.

2. Length of a string tied to two rigid supports is 4. A metal wire of linear mass density of 9.8 gm–1 is
40 cm. Maximum length (wavelength in cm) of a stretched with a tension of 10 kg-wt between
stationary wave produced on it, is two rigid supports 1 m apart. The wire passes at
[AIEEE 2002] its middle point between the poles of a permanent
(A) 20 (B) 80 (C) 40 (D) 120 magnet and it vibrates in resonance when carrying
Sol. an alternating current of frequency n. The
frequency n of the alternating source is
[AIEEE 2003]
(A) 50 Hz (B) 100 Hz
(C) 200 Hz (D) 25 Hz
Sol.

3. The displacement of a particle varies according


to the relation x  4 cos πt  sin πt . The amplitude
of the particle is [AIEEE 2003]
(A) - 4 (B) 4 (C) 4 2 (D) 8

: 0744-2209671, 08003899588 | url : www.motioniitjee.com, : info@motioniitjee.com


Page # 138 WAVES

5. The displacement y of a wave travelling in the Sol.


x-direction is given by [AIEEE 2003]

 π
y  10 4 sin  600 t  2 x   metre,
 3
where, x is expressed in metres and t in seconds.
The speeds of the wave-motion, in ms-1 is
(A) 300 (B) 600 (C) 1200 (D) 200
Sol.

6. The displacement y of a particle in a medium


can be expressed as
8. A wave travelling along the x-axis is described
 π
y  10 sin 100 t  20 x   m, where t is by the equation y(x,t) = 0.005 cos αx βt . If
6

 4 the wavelength and the time period of the wave


in second and x in metre. The speed of the wave are 0.08 m and 2.0s, respectively. then α and β
is [AIEEE 2004]
in appropriate units are [AIEEE 2008]
(A) 2000 ms–1 (B) 5 ms–1
(C) 20 ms–1 (D) 5 ms–1 (A) α  25.00 π, β  π
Sol.
0.08 2.0
(B) α  ,β 
π π
0.04 1.0
(C) α ,β 
π π
π
(D) α  12.50 π, β 
2.0
Sol.

7. A string is stretched between fixed points


separated by 75.0 cm. It is observed to have
resonant frequencies of 420 Hz and 315 Hz. There
are no other resonant frequencies between these
two. Then, the lowest resonant frequency for
this string is [AIEEE 2006]
(A) 105 Hz (B) 1.05 Hz
(C) 1050 Hz (D) 10.5 Hz

Corporate Head Office : Motion Education Pvt. Ltd., 394 - Rajeev Gandhi Nagar, Kota-5
WAVES Page # 139

9. The equation of a wave on a string of linear 11. A t rave l l i ng w ave re p res e nt e d b y


mass density 0.04 kg m–1 is given by  
y  Asin ωt  kx is superimposed on another
  t wave represented by y  Asin  ωt  kx  . The
x 
y = 0.02 (m) sin 2 π    resultant is [AIEEE 2011]
  0.04(s) 0.50(m)  (A) A s t and i ng w ave hav i ng node s at
The tension in the string is [AIEEE 2009]  1λ
(A) 4.0 N (B) 12.5 N
x   n   , n  0,1,2
 22
(C) 0.5 N (D) 6.25 N (B) A wave travelling along + x direction
Sol. (C) A wave travelling along - x direction
(D) A standing wave havi ng nodes at

x ; n=0, 1, 2
2
Sol.

10. The transverse displacement y (x, t) of awave


12. Statement I Two longitudinal waves given
on a string is given by [AIEEE 2011]
yx, t   e ax  x, t   2 asinωt kx  and
2 2
 bt  2 ab xt
This represents a by eq uati ons - y1

b y 2 x, t   asin2 ωt  2 kx  w i l l hav e e qual


(A) wave moving in-x direction with speed
a intensity. [AIEEE 2011]
(B) standing wave of frequency Statements II Intensity of waves of given
b
frequency in same medium is proportional to
1 square fo amplitude only.
(C) standing wave of frequency
b (A) Statement I is true, Statement II is true
a (B) Statement I is true, Statement II is false
(D) wave moving in + x direction with speed (C) Statemetn I is true, Statement II true;
b Statement II is the correct explanation of
Sol.
Statement I
(D) Statement I is true, Statement II is true;
Statement II is not correct explanation of
Statement I
Sol.

: 0744-2209671, 08003899588 | url : www.motioniitjee.com, : info@motioniitjee.com


Page # 140 WAVES

LEVEL - II JEE ADVANCED


1. A sonometer wire resonates with a given tuning Sol.
fork forming standing waves with five antinodes
between the two bridges when a mass of 9 kg is
suspended from the wire. When this mass is
replaced by mass M, the wire resonates with the
same tuning fork forming three antinodes for the
same positions of the bridges. The value of M is
[JEE-2002(Scr), 3]
(A) 25 kg (B) 5 kg (C) 12.5 kg (D) 1/25 kg
Sol.

4. A transverse harmonic disturbance is produced


in a string. The maximum transverse velocity is 3
m/s and maximum transverse acceleration is 90
m/s2. If the wave velocity is 20 m/s then find the
waveform. [JEE-2005]
Sol.

2. A stringe between x = 0 and x = l vibrates in


fundamental mode. The amplitude A, tension T
and mass per unit length  is given. Find the total
energy of the string. [JEE-2003]
x=0 x=l

Sol.

5. A massless rod is suspended by two identical


strings AB and CD of equal length. A block of
mass m is suspended from point O such that BO
is equal to ‘x’. Further, it is observed that the
frequency of 1st harmonic (fundamental frequency)
in AB is equal to 2nd harmonic frequency in CD.
3. A string fixed at both ends is in resonance in
Then, length of BO is [JEE-2006]
its 2nd harmonic with a tuning fork of frequency
f 1 . Now its one end becomes free. If the C
A
frequency of the tuning fork is increased slowly
from f1 then again a resonance is obtained when
the frequency is f2 . If in this case the string
vibrates in nth harmonic then [JEE-2005(Scr)] B D

3 5 L
(A) n = 3, f2  f1 (B) n = 3, f2  f1 x
4 4 m
5 3 L L 4L 3L
(C) n = 5, f2  f1 (D) n = 5, f2  f1 (A) (B) (C) (D)
4 4 5 4 5 4

Corporate Head Office : Motion Education Pvt. Ltd., 394 - Rajeev Gandhi Nagar, Kota-5
WAVES Page # 141

Sol. Sol.

8. W he n tw o prog re ss i v e wave s

 
6. A transverse sinusoidal wave moves along a y1 = 4 sin (2x – 6t) and y2 = 3 sin  2x  6t  
 2
string in the positive x-direction at a speed of 10
cm/s. The wavelength of the wave is 0.5 m and are superimposed, the ampl itude of the
its amplitude is 10 cm. At a particular time t, the resultant wave is : [JEE 2010]
snap-shot of the wave is shown in figure. The Sol.
velocity of point P when its displacement is 5 cm
is Figure : y
P

3  3 
(A) j m/s (B) – j m/s
50 50
3  3 
(C) i m/s (D) – i m/s
50 50
[JEE-2008]
Sol.

7. A 20 cm long string, having a mass of 1.0 g, is


fixed at both the ends. The tension in the string
is 0.5 N. The string is set into vibrations using an
external vibrator of frequency 100 Hz. Find the
separation (in cm) between the successive nodes
on the string. [JEE 2009]

: 0744-2209671, 08003899588 | url : www.motioniitjee.com, : info@motioniitjee.com


Page # 142 WAVES

:: ANSWER KEY ::

Exercise - I OBJECTIVE PROBLEMS (JEE MAIN)

1. B 2. D 3. B 4. A 5. A 6. D 7. D
8. C 9. A 10. A 11. D 12. C 13. C 14. C
15. A 16. D 17. B 18. D 19. D 20. C 21. C
22. A 23. D 24. B 25. C 26. A 27. C 28. A

Exercise - II
1. B 2. D 3. C 4. B 5. D 6. A 7. C
8. A 9. B 10. C 11. D 12. B 13. A,B,C,D
14. B,C,D 15. C 16. B 17. A,B,D 18. C,D 19. A,D 20. C
21. B,C 22. C,D 23. B,D 24. B,C 25. A,D 26. C,D 27. A,B
28. A,C 29. D 30. C,D 31. B 32. A,C 33. C 34. C
35. C,D

Exercise - III (JEE ADVANCED)

1. (a) amplitude A = 5 mm (b) wave number k = 1 cm–1

2  60
(c) wavelength  = = 2 cm (d) frequency v = = Hz
k 2 2

1 
(e) time period T = = s (f) wave velocity u = n = 60 cm/s
v 30

10 
2. (a) i m / s (b) –5.48 cm (c) 0.667 m, 5.00 Hz (d) 11.0 m/s
3
3. (a) 10  rad/s (b) /2 rad/m (c) y = (0.120m) sin (1.57x – 31.4 t) (d) 1.2  m/s (e) 118 m/s2
1 2
4. Ar = – cm, At = cm 5. 0.2 cm 6. 0.02 s 7. 0.12 m 8. 50 Hz, 4.0 cm, 2.0 m/s
3 3
9. (a) y = (7.50 cm) sin (4.19 x – 314 t) (b) 625 W 10. (a) 0.47 W, (b) 9.4 mJ

2 
11. y = 0.8 a sin ( vt  x  ) 12. 2 A, 8 A 13. (a) 0.52 m ; (b) 40 m/s ; (c) 0.40 m
 2

5 10 5 10
14. (a) Hz ; (b) 5 10 Hz ; (c) Hz 15. (a) 144 m/s ; (b) 60.0 cm ; (c) 241 Hz
2 2
16. (a) 105 Hz ; (b) 158 m/s 17. (a) 0.25 cm (b) 1.2 × 102 cm/s; (c) 3.0 cm; (d) 0
18. 36 N 19. (a) 100 Hz (b) 700 Hz 20. 300 Hz 21. 60 cm from an end.
22. (a) 10 hz, 30 Hz, 50 Hz

Corporate Head Office : Motion Education Pvt. Ltd., 394 - Rajeev Gandhi Nagar, Kota-5
WAVES Page # 143

(b) 8.00 m, 1.60 m


23. 1% 24. 25 kg 25. 1 × 109 Nm2 26. 2.5 kHz, 7.5 kHz

 1  
1. (a) negative x; (b) y = 4 × 10–3 sin 100  3 t  0.5 x  400  (x, y in meter) ; (c) 144  × 10–5 J 2.
2

vp
19.2 m/s
3. 1/48 sec1/24 sec t 4. 96% 5. 67.6 N 6. 10 kHz, 30 kHz
–19.2m/s

Exercise - IV PREVIOUS YEAR QUESTIONS

LEVEL - I JEE MAIN


1. B 2. B 3. C 4. A 5. A 6. B 7. A
8. A 9. D 10. A 11. A 12. B

LEVEL - II JEE ADVANCED

A 22 T 3
1. A 2. E  3. C 4. y = (10 cm) sin (30 t ± x + f))
4l 2
5. A 6. A 7. 5 8. 5

: 0744-2209671, 08003899588 | url : www.motioniitjee.com, : info@motioniitjee.com


Page # 144 SOUND WAVES

SOUND WAVES
1. SOUND WAVES
Sound is type of longitudnal wave. In general majority of longitudinal waves are termed as sound
waves. Sound is produced by a vibrating source, like when a gong of a bell is struck with a hammer,
sound is produced. The vibrations produced by gong are propagated through air, Through air these
vibrations reach to the ear and ear drum is set into vibrations and these vibrations are communicated
to human brain. By touching the gong of bell by hand, we can feel the vibrations.

2. PROPAGATION OF SOUND WAVES


Sound is a mechanical three dimensional and longitudinal wave that is created by a vibrating source
such a guitar string, the human vocal cords, the prongs of a tuning fork or the diaphragm of a
loudspeaker. Being a mechanical wavs, sound needs a medium having properties of inetia and elasticity
for its propagation. Sound waves propagate in any medium through a series of periodic compressions
and rarefactions of presure, which is produced by the vibrating source.
Consider a tuning fork producing sound waves.
A B

. .
. . . . . . .
. .
. . . . . . .
. .
. . . . . . .
. .
. . . . . . .
. .
. . . . . . .
. .
. . . . . . .
normal atmospheric
pressure
Undisturbed tuning fork
A B

. . . . . . . . .
When Prong B moves outward towards right it compresses . . . . . . . . .
. .... ...... . .v. . . . .
the air in front of it, causing the pressure to rise slightly. . ................ . . . . . . .
. . . . . . . . .
The region of increased pressure is called a compression
pulse and it travels away from the prong with the speed of normal atmospheric
pressure
sound Undisturbed tuning fork
After producing the compression pulse, the prong B reverses its motion and moves inward. This drages
away some air from the region in front of it, causing the pressure to dip slightly below the normal
pressure. This region of decreased pressure is called rarefaction pulse. Following immediately behind
the compression pulse, the rarefaction pulse also travels away from the prong with the speed of
sound.

A B

. . . . . . . . .
. . . . . . . . .
. . . . v. . . . .
. . . . . . . . .
. . . . . . . . .
rarefaction compression
pulse pulse

A longitudinal wave in a fluid is described either in tems of the longitudal displacements suffered by the
particles of the medium.

y  A sin t y  A sin ( t – x / v )

–A (0,0) A y A
v
x

Corporate Head Office : Motion Education Pvt. Ltd., 394 - Rajeev Gandhi Nagar, Kota-5 (Raj.)
SOUND WAVES Page # 145

Consider a wave going in the x-direction in a fluid. Suppose that at a time t, the particle at the
undistrubed position x suffers a displacement y in the x-direction.

 x
y = A sin   t –  ...(i)
 v 

Position of any particle from origin at any time = x + y


x = Distance of the mean position of the particle from the origin.
y = Displacement of the particle from its mean position.
General Equation :
(0,0)  y = A sin (t + )
(0,x)  y = A sin [(t – x/v ) + 
Displacement wave y = A sin (t – kx + )
• If we fix x = x0 then we are dealing with the particle whose mean position at distance x0 from origin &
this particle is performin SHM of amp. A with time period T phase difference = – kx + 

3. COMPRESSION WAVES
When a longitudinal wave propagated in a gaseous medium, it produces compression and rarefaction in
the medium periodically. The region where compression occurs, the pressure is more than the normal
pressure of the medium. Thus we can also describe longitudinal waves in a gaseous medium as
pressure waves and these are also termed as compression waves in which the pressure at different
point of medium also varies periodically with their displacements. Let us discuss the propagation of
excess pressure in a medium in longitudinal wave analytically.
y y+dy

A A' B B'
X
dx
x=x x=x+dx
Consider a longitudinal wave propagating in positive x-direction as shown in figure. Figure shows a
segmentAB of the medium of width dx. In this medium let a longitudinal wave is propagating whose
equation is given as
y = A sin (t – kx) ...(1)
Where y is the displacement of medium particle situated at a distance x from the origin, along the
direction of propagation of wave. In figure shown AB is the medium segment whose a medium particle
is at position x = x and B is at x = x +dx at an instant. If after some time t medium particle at A reaches
to a point A' which is displaced by y and the medium particle at b reaches to point B which is at a
displacmenet y + dy from B. Here dy is given by equation (3.116) as
dy = – Ak cos (t – kx) dx
Here due to displacement of section AB to AB the change in volume of it's section is given as
dV = – S dy [S  Area of cross-section]
= SA k cos (t – kx) dx
The volume of section AB is V = S dx
Thus volume strain in section AB is
dV – SAk cos(t – kx)dx dV
 or  – Ax cos(t – kx)
V Sdx V
If B is the bulk modulus of the medium, then the excess pressure in the section AB can be given as
dV 
P = – B  ...(2)
 V

: 0744-2209671, 08003899588 | url : www.motioniitjee.com, : info@motioniitjee.com


Page # 146 SOUND WAVES

P = BAk cos (t – kx)


or P = P0cos(t – kx) ...(3)
Here P0 is the pressure amplitude at a medium particle at position x from origin and P is the excess
pressure at that point. Equation shown that excess varies periodically at every point of the medium
with pressure amplitude P0, which is given as
2
P0 = BAk = AB ...(4)

Equation shown is also termed as the equation of pressure wave in gaseous medium. We can also see
that the pressure wave differs in phase is /2 from the displacement wave and pressure maxima occurs
where the displacement is zero and displacement maxima occur where the pressure is at its normal
level. Remembers that pressure maxima implies that the presure at a point is pressure amplitude times
more or less then the normal pressure level of the medium.
Ex.1 A sound wave of wavelenth 40 cm travels in air. If the difference between the maximum and
minimum pressures at a given point is 2.0 × 10–3 N/m2, find the amplitude of vibration of the
particles of the medium. The bulk modulus of air is 1.4 × 105 N/m2
Sol. The pressure amplitude is
2.0  10 –3 N / m2
p0   10 –3 N / m 2
2
The displacement amplitudes s0 is given by
p0 = B k s0
p 0 p0  10 –3 N / m 2  (40  10 –2 m) 100
or, s0     Å  6.6 Å
Bk 2B 2    14  10 4 N / m 2 7
Density Wave
In this section we will find the relation between pressure wave and density wave.
According to definition of bulk modulus (B),

 dp 
B = – 
 dV / V 

mass
Further, Volume = density

m m V
or V= or dV = – 2
.d  – .d
  

dV d
or –
V 
Substituting in Eq. (i), we get

( dP) dp  1 
d   2   2
B v B v 
Or this can be written as,
 1
  .P  2 P
B v
So, this relation relates the pressure equation with the density equation. For example, if
P = (P)m sin (kx – t)
then  = ()msin (kx – t)
 ( P )m
where, ()m = .( P )m 
B v2
Thus, density equation is in phase with the pressure equation and this is 90° out of phase with the
displacement equation.

Corporate Head Office : Motion Education Pvt. Ltd., 394 - Rajeev Gandhi Nagar, Kota-5 (Raj.)
SOUND WAVES Page # 147

3.1 Velocity and Acceleration of particle :


General equation of wave is given by
y = A sin (t – kx)
y
v =  t = A  cos (t – kx) ...(1)

2 y
a = = – A2 sin (t – kx) ...(2)
 t2

y
 –Ak cos(t – kx) ...(3)
x

y
Here x
= slope of (y, x) curve Now again differentiate eq. – 3

2 y
 –Ak 2 sin(t – kx) ...(4)
 x2
from eq. (2) & (4)
2 y 2 y
2
 v2
t x 2

4. VELOCITY OF SOUND/LONGITUDINAL WAVES IN SOLIDS


Consider a section AB of medium as shown in figure(a) of cross-sectional area S. Let A and B be two
cross section as shown. Let in this medium sound propagation is from left to right. If wave source is at
origin O and when it oscillates, the oscillations at that point propagate along the rod.
velocity of sound
A' B'
A B

O x+y dx+dy
O x dx
(b)
(a)
Here we say an elastic wave has propagated along the rod with a velocity determined by the physical
properties of the medium. Due to oscillations say a force F is developed at every point of medium which
produces a stress in rod and is the cause of strain or propagation of disturbance along the rod. This
stress at any cross-sectional area can be given as
F
Stress S1 = ...(1)
S
If we consider the section AB of medium at a general instant of time t. The end A is at a distance x
from O and B is at a distance x + dx from O. Let in time dt due to oscillations, medium particles at a are
displaced along the length of medium by y and those at B by y + dy. The resulting position of section
and A and B shown in figure (b), Here we can say that the section AB is deformed (elongated) by a
length dy. Thus strain produced in it is
dy
Strain in section AB E ...(2)
dx
If Young's modulus of the material of medium is Y, we have
Stess S
Young's Modulus Y = = 1
Strain E

: 0744-2209671, 08003899588 | url : www.motioniitjee.com, : info@motioniitjee.com


Page # 148 SOUND WAVES

F/S
From equation (1) and (2), we have Y 
dy / dx

dy
or F  YS ...(3)
dx
If net force acting of secting AB is dF then it is given as
dF = dma ...(4)
Where dm is the mass of section AB and a be its acceleration, which can be given as for a medium of
density .

d2 y
dm = Sdx and a=
dt 2

d2 y
From equation (4), we have dF = ( Sdx)
dx2

dF d2 y
or  S 2 ...(5)
dx dt
From equation (3) on differentiating w.r. to x, we can write

dF d2 y
 YS 2 ...(6)
dx dt
From equation (5) and (6) we get

d2 y  Y  d2 y
   ....(7)
dx2    dx2
Equation (7) is the differential form of wave equation, comparing it with previous equation we get the
wave velocity in the medium can be given as

Y
v

Similar to the case of a solid in fluid, instead of Young's Modulus we use Bulk modulus of the medium
hence the velocity of longitudinal waves in a fluid medium is given as

B
v

Where B is the Bulk modulus of medium.
For a gaseous medium bulk modulus is defined as
dp dP
B or B  –V
(– dV / V ) dV

4.1 Newton's Formula for velocity of Sound in Gases


Newton assumed that during sound propagation temperature of medium remains constant hence the
stated that propagation of sound in a gasesous medium is an isothermal phenomenon, thus Boyal's law
can be applied in the process. So for a section of medium we use
PV = constant
Differentiating we get
PdV + V dP = 0
dP
or –V P
dV

Corporate Head Office : Motion Education Pvt. Ltd., 394 - Rajeev Gandhi Nagar, Kota-5 (Raj.)
SOUND WAVES Page # 149

or bulk modulus of medium can be given as


B = P (Pressure of medium)
Newton found that during isothermal propagation of sound in a gaseous medium, bulk modulus of
medium is equal to the pressure of the medium, hence sound velocity in a gaseous medium can be
given as

B P
v  ...(1)
 

P RT
From gas law we have  ...(2)
 M

RT
From (1) & (2) we have v ...(3)
M

From the expression in equation (1) if we find the sound veloicty in air at normal temperature and
atompsheric pressure we have
Normal atmospheric pressure is P = 1.01 × 105 Pa
Density of air at NTP is  = 1.293 kg/m3

P .  10 5
101
Now from equation (1) v=   v = 279.45 m/s
1293
.
But the experimental value of veloicty of sound determined from various experiments gives the velocity
of sound at NTP, 332 m/s. Therefore there is a difference of about 52 m/s between the theoretical and
experimental values. This large difference can not be attributed to the experimental errors. Newton
was unable to explain error in his formula. This correction was explained by a French Scientist Laplace.

4.2 Laplace Correction


Laplace explained that when sound waves propagated in a gaseous medium. There is compression and
rarefaction in the particles of medium. Where there is compression, particles come near to each other
and are heated up, where there is rarefaction, medium expends and there is fall of temperature.
Therefore, the temperature of medium at every point does not remain constant so the process of
sound propagation is not isothermal. The total quantity of heat of the system as a whole remains
constant. medium does not gain or loose any heat to the surrounding. Thus in a gaseous medium sound
propagation is an adiabatic process. For adiabatic process the relation in pressure and volume of a
section of medium can be given as
PV = constant ...(1)
Cp
Here  = C , ratio of specific heats of the medium.
v

Differentiating equation (1) we get,


dPV + V–1 dVP = 0
PdV
or dP +  =0
dV
dP
or –V  P
dV
Bulk modulus of medium B = P
Thus Laplace found that during adiabatic propagation of sound, the Bulk modulus of gaseous medium is
equal to the product of ratio of specific heats and the pressure of medium. Thus velocity of sound
propagation can be given as

B P
v 
 

: 0744-2209671, 08003899588 | url : www.motioniitjee.com, : info@motioniitjee.com


Page # 150 SOUND WAVES

RT
From gas law v 
M
From above equation we find sound velocity in air at NTP, we have
Normal atmospheric pressure P = 1.01 × 105 Pa
Density of air at NTP P = 1.293 kg/m3
Cp
Ratio of specific heat of air   C  142
.
v

 142 .  10 5
.  101
 v = = 333.04 m/s
 1293
.
This value is in agreement with exerimental value.
1/ 2
R( 273  t) R273  t 
Now at any temperature t°C velocity of sound vt = = 1  
M M  273 

 t 
vt = v0  1  
 546 
4.3 Effect of Temperature on Velocity of Sound
We have velocity of sound propagation in a gasous medium as
RT
v
M
For a given gaseous medium , R and M remains constant, thus velocity of sound is directly proportional
to square root of absolute temperature of the medium. Thus
v T
If at two different temperatures T1 and T2, sound velocities in medium are v1 and v2 then from above
equation we have
v1 T1

v2 T2
4.4 Effect of Pressure on Velocity of Sound
We know form gas law
P RT

 M
If temperature of a medium remains constant then on changing pressure, density of medium proportionally
P
changes so that the ratio  remains constant.

Hence if in a medium, T = constant


P
Then,  = constant

P
Thus velocity of sound, v=  = constant
Therefore, the velocity of sound in air or in a gas is independent of change in pressure.
4.5 Effect of Humidity on Velocity of Sound
The density of water vapour at NTP is 0.8 kg/m3 whereas the density of dry air at NTP is 1.293 kg/3.
Therefore water vapour has a density less than the density of dry air. As atmospheric pressure
remains approximately same, the velocity of sound is more in moist air then the velocity of sound in
dry air.
vmoist air > vdry air (from the previous equation)

Corporate Head Office : Motion Education Pvt. Ltd., 394 - Rajeev Gandhi Nagar, Kota-5 (Raj.)
SOUND WAVES Page # 151

4.6 Effect of Wind on Velocity of Sound


If wind is blowing in the direction of propagation of sound, it will increase the velocity of sound, On the
other hand if wave propagation is opposite to the direction of propagation of wind, wave velocity is
decreased. If wind blows at speed v then sound velocity in the medium can be given as
  
v  vs  v w

Where v s is the velocity of sound in still air..

5. APPEARANCE OF SOUND TO HUMAN EAR


The appearance of sound to a human ear is characterised by three parameters (a) pitch (b) loudness
and (c) quality.
(a) Pitch and Frequency
Pitch of a sound is that sensation by which we differentiate a buffalo voice, a male voice and a female
voice. We say that a buffalo voice is of low pitch, a male voice has higher pitch and a female voice has
still higher pitch. This sensation primarly depends on the dominant frequency present in the sound.
Higher the frequency, higher will be the pitch and vice versa.
(b) Loudness and Intensity
The loudness that we sense is related to the intensity of sound though it is not directly proportional to
it. Our perception of loudness is beter correlated with the sound level measured in decibels (abbreviated
as dB) and defined as follows.

I
  10 log10  
 I0 
where I is the intensity of the sound and I0 is a constant reference intensity 10–12 W/m2 The reference
intensity represents roughly the minimum intensity that is just audible at intermediate frequencies. For
I = I 0 , the sound level  = 0.

(c) Quality and Waveform


A sound generated by a source may contain a number of frequency components in it. Different
frequency components have different amplitudes and superposition of them results in the actual
waveform. The appearance of sound depends on this waveform apart from the dominant frequency
and intensity. Figure shows waveforms for a tuning fork, a clarinet and a cornet playing the same note
(fundamental frequency = 440 Hz) with equal loudness.

p
t

p
t

p
t

We differenatiate between the sound from a tabla and that from a mridang by saying that they have
different quality.

: 0744-2209671, 08003899588 | url : www.motioniitjee.com, : info@motioniitjee.com


Page # 152 SOUND WAVES

(d) Energy in sound Waves


Pavg = 22 f2A2 v
Intensity = 22 A2 f 2  v
P  P0 cos( t – kx )

P0
P0  3 Ak  A  K

2 2 2
 2 A 2 v 2 P0 v 2 P0 v 3 P0 v 2 .v
Intensity = =  
2 2  2k 2 2  2 2 2 2

2
P0  .v   
 v 
2 2    

P02 V
I
2.

6. ANALYTICAL TREATMENT OF INTERFERENCE OF WAVES

S1
x1
A1 sin(t  kx) y1  A1 sin(t  kx1 )

y2  A 2 sin(t  kx2 )
x2
S2
A 2 sin(t  kx)

Interference implies super position of waves. Whenever two or more than two waves superimpose
each other at some position then the resutant displacement of the particle is given by the vector sum
of the individual displacements.
Let the two waves coming from sources S1 & S2 be
y1 = A1 sin (t + kx1 )
y2 = A2 sin (t + kx2) respectively.
Due to superposition
ynet = y1 + y2
ynet = A1 sin (t + kx1) + A2 sin (t + kx2)
Phase difference between y1 & y2 = k(x2 – x1)
i.e.,  = k(x2 – x1)
2
As  = x (where x = path difference &  = phase difference)

Anet = A 12  A 22  2A 1A 2 cos 

 A net 2  A 12  A 22  2A 1A 2 cos 

 Inet = I1 + I2 + 2 I1I2 cos  (as I  A2)


When the two displacements are in phase, then the resultant amplitude will be sum of the two
amplitude & Inet will be maximum, this is known of constructive interference.
For Inet to be maximum
cos = 1   = 2n where n = {0,1,2,3,4,5...........}
2
x  2n  x = n

Corporate Head Office : Motion Education Pvt. Ltd., 394 - Rajeev Gandhi Nagar, Kota-5 (Raj.)
SOUND WAVES Page # 153

For constructive interference


Inet = ( I1  I2 )2
When I1 = I2 = I
Inet = 4 I
Anet = A1 + A2
When superposing waves are in opposite phase, the resultant amplitude is the difference of two
amplitudes & Inet is minimum; this is known as destructive interference.
For Inet to be minimum,
cos  = – 1
 = (2n + 1)  where n = {0,1,2,3,4,5...........}
2 
x = (2n + 1)  x = ( 2n  1)
 2
For destructive interference
Inet = ( I1 – I2 )2
If I1 = I2
Inet = 0
Anet = A1 – A2
Generally,
Inet = I1 + I2 + 2 I1I2 cos 
If I1 = I2 = I
Inet = 2I + 2Icos

Inet = 2I(1 + cos ) = 4Icos2
2
( I1  I2 ) 2
Ratio of Imax & Imin =
( I1 – I2 )2

7. LONGITUDINAL STANDING WAVES


Two longitudinal waves of same frequency and amplitude travelling in opposite directions interfere to
produce a standing wave.
If the two interfering waves are given by
p1 = p0 sin (t – kx) and p2 =p0 sin (t + kx + )
then the equation of the resultant standing wave would be given by
 
p = p1 + p2 = 2p0cos (kx + ) sin (t + )
2 2

 p = p0' sin (t + ) ...(1)
2
This is equation of SHM* in which the amplitude p0' depends on position as

p0' = 2p0cos (kx + ) ...(2)
2
Points where pressure remains permanently at its average value, i.e., pressure amplitude is zero is
called a pressure node, and the condition for a pressure node would be given by
p0 ' = 0

i.e. cos(kx  )  0
2
 
i.e. kx + = 2n ± , n = 0,,1,2.......
2 2
Similarly points where pressure amplitude is maximum is called a pressure antinode and condition for a
pressure antinode would be given by

: 0744-2209671, 08003899588 | url : www.motioniitjee.com, : info@motioniitjee.com


Page # 154 SOUND WAVES

p0' = ±2p0

i.e. cos (kx + )=±1
2

or (kx + ) = n, n = 0, 1, 2,.......
2
Note :
• Note that a pressure node in a standing wave would correspond to a displacement antinode; and a
pressure anti-node would correspond to a displacement node.
• (when we label eqn. (1) as SHM, what we mean that excess pressure at any point varies simple
harmonically. if the sound waves were represented in terms of displacement waves, then the equation
of standing wave corresponding to (1) would be
 
s = s0' cos (t +) where s0' = 2s0 sin (kx + )
2 2
This can be easily observed to be an equation of SHM. It represents the medium particles moving
simple harmonically about their mean position at x.

8. REFLECTION OF SOUND WAVES


Reflection of sound waves from a rigid boundary (e.g. closed end of an organ pipe) as analogous to
reflection of a string wave from rigid boundary; reflection accompanied by an inversion i.e. an abrupt
phase change of p. This is consistent with the requirement of displacement amplitude of remains zero
at the rigid end, since a medium particle at the rigid end can not vibrate. As the excess pressure and
displacement corresponding to the same sound wave vary by /2 in term of phase, a displacement
minima at the rigid end will be a point of pressure maxima. This implies that the reflected pressure
waves from the rigid boundary will have same phase as the incident wave, i.e., a compression pulse is
reflected as a compression pulse and a rarefaction pulse is reflected as a rarefaction pulse
On the other hand, reflection of sound wave from a low pressure region (like open end of an organ
pipe) is analogies to reflection of string wave from a free end. This point corresponds to a displacment
maxima, so that the incident & reflected displacement wave at this point must be in phase. This would
imply that this point would be a minima for pressure wave (i.e. pressure at this point remains at its
average value), and hence thereflected presure wave would be out of phase by  with respect to the
incident wave. i.e. a compression pulse is reflected as a rarefaction pulse and vice-versa.

9. WAVES IN A VIBRATING AIR COLUMN


Hollow pipes have long used for making musical sounds. A hollow pipe we call organ pipe. To understand
how these work, first we examine the behaviour of air in a hollow pipe that is open at both ends. If we
blow air across one end, the disturbance due to the moving air at that end propagates along the pipe
to the far end. When it reaches far end, a part of the wave is reflected, similar in the case when a
wave is reflected along a string whose end point is free to move. Since the air particles are free to
move at the open end, the end point is an antinode. if one end of the pipe is closed off, the the air is
not free to move any further in that direction and closed end becomes a node. now the resonant
behaviour of pipe is completely changed. Similar in the case of string, here also all harmonic frequencies
are possible and resonance may take place it the frequency of external source matches with any of
the one harmonic freuquency of pipe. Let us discuss in detail.
9.1 Vibration of Air in a Closed Organ Pipe
When a tuning fork is placed near the open end of a pipe. The air in the pipe oscillates with the same
frequency as that of tuning fork. Here the open end should be an antinode and closed and should be a
node for perfect reflection of waves from either end or for formation of stationary waves. Since one
end is a node and other is an antinode, the lowest frequency (largest wavelength) vibration has no
other nodes or antinodes between ends as shown in figure(a). This is the fundamental (minimum)
frequency at which stationary waves can be formed in a closed organ pipe. Thus if the wavelength is
 then we can see from figure (a), which shows the displacement wave of longitudinal waves in the
closed organ pipe.

Corporate Head Office : Motion Education Pvt. Ltd., 394 - Rajeev Gandhi Nagar, Kota-5 (Raj.)
SOUND WAVES Page # 155


l= ...(1)
4
or =4l

(a) (b) (c)


Thus fundamental frequency of oscillations of closed organ pipe of length l be given as
 
n1 = = ...(2)
 4l
Similarly first overtone of closed pipe vibrations is shown in figure (b) here wavelength  and pipe
length l are related as
3'
l ...(3)
4
4l
or ' 
3
Thus frequency of first overtone oscillatinos of a closed organ pipe of length l can be given as
 3
n2 = = ...(4)
' 4l
= 3n1
This is three times the fundamental frequency thus after fundamental only third harmonic frequency
exist for a closed organ pipe at which resonance can take place or stationary waves can be formed in
it.
Similarly next overtone, second overtone is shown in figure(c). Here the wavelength  and pipe length
l are related as
5"
l
4
4l
or " 
5
Thus the frequency of second overtone oscillation of a closed organ pipe of length l can be given as
 5
n3 = =  5n1
' ' 4l
This is fifth harmonic frequency of fundamental oscillations.
( 2n – 1)v
In general f =
4
Here frequency of oscillation is called (2n – 1)th harmonic and (n – 1)th overtone
From above analysis it is clear that the resonant frequecies of the closed organ pipe are only odd
harnomics of the fundamental frequency.

: 0744-2209671, 08003899588 | url : www.motioniitjee.com, : info@motioniitjee.com


Page # 156 SOUND WAVES

9.2 Vibration of Air in Open Organ Pipe


Figure shown the resonant oscillations of an open organ pipe. The least frequency at which an open
organ pipe resonates is the one with longest wavelength when at both the open ends of pipe antinodes
are formed and there is one node is between as shown in figure (a). In this situation the wavelengths
of sound in air  is related to length of organ pipe as

l=
2
or =2l ...(1)
Thus the fundamental frequency of organ pipe can be given as
v v
n1  
 2l

(a) (b) (c)


Similarly next higher frequency at which the open organ pipe resonate is shown in figure (b) which we
call first overtone. Here the wavelength ' is related to the length of pipe as
l =  ....(2)
Thus here resonant frequency for first overtone is given as
v v
n2   ...(3)
' l
Which is second harmonic of fundamental frequency. Similarly as shown in figure (c), in second overtone
oscillations, the wavelength '' of sound is releated to the length of pipe as
3 ''
l ...(4)
2
2l
or ' ' ...(5)
3
Thus be frequency of second overtone oscillations of an open organ pipe can given as
v 3v
n3   ...(6)
'' 2l
= 3n1 ...(7)
Which is third harmonic of fundamental frequency.
nv
In General f =
2
we can say frequency of oscillation is called nth harmonic and (n – 1)th overtone
The above analysis shown that resonant frequencies for formation of stationary waves includes all the
possible harmonic frequencies for an open organ pipe.

Corporate Head Office : Motion Education Pvt. Ltd., 394 - Rajeev Gandhi Nagar, Kota-5 (Raj.)
SOUND WAVES Page # 157

9.3 End correction


As mentioned earlier the displacment antinode at an open end of an organ pipe lies slightly outside the
open lend. The distance of the antinode from the open end is called end correction and its value is
given by

Np A0 Np

2r
P
e = 0.6 r
6r
e=0
where r = radius of the organ pipe.
with end correction, the fundamental frequency of a closed pipe (fe) and an open argon pipe (f0) will be
given by
v v
fe  and f0 
4(  0.6r ) 2(   12
. r)

9.4 Resonance Tube


This an apparatus used to determine velocity of sound in
air experimentally and also to compare frequncies of two
tuning forks.

shown figure the setup of a resonance the experiment.


There is a long tube T in which initially water is filled upto
the top and the eaer level can be change by moving a
reservoir R up and down.
A tunning fork of known frequency n0 is struck gently on a
rubber pad and brought near the open and tube T due to
which oscillations are transferred to the air coloumn in the
tube above water level. Now we gradually decrease the
water level in the tube. This air column behaves like a
closed organ pipe and the water level as closed end of
pipe. As soon as water level reaches a position where
there is a node of corresponding stationary wave, in air
column, resonance takes place and maximum sound
intensity is detected. Let at this position length of air column
be I1. If water level is further decreased, again maximum
sound intensity is observed when water leel is at another
node i.e. at a length l2 as shown in figure. Here if we find
two successive resonance lengths l1 and l2, we can get
the wavelength of the wave as


l2 – l1 =
2

or   2(l2 – l1 )
Thus sound velocity in air can be given as
v  n0   2n0 (l2 – l1 )

: 0744-2209671, 08003899588 | url : www.motioniitjee.com, : info@motioniitjee.com


Page # 158 SOUND WAVES

Ex.2 A tube of certain diameter and of length 48 cm is open at both ends. Its fundamental frequency
of resonance is found to be 320 Hz. The velocity of sound in air is 320 m/s. Estimate the diameter
of the tube. One end of the tube is now closed. Calculate the lowest frequency of resonance for
the tube.
Sol. The displacement curves of longitudinal waves in a tube open at both ends is shown in figure (a) and
(b).
48 cm

A B A
N
N

(a)
(b)
Let r be the radius of the tube. We know the antinodes occur slightly outside the tube at a distance
0.6 r from the tube end.
The distance between two antinodes is given by

 48  2  0.6 r
2
v 32000
We have = = = 100 cm
n 320
or 50 = 48 + 1.2r
2
or r= = 1.67 cm
12
.
Thus diameter of the tube is D = 2r = 3.33
When one end is closed, then

= 48 + 0.6r = 48 + 0.6 × 1.67 = 49
4
or  = 4 × 49 = 196 cm
v 32000
Now n=  = 163.3 Hz
 196
9.5 Quink's Tube
This is an apparatus used to demonstrate the phenomenon of interference and also used to measure
velocity of sound in air. This is made up of two U-tubes A and B as shown in figure. Here the tube B can
slide in and out from the tube A. There are two openings P and Q in the tube A. At opening P, a tuning
fork or a sound source of known frequency n0 is placed and at the other opening a detector is placed
to detect the resultant sound of interference occurred due to superposition of two sound waves
coming from the tubes A and B.

Initially tube B is adjusted so that detector detects a maximum. At this instant if length of paths
covered by the two waves from P to Q from the side of A and side of B are l1 are l2 respectively then
for constructive interference we must have
l2 – l2 = N ...(1)
If now tube B is further pulled out by a distance x so that next maximum is obtained and the length of
path from the side of B is l2' then we have

Corporate Head Office : Motion Education Pvt. Ltd., 394 - Rajeev Gandhi Nagar, Kota-5 (Raj.)
SOUND WAVES Page # 159

l2'  l2  2x ...(2)
Where x is the displacement of the tube. For next constructive interference of sound at point Q, we
have
l2' – l1  (N  1) ..(3)
From equation (1), (2) and (3), we get

or x= ...(4)
2
Thus by experiment we get the wavelength of sound as for two sucessive points of constructive
interference, the path difference must be . As the tube B is pulled out by x, this introduces a path
difference 2x in the path of sound wave through tube B. If the frequency of the source is known, n0,
the velocity of sound in the air filled in tube can be given as
v = n0 = 2n0x ...(5)
Ex.3 In a large room a person recieves direct sound waves from a source 120 m away from him. he
also receives waves from the same source which reach him, being reflected from the 25m high
ceiling at a point halfway between them. For which wavelength will these two sound waves
interfere constructivity?
Sol. As shown in figure for reflection from the ceilling
Path SCP = SC + CP = 2SC [As i = r, SC = CP]
C
or Path SCP = 2 60 2  25 2 = 130 cm
So path differene between interfering waves
along path SCP, and SP, i r
x = 130 –120 = 10 m
Now for consructive interfernece at P.
x = n. i.e., 10 = n
10 S
or = with n = 1, 2, 3 ......... *
Source *P
n Detector
i.e.  = 10 m, 5m, (10/3) m and so on
Ex.4 Figure shown a tube structure in which a sound signal is sent from one end and is received at
the othe end. The semicircular part has a radius of 20.0 cm. The frequency of the sound source
can be varied electronically between 1000 and 4000 Hz. Find the frequencies at which maxima
of intensity are detected. The speed of sound in air = 340 m/s.

Sol. The sound wave reaches detector by two paths simultaneously be straight as well as semicircular
track. The wave through the straight path travels a distance l1 = 2 × 20 cm and the wave through the
curved part travles a distane l2 =  (20 cm) = 62.8 cm before they meet again and travel to the
receiver. The path difference between the two waves received is, therefore,
 l = l2 – l2 = 62.8 cm – 40 cm = 22.8 cm = 0.228 m
v 340
The wavelength of either wave is = . For constructive interfernece, l = N, where N is an
n n
integer.
 340 
or, 0.228  N 
 n 

 340 
or, n = N 0.228  = N(1491.2) Hz = N(1490) Hz

Thus, the frequencies within the specific range which cause maxima of intensity are 1490 Hz and 2980 Hz.

: 0744-2209671, 08003899588 | url : www.motioniitjee.com, : info@motioniitjee.com


Page # 160 SOUND WAVES

Ex.5 Two sources S1 and S2, separated by 2.0 m, vibrate according to equation y1 = 0.03 sin t and
y2 = 0.02 sin  t where y1, y2 and t are in M.K.S unit. They send out waves of velocity 1.5m/s.
Calculate the amplitude of the resultant motion of the particle co-linear with S1 and S2 and
located at a point (a) to the right of S2(b) to the left of S2 and (C) in the middle of S1 and S2.
Sol. The situation of shown in figure

1m 1m

P1 *S 1
Q *S2
P2
2m

The oscillations y1 and y2 have amplitudes A1 = 0.03 m and A2 = 0.02 respectively.

 1
The frequency of both sources in n = = = 0.5 Hz
2 2

v 15
.
Now wavelength of each wave  = = = 3.0 m
n 0.5
(a) The path difference for all points P2 to the right of S2 is
 = (S1P2 – S2P2) = S1S2 = 2m
2
Phase difference    Path difference

2 4
  2.0 
3 3
The resultant amplitude for this point is given by

R  A12  A 22  2A1 A 2 cos   (0.03)2  (0.02) 2  2  0.03  0.02  cos(4  / 3)


Solving we get R = 0.0265 m
(b) The path differnece for all point P, to the left of S1
 = (S2P – S1P) = S1S2 = 2.0 m
Hence the resultant amplitude for all points to the left of S1 is also 0.0265 m
(c) For a point Q, between S1 and S2, the path difference is zero i.e.,  = 0. Hence constructive interference
take place at Q, thus amplitude at this point is maximum and given as

R  A 12  A 22  2A1A 2 = A1 + A2 = 0.03 + 0.02 = 0.05 m

Ex.6 Two point sources of sound are placed at a distance d and a detector moves on a straight line
parallel to the line joining the sources as shown in figure. at a distane D away from sources.
Initially Detector is situated on the line so that it is equidistant from both the sources. Find the
displacement of detertor when it detects nth maximum sound and also find its displacement
when it detects nth minimum sound.

S1 *

d Detector
(D)
S2 (D>>d)
*

Corporate Head Office : Motion Education Pvt. Ltd., 394 - Rajeev Gandhi Nagar, Kota-5 (Raj.)
SOUND WAVES Page # 161

Sol. The situation is shown in figure


Let us consider the situation when detector move by a distance x
as shown. Let at this position the path diference between the D
waves
S1 * x
from S1 and S2 to detector by  then we have 

 = S2D – S1D ~
– S2Q [Where S1Q is perpendicular on line S2D] d O
D
Here if  is small angle as D >> d, we have S2 Q
*
x
S2Q = d sin  ~– d tan = d
D
Thus at the position of detector, path deference is
dx
= ....(1)
D
The expression for path difference in equation (1) is an important formula for such problems. Students
are advised to keep this formula in mind for future use.
When detector was at point O, path difference was zero and it detects a maxima, now if detector
detects nth maximum then its path difference at a distance x from O can be given as
=n
dx nD
or =n or x =
D d
Similarly if detector detects nth minima then the path difference between two waves at detector can
be given as
 dx  ( 2n  1)D
 = (2n + 1) or  (2n  1) or x=
2 D 2 2d
Ex.7 Two coherent narrow slits emitting of wavelength  in the same phase are placed parallel to
each other at a small separation of 2. the sound is detected by moving a detector on the screen
S at a distance D (>> ) from the slit S1 as shown in figure. Find the distance x such that the
intensity at P is equal to the intensity at O.

*P
x
2
* * O
S1 S2

D S
Sol. When detector is at O, we can see that the path differnece in the two waves reaching O is d = 2 thus
at O detector receives a maximum sound. When it reaches P and again there is a maximum sound
detected at P the path difference between two waves must  = . Thus shown figure the path
difference at P can be given as
 = S1P – S2P ~
– S1Q
*P
= d cos  = 2 cos 
Q x
And we have at point P, path difference  = , Thus 
* * O
 = 2 cos  =  S1 d  2 S2

1
or, cos  = D
2

or, =
3
 
Thus the value of x can be written as x = D tan  = D tan  3  = 3D

: 0744-2209671, 08003899588 | url : www.motioniitjee.com, : info@motioniitjee.com


Page # 162 SOUND WAVES

Ex.8 Figure shown two coherent sources S1 and S2 which emit sound
of wavelength  in phase. The separation between the sources
is 3. A ciruclar wire of large radius is placed in such a way
that S1S2 lies in its plane and the middle point of S1S2 is at the
centre of the wire. Find the angular position  on the 
wire for which constructive interference takes place. S1 S2

B
P


C A
S1 S2
Sol.
d  3

From previous question, we can say that for a point P on the circle shown in fiure shown. The path
difference in the two waves at P is
 = S1P – S2P = d cos  = 3  cos 
We know for constructive interference at P. The path difference must be an integral multiple of
wavelength . Thus for a maxima at P, we have
3  cos  = 0 ; 3  cos  =  ; 3  cos  = 2 ; 3  cos  = 3 ;
 –1 1 2
or, = or  = cos or  = cos–1 or =0
2 3 3

There are four points A, B, C and D on circle at which  = 0 or and there are two points in each
2
–11 –1 2
quadrant at   cos and  = cos at which constructive interference takes place. Thus there are
3 3
total twelve points on circle at which maxima occurs.

9.6 Vibrations of Clamped Rod


We have discussed the resonant vibrations of a string clamped at two ends. Now we discuss the
ocillations of a rod clamped at a point on its length as shown in figure. Figure shows a rod AB clamped
at its middle point. If we gently hit the rod at its one end, it begin to oscillate and in the natural
oscillations the rod vibrates at its lowest frequency and maximum wavelength, which we call fundamental
mode of oscillations. With maximum wavelength when transverse stationary waves setup in the rod,
the free ends vibrates as antinodes and the clamped end a node as shown in figure. Here if  be the
wavelength of the wave, we have

l= or  =2l
2
C

A B

Corporate Head Office : Motion Education Pvt. Ltd., 394 - Rajeev Gandhi Nagar, Kota-5 (Raj.)
SOUND WAVES Page # 163

Thus the frequency of fundamental oscillations of a rod damped at mid point can be given as

v I Y
n0   ...(1)
 2l 
Where Y is the Young's modulus of the material of rod and  is the density of the material of rod.
Next higher frequency at which rod vibrates will be then one when wave length is decreased to a value
so that one node is inserted between mid point and an end of rod as shown in figure
C
A B

l
In this case if  be the wavelength of the waves in rod, we have
3
l=
2
2l
or  ...(2)
3
Thus in this case the oscillation frequency of rod can be given as

v 3 Y
n1    3n0 ...(3)
 2l 
This is called first overtone frequency of the damped rod or thid harmonic frequency. Similarly, the
next higher freqnecy of oscillation i.e. second overtone of the oscillating rod can be shown in figure
shown. Here is  be the wavelength of the wave then it can be given as
5 2l
l or  ...(4)
2 5
C
A B

l
Thus the frequency of oscillation of rod can be given as

v 5 Y
n2    5n0 ..(5)
 2l 
Thus the second overtone frequency is the fifth harmonic of the fundamental oscillation frequency of
rod. We can also see from the above analysis that the resonant freqencies at which stationary waves
are setup in a damped rod are only odd harmonics of fundamental frequency.
Thus when an external source of frequency matching with any of the harmonic of the damped rod then
stationary waves are setup in the rod.
9.7 Natural Oscillation of Organ Pipes
When we initiate some oscillations in an organ pipe, which harmonics are excited in the pipe depends
on how initial disturbance is produced in it. For example, if you gently blow across the top of an organ
pipe it resonates softly at its fundamental frequency. But if you blow must harder you hear the higher
pitch of an overtone because the faster airsteam higher frequencies in the exciting disturbance. This
sound effect can also be achieved by increasing the air pressure to an organ pipe.
9.8 Kundt's Tube
This is an apparatus used to find velocity of sound in a gaseous medium or in different materials. It
consists of a glass tube as shown in figure. one end of which a piston B is fitted which is attached to
a wooden handle H and can be moved inside and outside the tube and fixed, the rod M of the required
material is fixed at clamp C in which the velocity of sound is required, at one end of rod a disc A is fixed
as shown.

: 0744-2209671, 08003899588 | url : www.motioniitjee.com, : info@motioniitjee.com


Page # 164 SOUND WAVES

In tube air is filled at room temperature and a thin layer of lycopodium powder is put along the length
of the tube. It is a very fine powder particles of which can be displaced by the air particles also.
When rod M is gently rubbed with a resin cloth or hit gently, it starts oscillating in fundamental mode as
shown in figure, frequency of which can be given as
v 1 Y 
nrod   [As 0 = ]
 2l0  2

10. BEATS
When two sources of sound that have almost the same frequency are sounded together, an interesting
phenomenon occurs. A sound with a frequency average of the two is heard and the loudness of sound
repeatedly grows and then decays, rather than being constant. Such a repeated variation in amplitude
of sound are called 'beats".
If the frequency of one of the source is changed, there is a corresponding change in the rate at which
the amplitude varies. This rate is called beat fequency. As the frequencies come close together, the
beat frequency becomes slower. A musician can tune a guitar to another source by listening for the
beats while increasing or decreasing the tension in each string, eventually the beat frequency becomes
very low so that effectively no beats are heard, and the two sources are then in tune.
We can also expalin the phenomenon of beat mathematically. Let us consider the two superposing
waves have frequencies n1 and n2 then their respective equations of oscillation are
y1 = A sin 2n1t ...(1)
and y2 = A sin 2n2t ...(2)
On superposition at a point, the displacement of the medium particle is given as
y = y1 + y2
y = A sin 2n1t + A sin 2n2t

 n – n2   n1  n 2 
y  2A cos 2 1  t sin 2 t ...(3)
 2   2 

 n  n2 
y  R sin 2  1 t ...(4)
 2 
There equation (4) gives the displacement of medium particle where susperposition takes place, it
n1  n 2
shows that the particle executes SHM with frequency , average of the two superposing
2
frequencies and with amplitude R which varies with time, given as

Corporate Head Office : Motion Education Pvt. Ltd., 394 - Rajeev Gandhi Nagar, Kota-5 (Raj.)
SOUND WAVES Page # 165

 n – n2 
R  2A cos 2  1 t ...(5)
 2 
Here R becomes maximum when
 n – n2 
cos 2 1  t  1
 2 

 n – n2 
or 2  1  t  N [N  I]
 2 

N
or t
n1 – n2
1 2
or at time t  0, , ,.........
n1 – n2 n1 – n 2
At all the above time instants the sound of maximum loudness is heard, similarly we can find the time
instans when the loudness of sound is minimum, it occurs when

 n – n2 
cos 2 1 t0
 2 

 n – n2  
or 2  1  t  (2N  1) [N  I]
 2  2

2N  1
or t
2(n1  n2 )

1 3
or at time instants t , ,.......
2(n1 – n 2 ) 2(n1 – n 2 )

Here we can see that these time instants are exactly lying in the middle of the instans when loudest
sound is heard. Thus on superposition of the above two frequencies at a medium particle, the sound
will be increasing, decreasing, again increasing and decreasing and so on. This effect is called beats.
Here the time between two successive maximum or minimum sounds is called beat period, which is
given as
Beat Period TB = time between two successive maxima = time between two successive minima

1

n1 – n 2

Thus beat frequency or number of beats heard per second can be given as

1
fB   n1 – n 2
TB

The superposition of two waves of slightly different frequencies is graphically shown in figure. The
resulting envelope of the wave formed after superposition is also shown in figure (b). Such a wave
when propagates, produces "beat" effect at the medium particles.

: 0744-2209671, 08003899588 | url : www.motioniitjee.com, : info@motioniitjee.com


Page # 166 SOUND WAVES

10.1 Echo
The repetition of sound produced due to reflection by a distant extended surface like a different, hill
well, building etc. is called an echo. The effect of sound on human ear remains for approximately one
tenth of a second. If the sound is reflected back in a time less then 1/10 of a second, no echo is heard.
Hence human ears are not able to distinguish a beat frequency of 10 Hz or more than 10 Hz.

11. DOPPLER'S EFFECT


When a car at rest on a road sounds its high frequency horn and you are also standing on the road
near by, you'll hear the sound of same frequency it is sounding but when the car approaches you with
its horn sounding, the pich (frequency) of its sound seems to drop as the car passes. This phenomenon
was first described by an Austrain Scientist Christien Doppler, is called the Doppler effect, He explained
that when a source of sound and a listener are in motion relative to each other, the frequency of the
sound heard by the listener is not the same as the source frequency. Lets discuss the Doppler effect in
detail for different cases.

11.1 Stationary Source and Stationary Observer


Figure shown a stationary sources of frequency n0 which produces sound waves in air of wavelength 0
given as
v
0  [v = speed of sound in air]
n0

v
0 
n0
v

*
S *
Observer
Source
(n0 Hz)

Corporate Head Office : Motion Education Pvt. Ltd., 394 - Rajeev Gandhi Nagar, Kota-5 (Raj.)
SOUND WAVES Page # 167

Although sound waves are longitudinal, here we represent sound weaves by the transverse displacement
curve as shown in figure to understand the concept in a better way. As source produces waves, these
waves travel towards, stationary observer O in the medium (air) with speed v and wavelength 0. As
observer is at rest here it will observe the same wavelength 0 is approaching it with speed v so it will
listen the frequency n given as
v
n
 n0 [same as that of source] ...(1)
0
This is why when a stationary observer listends the sound from a stationary source of sound, it
detects the same frequency sound which the source is producting. Thus no Doppler effect takes place
if there is no relative motion between source and observer.
11.2 Stationary Source and Moving Observer
Figure shown the case when a stationary sources of frequency n0 produces sound waves which have
wavelength in air given as
v
0 
n0
v
0 
n0
v
v0

* O
S Observer
Source
(n0 Hz)
These waves travel toward moving observer with velocity v0 towards, the source. When sound waves
approach observer, it will receive the waves of wavelength 0 with speed v + v0 (relative speed). Thus
the frequency of sound heard by observer can be given as
v  v0
Apparent frequency nap = 0

v  v0  v  v0 
  n0  
 v  v 
  ...(2)
 n0 
Similarly we can say that if the obsever is receding away from the source the apparent frequency
heard by the observer will be given as

 v – v0 
n ap  n0   ...(3)
 v 
11.3 Moving Source and Stationary Observer
Figure shows the situation when a moving source S of frequency n0 produces sound waves in medium
(air) and the waves travel toward observer with velocity v.

0
 ap v

A
S
*
S' *O
vs Observer
Source
(n0 Hz)

 1
vs  
 n0 

: 0744-2209671, 08003899588 | url : www.motioniitjee.com, : info@motioniitjee.com


Page # 168 SOUND WAVES

Here if we carefully look at the initial situation when source starts moving with velocity vs as well as it
 1
starts producting waves. The period of one oscillation is  n  sec and in this duration source emits
 0
one wavelength 0 in the direction of propagation of waves with speed v, but in this duratin the source
 1
will also move forward by a distance vs  n  . Thus the effective wavelength of emitted sound in air is
 0
slightly compressed by this distance as shown in figure. This is termed as apparent wavelength of
sound in medium (air) by the moving source. This is given as

 1
Apparent wavelength  ap   0 – v s  n  ...(1)
 0

 0n 0 – v s v – v s
 
n0 n0
Now this wavelength will approach observer with speed v ( O is at rest). Thus the frequency of sound
heard by observer can be given as
v
Apparent frequecy n ap 
 ap

v  v 
= ( v – v ) / n = n0  v – v  ...(2)
s 0  s

Similarly if source is receding away from observer, the apparent wavelength emitted by source in air
toward observer will be slightly expanded and the apparent frequency heard by the stationary observer
can be given as
 v 
nap  n0   ...(3)
 v  vs 
11.4 Moving Source and Moving Observer
Let us consider the situation when both source and observer are moving in same direction as shown in
figure at speeds vs and v0 respectively.
0
v
vs

S* *O

In this case the apparent wavelength emitted by the source behind it is given as
v  vs
 ap 
n0
Now this wavelength will approach the observer at relative speed v + v0 thus the apparent frequency
of sound heard by the observer is given as

v  v0  v  v0 
n ap  = n0   ...(1)
 ap  v  vs 
By looking at the expression of apparent frequency given by equation, we can easily develop a general
relation for finding the apparent frequency heard by a moving observer due to a moving source as

v  v0 
nap  n0   ...(2)
 v  vs 

Corporate Head Office : Motion Education Pvt. Ltd., 394 - Rajeev Gandhi Nagar, Kota-5 (Raj.)
SOUND WAVES Page # 169

Here + and – signs are chosen according to the direction of motion of source and observer. The sign
covention related to the motion direction can be stated as :
(i) For both source and observer v0 and vs are taken in equation with –ve sign if they are moving in the

direction of v i.e. the direction of propagation of sound from source to observer..
(ii) For both source and observer v0 and vs are taken in equation (2) with +ve sign if they are moving in

the direction opposite to v i.e. opposite to the direction of propagation of sound from source to
observer.

11.5 Doppler Effect in Reflected Sound


When a car is moving toward a stationary wall as shown in figure. If the car sounds a horn, wave
travels towards the wall and is reflected from the wall. When the reflected wave is heard by the driver,
it appears to be of relatively high pitch. If we wish to measure the frequency of reflected sound then
the problem must be handled in two steps.

reflected sound

vc

First we treat the stationary wall as stationary observer and car as a moving source of sound of
frequency n0. In this case the frequency received by the wall is given as

 v 
n1  n 0   ...(1)
 v – vc 
Now wall reflects this frequency and behaves like a stationary source of sound of frequency n1 and car
(driver) behave like a moving observer with velocity vc. Here the apparent frequency heard by the car
driver can be given as

 v  vc 
n ap  n1 
 v 

 v   v  vc   v  vc 
= n 0  v – v    v  = n 0  v – v  (2)
 c   c

Same problem can also be solved in a different manner by using method of sound images. In this
procedure we assume the image of the sound source behind the reflector. In previous example we can
explain this by situation shown in figure.

ve
vc

Here we assume that the sound which is reflected by the stationary wall is coming from the image of
car which is at the back of it and coming toward it with velocity vc. Now the frequency of sound heard
by car driver can directly be given as

: 0744-2209671, 08003899588 | url : www.motioniitjee.com, : info@motioniitjee.com


Page # 170 SOUND WAVES

 v  vc 
nap = n0  v – v  ...(3)
 c

This method of images for solving problems of Doppler effect is very convenient but is used only for
velocities of source and observer which are very small compared to the speed of sound and it should
not be used frequenctly when the reflector of sound is moving.

11.6 Doppler's Effect for Accelerated Motion


For the case of a moving source and a moving observer, we known the apparent frequency observer
can be given as

v  v0 
nap  n0   ...(4)
 v  vs 
Here v is the velocity of sound and v0 and vs are the velocity of observer and source respectively.
When a source of observer has accelerated or ratarded motion then in equation (4) we use that value
of v0 at which observer receies the sound and for source, we use that value of vs at which it has
emitted the wave.
The alternative method of solving this case is by the traditional method of compressing or expending
wavelength of sound by motion of source and using relative velocity of sound with respect to observer
11.7 Doppler's Effect when Source and Observer are not in Same Line of Motion
Consider the situation shown in figure. Two cars 1 and 2 are moving along perpendicular roads at speed
v1 and v2. When car - 1 sound a horn of frequency n0, it emits sound in all directions and say car - 2 is
at the position, shown in figure. when it receives the sound. In such cases we use velocity components
of the cars along the line joining the source and observer thus the apparent frequency of sound heard
by car-2 can be given as

 v  v 2 cos  2 
n ap  n0   ...(6.266)
 v – v1 cos 1 

car-1

1 v1

v2

Car-2

2

Corporate Head Office : Motion Education Pvt. Ltd., 394 - Rajeev Gandhi Nagar, Kota-5 (Raj.)
SOUND WAVES Page # 171

Exercise - I OBJECTIVE PROBLEMS (JEE MAIN)

1. The elevation of a cloud is 60°above the horizon. A Sol.


thunder is heard 8 s after the observation of lighting.
The speed of sound is 330 ms–1. The vertical height of
cloud from ground is

Cloud

4. The energy per unit area associated with a pro-


gressive sound wave will be doubled if :
(A) the amplitude of the wave is doubled
60° (B) the amplitude of the wave is increased by 50%
Horizon (C) the amplitude of the wave is increased by 41%
(A) 2826 m (B) 2682 m (D) None of these
(C) 2286 m (D) 2068 m Sol.
Sol.

2. The ratio of speed of sound in neon to that in


water vapours at any temperature (when molecular 5. A sound level I is greater by 3.0103 dB from an-
weight of neon is 2.02 × 10–2 kg mol–1 and for water other sound of intensity 10 nW cm–2. The absolute
vapours is 1.8 × 10–2 kg mol–1) value of intensity of sound level I in Wm–2 is :
(A) 1.06 (B) 1.60 (C) 6.10 (D) 15.2 (A) 2.5 × 10–4 (B) 2 × 10–4
Sol. (C) 2.0 × 10–2 (D) 2.5 × 10–2
Sol.

3. A Firecracker exploding on the surface of a lake is


heard as two sounds a time interval t apart by a man
on a boat close to water surface. Sound travels with 6. A wave travels uniformly in all directions from a
a speed u in water and a sped v in air. The distance point source in an isotropic medium. The displacement
from the exploding firecracker to the boat is of the medium at any point at a distancer r from the
uvt t (u  v ) source may be represented by (A is a constant
(A) (B)
uv uv representing strength of source)
t (u  v) uvt
(C) (D) (A) [ A / r ] sin (kr – t) (B) [A/r] sin (kr – t)
uv uv
(C) [Ar] sin (kr – t) (D) [A/r2] sin (kr – t)

: 0744-2209671, 08003899588 | url : www.motioniitjee.com, : info@motioniitjee.com


Page # 172 SOUND WAVES

Sol. Sol.

7. How many times more intense is 90 dB sound than


40 dB sound?
(A) 5 (B) 50 (C) 500 (D) 105
10. S1 and S2 are two sources of sound emitting sine
Sol.
waves. The two sources are in phase. The sound
emmited by the two sources interfere at point F. The
waves of wavelength :

2m 4m
S1 S2 F

8. When two waves with same frequency and con- (A) 1 m will result in constructive interference
stant phase difference interfere,
2
(A) there is a gain of energy (B) m will result in constructive interference
3
(B) there is a loss of energy
(C) 4m will result in destructive interference
(C) the energy is redistributed and the distribution
(D) All the above
changes with time
Sol.
(D) the energy is redistributed and the distribution
remains constant in time
Sol.

11. The ratio of intensities between two coherent


sound sources is 4:1 The difference of loudness in dB
between maximum and minimum intensities when they
interfere in space is
(A) 10log 2 (B) 20 log 3
9. Sound waves from a tuning fork F reach a point P
(C) 10 log 3 (D) 20 log 2
by two separate routes FAP and FBP (when FBP is
Sol.
greater than FAP by 12 cm there is silence at P). If
the difference is 24 cm the sound becomes maximum
at P but at 36 cm there is silence again and so on. If
velocity of sound in air is 330 ms–1, the least fre-
quency of tuning fork is :
(A) 1537 Hz (B) 1735 Hz
(C) 1400 Hz (D) 1375 Hz

Corporate Head Office : Motion Education Pvt. Ltd., 394 - Rajeev Gandhi Nagar, Kota-5 (Raj.)
SOUND WAVES Page # 173

12. In Quincke’s tube a detector detects minimum Sol.


intensity. Now one of the tube is displaced by 5 cm.
During displacement detector detects maximum
intensity 10 times, then finally a minimum intensity
(when displacement is complete). The wavelength of
sound is:
(A) 10/9 cm (B) 1 cm
(C) 1/2 cm (D) 5/9 cm
Sol.

15. Sound waves of frequency 660 Hz fall normally on


a perfectly reflecting wall. The shortest distance from
the wall at which the air particle has maximum amplitude
of vibration is (velocity of sound in air is 330 m/s)
(A) 0.125 m (B) 0.5 m
(C) 0.25 m (D) 2 m
13. Two waves of sound having intensities I and 4I Sol.
interfere to produce interference pattern. The phase

difference between the waves is at point A and  at
2
point B. Then the difference between the resultant
intensities at A and B is
(A) 2I (B) 4I (C) 5I (D) 7I
Sol.

16. At the closed end of an organ pipe :


(A) the displacement is zero
(B) the displacement is maximum
(C) the wave pressure is zero
(D) None of these
Sol.

14. A cylindrical tube, open at one end and closed at


the other, is in acoustic unison with an external source
of frequency held at the open end of the tube, in its
fundamental note. Then
(A) the displacement wave from the source gets re- 17. An open organ pipe of length L vibrates in its
flected with a phase change of  at the closed end fundamental mode. The pressure variation is maximum
(B) the pressure wave from the source get reflected with- (A) at the two ends
out a phase change at the closed end (B) at the middle of the pipe
(C) the wave reflected from the closed end again (C) at distance L/4 inside the ends
gets reflected at the open end (D) at distance L/8 inside the ends
(D) All the above

: 0744-2209671, 08003899588 | url : www.motioniitjee.com, : info@motioniitjee.com


Page # 174 SOUND WAVES

Sol. Sol.

18. The effect of making a hole exactly at (1/3rd) of


the length of the pipe from its closed end is such 21. A sufficiently long close organ pipe has a small
that: hole at its bottom. Initially the pipe is empty. Water is
(A) its fundamental frequency is an octave higher than poured into the pipe at a constant rate. The
the open pipe of same length fundamental frequency of the air column in the pipe
(B) its fundamental frequency is thrice that before (A) Continuously increasing
making a hole (B) First increases and them becomes constant
(C) the fundamental alone is changed while the har- (C) Continuously decreases
monics expressed as ratio of fundamentals remain the (D) First decreases and them become constant
same Sol.
(D) All the above
Sol.

22. A tuning fork of frequency 340 Hz is vibrated just


above a cylindrical tube of length 120 cm. Water is
19. An open organ pipe of length L vibrates in second slowly poured in the tube. If the speed of sound is
harmonic mode. The pressure vibration is maximum 340 ms–1 then the minimum height of water required
(A) At the two ends for resonance is:
(B) at a distance L/4 from either end inside the tube (A) 95 cm (B) 75 cm (C) 45 cm (D) 25 cm
(C) At the mid-point of the tube Sol.
(D) None of these
Sol.

20. An open organ pipe of length l is sounded together


with another organ pipe of length l + x in their 23. An organ pipe P1 closed at one end vibrating in its
fundamental tones (x << l). The beat frequency heard first overtone. Another pipe P2 open at both ends is
vibrating in its third overtone. They are in a resonance
will be (speed of sound is v) :
with a given tuning fork. The ratio of the length of P1
vx vl 2 vx vx 2
(A) (B) (C) (D) to that of P2 is:
4l 2 2x 2l 2 2l (A) 8/3 (B) 3/8 (C) 1/2 (D) 1/3

Corporate Head Office : Motion Education Pvt. Ltd., 394 - Rajeev Gandhi Nagar, Kota-5 (Raj.)
SOUND WAVES Page # 175

Sol. Sol.

27. The number of beats heard per second if there


are three sources of frequencies (n – 1), n and (n+ 1)
24. A pipe’s lower end is immersed in water such that of equal intensities sounded together is :
the length of air column from the top open end has a (A) 2 (B) 1 (C) 4 (D) 3
certain length 25 cm. The speed of sound in air is 350
Sol.
m/s. The air column is found to resonate with a tuning
fork of frequency 1750 Hz. By what minimum distance
should the pipe be raised in order to make the air
column resonate again with the same tuning fork
(A) 7 cm (B) 5 cm (C) 35 cm (D) 10 cm
Sol.

28. A tuning fork of frequency 280 Hz produces 10


beats per sec when sounded with a vibrating sonometer
string. When the tension in the string increases slightly,
it produces 11 beats per sec. The original frequency of
the vibrating sonometer string is:
(A) 269 Hz (B) 291 Hz
(C) 270 Hz (D) 290 Hz
25. In case of closed organ pipe which harmonic the Sol.
pth overtone will be
(A) 2p + 1 (B) 2p – 1
(C) P + 1 (D) p – 1
Sol.

29. The speed of sound in a gas, in which two waves


of wavelength 1.0 m and 1.02 m produce 6 beats per
second, is approximately :
(A) 350 m/s (B) 300 m/s
(C) 380 m/s (D) 410 m/s
Sol.
26. A closed organ pipe has length ‘l’. The air in it is
vibrating in 3rd overtone with maximum displacement
amplitude ‘a’. The displacement amplitude at distance
l / 7 from closed end of the pipe is :
(A) 0 (B) a
(C) a / 2 (D) none of these

: 0744-2209671, 08003899588 | url : www.motioniitjee.com, : info@motioniitjee.com


Page # 176 SOUND WAVES

30. Consider two sound sources S1 and S2 having same 33. An observer starts moving with uniform acceleration
frequency 100Hz and the observer O located between ‘a’ towards a stationary sound source of frequency f.
them as shown in the fig. All the three are moving with As the observer approaches the source, the apparent
same velocity in same direction. The beat frequency of frequency f’ heard by the observer varies with time t
the observer is as:
f f
–1 –1
S1 30ms O 30ms S2 30ms–1
(A) 50 Hz (B) 5 Hz (C) zero (D) 2.5 Hz (A) (B)
Sol. t t
f
f'
(C) (D)
t
t
Sol.

31. A source S of frequency f0 and an observer O,


moving with speeds v1 and v2 respectively, are moving
away from each other. When they are separated by
distance a (t = 0), a pulse is emitted by the source.
This pulse is received by O at time t1 then t1, is equal 34. A source of sound S having frequency f. Wind is
to
blowing from source to observer O with velocity u. If
a a
(A) v  v (B) v  v speed of sound with respect to air is C, the wavelength
s 2 1 s of sound detected by O is :
a a
(C) v  v (D) v  v  v Cu C–u
s 2 1 2 s (A) (B)
Sol. f f
C(C  u) C
(C) (D)
( C – u)f f
Sol.

32. A detector is released from rest over a source of


sound of frequency f0 = 103 Hz. The frequency observed
by the detector at time t is f(Hz)

plotted in the graph. The speed 2000


of sound in air is (g = 10 m/s )2
1000
(A) 330 m/s (B) 350 m/s
(C) 300 m/s (D) 310 m/s 30 t(s)

Sol.

Corporate Head Office : Motion Education Pvt. Ltd., 394 - Rajeev Gandhi Nagar, Kota-5 (Raj.)
SOUND WAVES Page # 177

Exercise - II
SINGLE CORRECT Sol.
1. In a test of subsonic Jet flies over head at an
altitude of 100m. The sound intensity on the ground
as the Jet passes overhead is 160 dB. At what altitude
should the plane fly so that the ground noise is not
greater than 120 dB.
(A) above 10km from ground
(B) above 1 km from ground
(C) above 5 km from ground 4. The ratio of maximum to minimum intensity due to
(D) above 8 km from ground 49
Sol. superposition of two waves is . Then the ratio of
9
the intensity of component waves is
25 16 4 9
(A) (B) (C) (D)
4 25 49 49
Sol.

2. Three coherent waves of equal frequencies having


amplitude 10 m, 4m, and 7m respectively, arrive at a

given point with successive phase difference of  / 2.


the amplitude of the resulting wave in m is given by 5. The displacement sound wave in a medium is given
(A) 5 (B) 6 (C) 3 (D) 4 by the equation Y = A cos (ax + bt) where A, a and b
Sol. are positive constants. The wave is reflected by an
obstacle situated at x = 0. The intensity of the re-
flected wave is 0.64 times that of the incident wave.
Tick the statement among the following that is incor-
rect.
(A) the wavelength and frequency of the wave are
2/a and b/2 respectively
(B) the amplitude of the reflected wave is 0.8 A
(C) the resultant wave formed after reflection is y = A
cos (ax + bt) + [–0.8 A cos (ax – bt)] and Vmax (maxi-
mum particle speed) is 1.8 bA
(D) the equation of the standing wave so formed is y
3. A person standing at a distance of 6 m from a = 1.6 A sin ax cos bt
source of sound receives sound wave In two ways, Sol.
one directly from the source and other after reflection
S
from a rigid boundary as shown in 5m
figure. The maximum wavelength for
6m
which, the person Will receive
4m
maximum sound intensity, is
P
16 8
(A) 4 m (B) m (C) 2 m (D) m
3 3

: 0744-2209671, 08003899588 | url : www.motioniitjee.com, : info@motioniitjee.com


Page # 178 SOUND WAVES

6. A tube of diameter d and of length  unit is open at Sol.


both ends. Its fundamental frequency of resonance is
found to be 1. The velocity of sound in air is 330 m/
sec. One end of tube is now closed. The lowest
frequency of resonance of tube is 2. Taking into
2
consideration the end correction,  is -
1
d

9. First overtone frequency of a closed organ pipe is


equal to the first overtone frequency of an open organ
pipe. Further nth harmonic of closed organ pipe is also
(   0.6d) 1 (   0.3d) equal to the mth harmonic of open pipe, where n and
(A) (B) m are :
(   0.3d) 2 (   0.6d)
(A) 5, 4 (B) 7, 5 (C) 9, 6 (D) 7, 3.
1 (   0.6d) 1 ( d  0.3 ) Sol.
(C) (D)
2 (   0.3d) 2 ( d  0.6 )
Sol.

10. If I1 and I2 are the lengths of air column for the


first and second resonance when a tuning fork of
7. In a closed end pipe of length 105 cm, standing frequency n is sounded on a resonance tube, then
waves are set up corresponding to the third overtone. the distance of the displacement antinode from the
What distance from the closed end, amongst the top end of the resonance tube is :
following, is a pressure Node? 1
(A) 2(l2 – l1) (B) (2l1  l2 )
(A) 20 cm (B) 60 cm (C) 85 cm (D) 45 cm 2
Sol. l 2  3l1 l2  l1
(C) (D)
2 2
Sol.

8. A closed organ pipe of radius r1 and an open organ


pipe of radius r2 and having same length L resonate
when excited with a given tunning fork. Closed organ
pipe resonates in its fundamental mode where as open 11. The first resonance length of a resonance tube is
organ pipe resonates in its first overtone, then 40cm and the second resonance length is 122 cm.
(A) r2 – r1 = L (B) r2 – r1 = L/2 The third resonance length of the tube will be
(C) r2 – 2r1 = 2.5 L (D) 2r2 – r1 = 2.5 L (A) 200 cm (B) 202 cm
(C) 203 cm (D) 204 cm

Corporate Head Office : Motion Education Pvt. Ltd., 394 - Rajeev Gandhi Nagar, Kota-5 (Raj.)
SOUND WAVES Page # 179

Sol. Sol.

12. The tuning forks A & B produce notes of frequen-


cies 256 Hz & 262 Hz respectively. An unknown note
sounded at the sametime as A produces beats. When
the same note is sounded with B, beat frequency is
twice as large. The unknown frequency could be :
(A) 268 Hz (B) 250 Hz
(C) 260 Hz (D) none of these
Sol.

15. Source and observer both start moving simulta-


neously from origin, one along X-axis and the other
along Y-axis with speed of source equal to twice the
speed of observer. The graph between the apparent
frequency (n) observed by observer and time t would
be : (n is the frequency of the source)
n' n'

13. A closed organ pipe and an open pipe of same


n' n
length produce 4 beats when they are set into (A) (B)
n n'
vibrations simultaneously. If the length of each of them
were twice their initial lengths, the number of beats t t
produced will be n' n'
(A) 2 (B) 4 (C) 1 (D) 8
Sol. n' n
(C) (D)
n n'
t t
Sol.

14. Two trains move towards each other with the


same speed. Speed of sound is 340 ms–1. If the pitch
of the tone of the whistle of one when heard on the
other changes by 9/8 times, then the speed of each
train is :

v v
(A) 2 ms–1 (B) 40 ms–1
(C) 20 ms–1 (D) 100 ms–1

: 0744-2209671, 08003899588 | url : www.motioniitjee.com, : info@motioniitjee.com


Page # 180 SOUND WAVES

16. A small source of sound moves on a circle as Sol.


shown in fig. and an observer is sitting at O. Let at 1,
2 3 be the frequencies heard when the source is at
A, B, and C respectively.
A

C O

19. A stationary sound source ‘s’ of frequency 334 Hz


B
(A) 1 > 2 > 3 (B) 1 = 2 > 3 and a stationary observer ‘O’ are placed near a
(C) 2 > 3 > 1 (D) 1 > 3 > 2 reflecting surface moving away from the source with
Sol. velocity 2m/sec as shown in the figure. If the velocity
of the sound waves in air is V = 330 m/sec, the
apparent frequency of the echo is

o s

17. The frequency changes by 10% as a sound source (A) 332 Hz (B) 326 Hz
approaches a stationary observer with constant speed (C) 334 Hz (D) 330 Hz
vs. What would be the percentage change in frequency Sol.
as the source recedes the observer with the same
speed. Given that vs < v. (v = speed of sound in air)
(A) 14.3% (B) 20%
(C) 10.0% (D) 8.5%
Sol.

20. A sounding body of negligible dimension emitting a


frequency of 150 Hz is dropped from a height. During
its fall under gravity it passes near a balloon moving
up with a constant velocity of 2m/s one second after
it started to fall. The difference in the frequency
observed by the man in balloon just before and just
after crossing the body will be : (Given that - velocity
18. An engine whistling at a constant frequency n0 of sound = 300m/s ; g = 10 m/s2)
and moving with a constant velocity goes past a (A) 12 (B) 6 (C) 8 (D) 4
stationary observer. As the engine crosses him, the Sol
frequency of the sound heard by him changes by a
factor f. The actual difference in the frequencies of
the sound heard by him before and after the engine
crosses him is

1 1  1 f 2 
(B) 2 n 0  f 
2
(A) n0 (1  f )
2  
 1– f  1  1– f 
(C) n 0  1  f  (D) 2 n0  1  f 

Corporate Head Office : Motion Education Pvt. Ltd., 394 - Rajeev Gandhi Nagar, Kota-5 (Raj.)
SOUND WAVES Page # 181

21. Two sound sources each emitting sound of wave- 25. The maximum displaced points are
length  are fixed some distance apart. A listener moves (A) a (B) e (C) g (D) i
with a velocity u along the line joining the two sources. Sol
The number of beats heard by him per second is -
2u u u 2
(A) (B) (C) (D)
  3 u
Sol.

26. The points of maximum compression are


MULTIPLE CORRECT (A) c (B) g (C) e (D) k
The figure represents the instantaneous picture Sol
of a longitudinal harmonic wave travelling along the
negative x-axis. Identify the correct statement(s)
related to the movement of the points shown in the
figure.
y v i
a
b h j
g
o c x 27. The points of maximum rarefaction are
d f k (A) a (B) e (C) g (D) i
e Sol
22. The points moving in the direction of wave are
(A) b (B) c (C) f (D) i
Sol

28. Which of the following graphs is/are correct.


23. The points moving opposite to the direction of
sound in air)

sound in air)2

propagation are
(Velocity of

(Velocity of

(A) a (B) d (C) f (D) j (T=constat)


Sol (A) Parabola (B)
Pressure
Temperature
(Fundamental freq.
wave in a string)

of an organ pipe)
(Velocity of
transverse

(C) Parabola
(D)
Tension Length of organ pipe

Sol

24. The stationary points are


(A) a (B) c (C) g (D) k
Sol

: 0744-2209671, 08003899588 | url : www.motioniitjee.com, : info@motioniitjee.com


Page # 182 SOUND WAVES

29. Which of the following statements are wrong about 31. If a maxima is formed at the detector then, the
the velocity of sound in air : magnitude of wavelength  of the wave produced is
(A) decreases with increases in temperature given by
(B) increases with decrease in temperature R R 2 R
(C) decreases as humidity increases (A) R (B) (C) (D)
2 4 3
(D) independent of density of air. Sol
Sol

30. Two interfering waves have the same wavelength, 32. If the minima is formed at the detector then, the
frequency, and amplitude, They are traveling in the
magnitude of wavelength  of the wave produced is
same direction but are 90° out of phase. Compared to
given by
the individual waves, the resultant wave will have the
3 R 2 R 2 R
same. (A) 2R (B) (C) (D)
(A) amplitude and velocity but different wavelength 2 3 5
(B) amplitude and wavelength but different velocity Sol.
(C) wavelength and velocity but different amplitude
(D) amplitude and frequency but different velocity.
Sol

33. The maximum intensity produced at D is given by


(A) 4I0 (B) 2I0 (C) I0 (D) 3I0
Sol

Question No. 31 to 35 (5 questions)

R
S

D
A narrow tube is bent in the form of a circle of radius
R, as shown in the figure. Two small holes S and D are
made in the tube at the positions right angle to each
34. The maximum value of  to produce a maxima at D
other. A source placed at S generated a wave of
is given by
intensity I0 which is equally divided into two parts :
One part travels along the longer path, while the other R 3 R
travels along the shorter path. Both the part waves (A) R (B) 2R (C) (D)
2 2
meet at the point D where a detector is placed

Corporate Head Office : Motion Education Pvt. Ltd., 394 - Rajeev Gandhi Nagar, Kota-5 (Raj.)
SOUND WAVES Page # 183

Sol Sol

35. The maximum value of  to produce a minima at D 38. A gas is filled in an organ pipe and it is sounded
is given by with an organ pipe in fundamental mode. Choose the
correct statement(s) : (T = constant)
R 3 R
(A) R (B) 2R (C) (D) (A) If gas is changed from H2 to O2, the resonant
2 2 frequency will increase
Sol (B) If gas is changed from O2 to N2, the resonant
frequency will increase
(C) If gas is changed from N2 to He, the resonant
frequency will decrease
(D) If gas is changed from He to CH4, the resonant
frequency will decrease
Sol

36. The second overtone of an open organ pipe A and


a closed pipe B have the same frequency at a given
temperature. If follows that the ratio of the
(A) length of A and B is 4 : 3
(B) fundamental frequencies of A & B is 5 : 6
(C) lengths of B to that of A is 5 : 6 39. A closed organ pipe of length 1.2 m vibrates in its
(D) frequencies of first overtone of A & B is 10 : 9 first overtone mode. The pressure variation is maximum
Sol at :
(A) 0.8m from the open end
(B) 0.4 m from the open end
(C) at the open end
(D) 1.0 m from the open end
Sol

37. Four open organ pipes of different lengths and


different gases at same temperature as shown in figure.
Let fA, fB, fC and fD be their fundamental frequencies
then : [Take  CO 2 = 7/5]
H2 40. For a certain organ pipe three successive
resonance frequencies are observed at 425 Hz, 595
Hz and 765 Hz respectively. If the speed of sound in
O2 N2 air is 340 m/s, then the length of the pipe is
l
CO2
(A) 2.0 m (B) 0.4 m (C) 1.0 m (D) 0.2 m
l/2 2l/3 Sol
l/3

(A) (B) (C) (D)


(A) fA/ fB = 2 (B) fB / fC  72 / 28
(C) fC/fD = 11 / 28 (D) fD /fA = 76 / 11

: 0744-2209671, 08003899588 | url : www.motioniitjee.com, : info@motioniitjee.com


Page # 184 SOUND WAVES

41. In an organ pipe whose one end is at x = 0, the Sol


3 x
pressure is expressed by p  p 0 cos sin 300  t where
2
x is in meter and t in sec. The organ pipe can be
(A) cl osed at one end, open at another with
length = 0.5 m
(B) open at both ends, length = 1m
(C) closed at both ends, length = 2m
(D) closed at one end, open at another with
2
length = m
3
Sol

44. A car moves towards a hill with speed vc. It blows


a horn of frequency f which is heard by an observer
following the car with speed v0. The speed of sound in
air is v.
42. Two whistles A and B each have a frequency of
v
500 Hz. A is stationary and B is moving towards the (A) the wavelength of sound reaching the hill is
right (away from A) at a speed of 50 m/s. An observer f
is between the two whistles moving towards the right v  vc
with a speed of 25 m/s. The velocity of sound in air is (B) the wavelength of sound reaching the hill is
f
350 m/s. Assume there is no wind. Then which of the (C) the beat frequency observed by the observer is
following statements are true.
(A) The apparent frequency of whistle B as heard by  v  v0 
 f
A is 444Hz approximately  v  vc 
(B) The apparent frequency of whistle B as heard by (D) the beat frequency observed by the observer is
the observer is 469 Hz approximately
2v c ( v  v 0 ) f
(C) The difference in the apparent frequencies of A
and B as heard by the observer is 4.5 Hz v 2  v 2c
(D) The apparent frequencies of the whistles of each Sol
other as heard by A and B are the same
Sol

43. A source of sound moves towards an observer


(A) the frequency of the source is increased
(B) the velocity of sound in the medium is increased
(C) the wavelength of sound in the medium towards
the observer is decreased
(D) the amplitude of vibration of the particles is
increased

Corporate Head Office : Motion Education Pvt. Ltd., 394 - Rajeev Gandhi Nagar, Kota-5 (Raj.)
SOUND WAVES Page # 185

Exercise - III (JEE ADVANCED)

1. A sound wave of frequency 100 Hz is travelling in Sol.


air. The speed of sound in air is 350 m/s. (a) By how
much is the phase changed at a given point in 2.5
ms? (b) What is the phase difference at a given in-
stant between two points separated by a distance of
10.0 cm along the direction of propagation?
Sol.

5. In a mixture of gases, the average number of degrees


of freedom per molecule is 6. The rms speed of the
molecules of the gas is c. Find the velocity of sound
in the gas.
Sol.

2. The equation of a travelling sound wave is y = 6.0


sin (600 t - 1.8 x) where y is measured in 10-5 m, t in
second and x in metre. (a) Find the ratio of the dis-
placement amplitude of the particles to the wave-
length of the wave. (b) Find the ratio of the velocity
amplitude of the particles to the wave speed.
Sol.

6. Find the intensity of sound wave whose frequency


is 250 Hz. The displacement amplitude of particles of
the medium at this position is 1 × 10–8 m. The density
of the medium is 1 kg/m3, bulk modulus of elasticity of
the medium is 400 N/m2.
Sol.

3. A man stands before a large wall at a distance of


100.0 m and claps his hands at regular intervals. In
such way that echo of a clap merges with the next
clap. If he has to clap 5 times during every 3 sec-
onds, find the velocity of sound in air.
Sol.

7. Two identical sounds A and B reach a point in the


same phase. The resultant sound is C. The loudness
of C is n dB higher than the loudness of A. Find the
value of n.
Sol.

4. Calculate the speed of sound in oxygen from the


following data. The mass of 22.4 litre of oxygen at
STP (T = 273 K and p= 1.0 x 105 N/m2) is 32 g, the molar
heat capacity of oxygen at constant volume is Cv = 2.5
R and that at constant pressure is Cp = 3.5 R.

: 0744-2209671, 08003899588 | url : www.motioniitjee.com, : info@motioniitjee.com


Page # 186 SOUND WAVES

8. The loudness level at a distance R from a long 11. A source of sound S and a detector D are placed
linear source of sound is found to be 40dB. At this at some distance from one another. A big cardboard is
point, the amplitude of oscillations of air molecules is placed near the detector and perpendicular to the
0.01 cm. Then find the loudness level & amplitude at line SD as shown in figure. It is gradually moved away
a point located at a distance ‘10R’ from the source. and it is found that the intensity changes from a maxi-
Sol. mum to a minimum as the board is moved through a
distance of 20 cm. Find the frequency of the sound
emitted. Velocity of sound in air is 336 m/s.

S D

Sol.
9. Two point sound sources A and B each of power
25 W and frequency 850 Hz are 1 m apart.
(a) Determine the phase difference between the waves
emitting from A and B received by detector D as in
figure. B
(b) Also determine the intensity of the resultant
sound wave as recorded by detector D . Velocity of sound
= 340 m/s.
12. Sound of wavelength  passes through a Quincke’s
A 2.4m
D tube, which is adjusted to give a maximum intensity
90° I0. Find the distance through the sliding tube should
be moved to give an intensity I0/2.
Sol.
1m

B
Sol.

13. The stationary wave y = 2a sin kx cos w t in a


closed organ pipe is the result of the superposition
of y = a sin (wt – kx) &
Sol.

10. Two identical loudspeakers are located at points


A & B, 2 m apart. The loudspeakers are driven by the
same amplifier. A small detector is moved out from
point B along a line perpendicular to the line connect-
ing A & B. Taking speed of sound in air as 332 m/s.
Find the frequency below which there will be no posi-
tion along the line BC at which destructive interfer- 14. The equation of a longitudinal standing wave due
ence occurs. to superposition of the progressive waves produced
A by Two sources of sound is s = –20 sin 10 x sin 100
2m t where s is the displacement from mean position
measured in mm, x is in metres and t in seconds. The
B specific gravity of the medium is 10-3. Density of wa-
ter = 103 kg/m3. Find:
Sol. (a) Wavelength, frequency and velocity of the pro-
gressive waves.
(b) Bulk modulus of the medium and the pressure am-
plitude .
(c) Minimum distance between pressure antinode and
a displacement antinode.
(d) Intensity at the displacement nodes.

Corporate Head Office : Motion Education Pvt. Ltd., 394 - Rajeev Gandhi Nagar, Kota-5 (Raj.)
SOUND WAVES Page # 187

Sol. 18. The first overtone of a pipe closed at one end


resonates with the third harmonic of a string fixed at
its ends. The ratio of the speed of sound to the speed
of transverse wave travelling on the string is 2 : 1.
Find the ratio of the length of pipe to the length of
string.
Sol.

15. A tube 1.0 m long is closed at one end. A wire of


length 0.3 m and mass 1 × 10–2 kg is stretched be-
tween two fixed ends and is placed near the open
end. When the wire is plucked at its mid point the air
column resonates in its 1st overtone. Find the tension
in the wire if it vibrates in its fundamental mode.
[Vsound = 330 m/s]
Sol.
19. In a resonance-column experiment, a long tube,
open at the top, is clamped vertically. By a separate
device, water level inside the tube can be moved up
or down. The section of the tube from the open end
to the water level act as a closed organ pipe. A vibrating
tuning fork is held above the open end, first and the
second resonances occur when the water level is 24.1
cm and 74.1 cm respectively below the open end.
16. A closed organ pipe of length  = 100 cm is cut
Find the diameter of the tube. [Hint : end correction
into two unequal pieces. The fundamental frequency
is 0.3d]
of the new closed organ pipe piece is found to be
Sol.
same as the frequency of first overtone of the open
organ pipe piece. Determine the length of the two
pieces and the fundamental tone of the open pipe
piece. Take velocity of sound = 320 m/s.
Sol.

20. An open organ pipe filled with air has a fundamental


17. Find the number of possible natural oscillations of frequency 500 Hz. The first harmonic of another organ
air column in a pipe whose frequencies lie below v0 = pipe closed at one end and filled with carbon dioxide
1250 Hz. The length of the pipe is  = 85 cm. The has the same frequency as that of the first harmonic
velocity of sound is v = 340 m/s. of the open organ pipe. Calculate the length of each
Consider the two cases: pipe. Assume that the velocity of sound in air and in
(a) the pipe is closed from one end carbondioxide to be 330 and 264 m/s respectively.
(b) the pipe is opened from both ends. Sol.
The open ends of the pipe are assumed to be the
antinodes of displacement.
Sol.

: 0744-2209671, 08003899588 | url : www.motioniitjee.com, : info@motioniitjee.com


Page # 188 SOUND WAVES

21. Two identical piano wires have a fundamental fre- 24. Tuning fork A when sounded with a tuning fork B
quency of 600 vib/sec, when kept under the same of frequency 480 Hz gives 5 beats per second. When
tension. What fractional increase in the tension of the prongs of A are loaded with wax, it gives 3 beats
one wire will lead to the occurrence of six beats per per second. Find the original frequency of A.
second when both wires vibrate simultaneously. Sol.
Sol.

22. A metal wire of diameter 1 mm, is held on two


knife edges separated by a distance of 50 cm. The
tension in the wire is 100 N. The wire vibrating in its
25. A, B and C are three tuning forks. Frequency of A
fundamental frequency and a Vibrating tuning fork to-
is 350 Hz. Beats produced by A and B are 5 per second
gether produces 5 beats per sec. The tension in the
and by B and C are 4 per second. When a wax in put
wire is then reduced to 81 N. When the two are ex-
on A beat frequency between A and B is 2Hz and
cited, beats are again at the same rate. Calculate
between A and C is 6Hz. Then, find the frequency of
(a) the frequency of the fork
B and C respectively.
(b) the density of the material of the wire.
Sol.
Sol.

23. Two stationary sources A and B are sounding notes


of frequency 680 Hz. An observer moves from A to B
with a constant velocity u. If the speed of sound is 26. S, O & W represent source of sound (of frequency
340 ms–1, what must be the value of u so that he f), observer & wall respectively. V0, Vs, VD, V are ve-
hears 10 beats per second locity of observer, source, wall & sound (in still air)
Sol. respectively. VW is the velocity of wind. They are mov-
ing as shown. Find

vw vD

V0
vs

(i) The wavelength of the waves coming towards the


observer from source.
(ii) The wavelength of the waves incident on the wall.
(iii) The wavelength of the waves coming towards
observer from the wall.
(iv) Frequency of the waves (as detected by O) com-
ing from wall after reflection.

Corporate Head Office : Motion Education Pvt. Ltd., 394 - Rajeev Gandhi Nagar, Kota-5 (Raj.)
SOUND WAVES Page # 189

Sol. 29. The loudness level at a distance R from a long


linear source of sound is found to be 40dB. At this
point, the amplitude of oscillations of air molecules is
0.01 cm. Then find the loudness level & amplitude at
a point located at a distance ‘10R’ from the source.
Sol.

27. S is source R is receiver. R and S are at rest. Fre-


quency of sound from S is f. Find the beat frequency
registered by R. Velocity of sound is v.
u
(const.)

S R 30. A fixed source of sound emitting a certain frequency


appears as fa when the observer is approaching the
source with speed v and frequency fr when the
observer recedes from the source with the same
Sol. speed. Find the frequency of the source.
Sol.

31. The first overtone of an open organ pipe beats


with the first overtone of a closed organ pipe with a
28. A car moving towards a vertical wall sounds a beat frequency of 2.2 Hz. The fundamental frequency
horn. The driver hears that the sound of the horn of the closed organ pipe is 110 Hz. Find the lengths of
reflected from the cliff has a pitch half-octave higher the pipes. Velocity of sound = 330 m/s
than the actual sound. Find the ratio of the velocity Sol.
of the car and the velocity of sound.
Sol.

: 0744-2209671, 08003899588 | url : www.motioniitjee.com, : info@motioniitjee.com


Page # 190 SOUND WAVES

1. The displacement of the medium in a sound wave is minimum frequency such that there is node (displace-
given by the equation ; y1 = A cos(ax + bt) where A, ment) at B and antinode (displacement) at A. Find
a & b are positive constants. The wave is reflected (i) the fundamental frequency of air column.
by an obstacle situated at x = 0. The intensity of the (ii) Find the frequency of tuning fork.
reflected wave is 0.64 times that of the incident wave. Assume velocity of sound in the first and second cham-
(a) what are the wavelength & frequency of the ber be 1100 m/s and 300 m/s respectively. Velocity of
incident wave. sound in air 330 m/s.
(b) write the equation for the reflected wave. A B C
(c) in the resultant wave formed after reflection, find 1 2
the maximum & minimum values of the particle speeds
in the medium. v1= 1100 m/s
v2 = 300 m/s

2. (a) A standing wave in second overtone is main-


tained in a open organ pipe of length l. The distance
0.5 m 1.0 m
between consecutive displacement node and pres-
sure node is ______________. 5. A source emits sound waves of frequency 1000 Hz.
(b) Two consecutive overtones produced by a narrow The source moves to the right with a speed of 32 m/
air column closed at one end and open at the other s relative to ground. On the right a reflecting surface
are 750 Hz and 1050 Hz. Then the fundamental fre- moves towards left with a speed of 64 m/s relative to
quency from the column is ______________. the ground. The speed of sound in air is 332 m/s. Find
(c) A standing wave of frequency 1100 Hz in a column (a) the wavelength of sound in air incident on
of methane at 20°C produces nodes that are 20 cm reflecting surface
apart. What is the ratio of the heat capacity at con- (b) the number of waves arriving per second which
stant pressure to that at constant volume. meet the reflecting surface.
(c) the speed of reflected waves.
3. Two speakers are driven by the same oscillator (d) the wavelength of reflected waves.
with frequency of 200 Hz. They are located 4 m apart
on a vertical pole. A man walks straight towards the 6. A supersonic jet plane moves parallel to the ground
lower speaker in a direction perpendicular to the pole, at speed v = 0.75 mach (1 mach = speed of sound).
as shown in figure. The frequency of its engine sound is 0 = 2kHz and
the height of the jat plane is h = 1.5 km. At some
instant an observer on the ground hears a sound of
frequency  = 20, Find the instant prior to the in-
stant of hearing when the sound wave received by
the observer was emitted by the jet plane. Velocity of
sound wave in the condition of observer = 340 m/s.
(a) How many times will he hear a minimum in sound
intensity, and 7. A train of length l is moving with a constant speed
(b) how far is he from the pole at these moments ? v along a circular track of radius R, The engine of the
Take the speed of sound to be 330 m/s, and ignore train emits a whistle of frequency f. Find the fre-
any sound reflections coming off the ground. quency heard by a guard at the rear end of the train.
Make suitable assumption.
4. A cylinder ABC consists of two chambers 1 and 2
which contains two different gases. The wall C is rigid 8. A bullet travels horizontally at 660 m/s at a height
but the walls A and B are thin diaphragms. A vibrating of 5 m from a man. How far is the bullet from the man
tuning fork approaches the wall A with velocity u = 30 when he hears its whistle? Velocity of sound in air =
m/s and air columns in chamber 1 and 2 vibrates with 340 m/s.

Corporate Head Office : Motion Education Pvt. Ltd., 394 - Rajeev Gandhi Nagar, Kota-5 (Raj.)
SOUND WAVES Page # 191

Exercise - IV PREVIOUS YEAR QUESTIONS

LEVEL - I JEE MAIN


1. A tuning fork arrangement (pair) produces 4 beats Sol.
per second with one fork of frequency 288 cps. A
little wax is placed on the unknown fork and it then
produces 2 beats per second. The frequency of the
unknown fork is [AIEEE 2002]
(a) 286 cps (b) 292 cps
(c) 294 cps (d) 288 cps
Sol.

4. A tuning fork of known frequency 256 Hz makes 5


beats per second with the vibrating string of a piano.
The beat frequency decreases to 2 beats per second
when the tension in the piano string is slightly
increased. The frequency of the piano string before
increasing the tension was [AIEEE 2003]
(a) (256 + 2) Hz (b) (256 - 2) Hz
(c) (256 - 5) Hz (d) (256 + 5) Hz
Sol.

2. Tube A has both ends open while tube B has one


end closed, otherwise they are identical. The ratio of
fundamental frequency of tubes A and B is
[AIEEE 2002]
(a) 1 : 2 (b) 1 : 4
(c) 2 : 1 (d) 4 : 1
Sol.

5. An observer moves towards a stationary source of


sound, with a velocity one-fifth of the velocity of
sound. What is the percentage increase in the
apparent frequency ? [AIEEE 2005]
(a) zero (b) 0.5% (c) 5% (d) 20%
Sol.

3. When temperature increases, the frequency of a


tuning fork [AIEEE 2002]
(a) increases (b) decreases
(c) remains same
(d) increases or decreases depending on the material

: 0744-2209671, 08003899588 | url : www.motioniitjee.com, : info@motioniitjee.com


Page # 192 SOUND WAVES

6. When two tuning forks (forks 1 and forks 2) are 9. While measuring the speed of sound by performing
sounded simultaneously, 4 beats per second are heard. a resonance column experiment, a student gets the
Now, some tape is attached on the prong of the fork first resonance condition at a column length of 18 cm
2. When the tuning forks are sounded again, 6 beats during winter. Repeating the same experiment during
per second are heard. If the frequency of fork is 200 summer she measures the column length to be x cm
Hz, then what was the original frequency of fork 2 ? for the second resonance. Then [AIEEE 2008]
[AIEEE 2005] (a) 18 > x (b) x > 54
(a) 200 Hz (b) 202 Hz (c) 54 > x > 36 (d) 36 > x > 18
(c) 196 Hz (d) 204 Hz Sol.
Sol.

7. A whistle producing sound waves of frequencies


9500 Hz and above is approaching a stationary person
with speed v ms–1. The velocity of sound in air is 300 10. The speed of sound in oxygen (O2) at a certain
ms–1 . If the person can hear frequencies upto a temperature is 460 ms–1. The speed of sound in helium
maximum of 10,000 Hz, the maximum value of v upto (He) at the same temperature will be (assume both
which he can hear the whistle is [AIEEE 2006] gases to be ideal) [AIEEE 2008]
(a) 1420 ms–1 (b) 500 ms–1
(a) 15 2 ms1 (b) 15 / 2 ms1 (c) 650 ms–1 (d) 330 ms–1
(c) 15 ms 1 (d) 30 ms 1 Sol.
Sol.

11. Three sound waves of equal amplitudes have


8. A sound absorber attenuates the sound level by frequencies (v - 1), v, (v + 1). They superpose to
20 db. The intensity decreases by a factor of give beat. The number of beats produced per second
[AIEEE 2007] will be [AIEEE 2009]
(a) 1000 (b) 10000 (a) 4 (b) 3 (c) 2 (d) 1
(c) 10 (d) 100 Sol.
Sol.

Corporate Head Office : Motion Education Pvt. Ltd., 394 - Rajeev Gandhi Nagar, Kota-5 (Raj.)
SOUND WAVES Page # 193

12. A motor cycle starts from rest & accelerates along 14. A sonometer wire of length 1.5 m is made of steel.
a straight path at 2 ms–2. At the starting point of the The tension in it produces an elastic strain of 1%.
motor cycle there is a stationary electric sire. HOw far What is the fundamental frequency of steel if density
has the motor cycle gone when the driver hears the and elasticity of steel are 7.7 × 103 kg/m3 and 2.2 ×
frequency of the siren at 94% of its value when the 1011 N/m2 respectively? [JEE MAIN 2013]
motor cycle was at rest ? (speed of sound = 330 ms–1) (A) 200.5 Hz (B) 770 Hz
[AIEEE 2009] (C) 188.5 Hz (D) 178.2 Hz
(a) 49 m (b) 98 m (c) 147 m (d) 196 m Sol.
Sol.

13. A cylindrical tube, open at both ends, has a


fundamental frequency, f, in air. The tube is dipped
vertically in water so that half of it is in water. The
fundamental frequency of the air-column is now
[AIEEE 2012]
Sol.
f 3f
(a) f (b) (c) (d) 2f
2 4

: 0744-2209671, 08003899588 | url : www.motioniitjee.com, : info@motioniitjee.com


Page # 194 SOUND WAVES

LEVEL - II JEE ADVANCED

1. A siren placed at a railway platform is emitting sound Sol.


of frequency 5 kHz. A passenger sitting in a moving
train A records a frequency of 5.5 kHz while the train
approaches the siren. During his return journey in a
different train B he records a frequency of 6.0 kHz.
while approaching the same siren. The ratio of the
velocity of train B to that of train A is
[JEE-2002(Scr), 3]
(A) 242/252 (B) 2
(C) 5/6 (D) 11/6
Sol.

4. A cylindrical tube when sounded with a tuning fork


gives, first resonance when length of air column is 0.1
and gives second resonance when the length of air
column is 0.35 m. Then end correction is
[JEE-2003 (Scr)]
(A) 0.025 m (B) 0.020 m
2. Two narrow cylindrical pipes A and B have the same (C) 0.018 m (D) 0.012 m
length. Pipe A is open at both ends and is filled with a Sol.
monoatomic gas of malar mass MA. Pipe B is open at
one end and closed at the other end, and is filled with
a diatomic gas of molar mass MB. Both gases are at
the same temperature.
(a) If the frequency of the second harmonic of the
fundamental mode in pipe A is equal to the frequency
of the third harmonic of the fundamental mode in pipe
B, determine the value of MA/MB.
(b) Now the open end of pipe B is also closed (so that
the pipe is closed at both ends). Find the ratio of the
fundamental frequency in pipe A to that in pipe B.
[JEE-2002, 3 + 2]
Sol.
5. A tuning fork of frequency 480 Hz resonates with a
tube closed at one end of length, 16cm and diameter
5 cm in fundamental mode. Calculate velocity of sound
in air. [JEE-2003]
Sol

3. A police van moving with velocity 22 m/s and


emitting sound of frequency 176 Hz, follows a motor
cycle in turn is moving towards a stationary car and
away from the police van. The stationary car is
emitting frequency 165 Hz. If motorcyclist does not
hear any beats then his velocity is
[JEE-2003 (Scr)]
(A) 22 m/s (B) 24 m/s
(C) 20 m/s (D) 18 m/s

Corporate Head Office : Motion Education Pvt. Ltd., 394 - Rajeev Gandhi Nagar, Kota-5 (Raj.)
SOUND WAVES Page # 195

6. A closed organ pipe of length L and an open organ Sol.


pipe contain gases of densities 1 and 2 respectively.
The compressibility of gases are equal in both the
pipes. Both the pipes are vibrating in their first overtone
with same frequency. The length of the open organ
pipe is [JEE-2004 (Scr)]
L 4L
(A) (B)
3 3
4 L 1 4 L 2
(C) 3  (D) 3 
2 1
9. A whistling train approaches a junction. An observer
Sol.
standing at junction observers the frequency to be
2.2 KHz and 1.8 KHz of the approaching and the
receding train. Find the speed of the train (speed
sound = 300 m/s). [JEE-2005]
Sol.

Passage (Q.10 to 12)


Two plane harmonic sound waves are expressed by
the equations.
7. A source of sound of frequency 600 Hz is placed y1 (x, t) = A cos (x – 100 t)
inside water. The speed of sound in water is 1500 m/s y2 (x, t) = A cos (0.46 x – 92 t)
and in air it is 300 m/s. The frequency of sound (All parameters are in MKS) [JEE 2006]
recorded by an observer who is standing in air is
[JEE-2004(Scr)] 10. How many times does an obsever hear maximum
(A) 200 Hz (B) 3000 HZ intensity in one second?
(C) 120 Hz (D) 600 Hz (A) 4 (B) 10 (C) 6 (D) 8
Sol. Sol.

11. What is the speed of the second?


(A) 200 m/s (B) 180 m/s
(C) 192 m/s (D) 96 m/s
Sol.

12. At x = 0 how many times the amplitude of y 1


+ y2 is zero in one second?
8. In a resonance column method, resonance occurs (A) 192 (B) 48 (C) 100 (D) 96
at two successive level of l1 = 30.7 cm and l2 = 63.2 Sol.
cm using a tuning fork of f = 512 Hz. What is the
maximum error in measuring speed of sound using
relations v = f  &  = 2(l2 – l1) [JEE-2005 (Sc)]
(A) 256 cm/sec (B) 92 cm/sec
(C) 128 cm/sec (D) 102.4 cm/sec

: 0744-2209671, 08003899588 | url : www.motioniitjee.com, : info@motioniitjee.com


Page # 196 SOUND WAVES

Passage (Q.13 to 15) 15. The spread of frequency as observed by the


Two trains A and B are moving with speeds 20 m/ passengers in train B is :
s and 30 m/s respectively in the same direction on (A) 310 Hz (B) 330 Hz (C) 350 Hz (D) 290 Hz
the same straight track, with B ahead of A. The
engines are at the front ends. The engine of train
A blows a long whistle. [JEE 2007]
Intensity

f1 f2 Frequency

Assume that the sound of the whistle is composed


of components varying in frequency from f1 = 800 16. A vibrating string of certain length l under a
Hz to f2 = 1120 Hz, as shown in the figure. The tension T resonates with a mode corresponding to
spread in the frequency (highest frequency–lowest the first overtone (third harmonic) of an air column
frequency) is thus 320 Hz. The speed of sound in of length 75 cm inside a tube closed at one end.
air is 340 m/s. The string also generates 4 beats/s when excited
along with a tuning fork of frequency n. Now when
13. The speed of sound of the whistle is : the tension of the string is slightly increased the
(A) 340 m/s for passengers in A and 310 m/s for number of beats reduces to 2 per second. Assuming
passengers in B. the velocity of sound in air to be 340 m/s, the
(B) 360 m/s for passengers in A and 310 m/s for frequency n of the tuning fork in Hz is :
passengers in B. [JEE 2008]
(C) 310 m/s for passengers in A and 360 m/s for (A) 344 (B) 336 (C) 117.3 (D) 109.3
passengers in B. Sol.
(D) 340 m/s for passengers in both the trains.
Sol.

14. The distribution of the sound intensity of the


whistle as observed by the passengers in train A
is best represented by :
17. A student performed the experiment to measure
Intensity

Intensity

the speed of sound in air using resonance aircolumn


method. Two resonances in the aircolumn ware
(A) (B) obtained by lowering the water level. The resonance
with the shorter aircolumn is the first resonance and
f1 f2 Frequency f1 f2 Frequency that with the longer aircolumn i s the second
resonance. Then, [JEE 2009]
Intensity

Intensity

(A) the intensity of the sound heard at the first


resonance was more than that at the second
(C) (D) resonance
(B) the prongs of the tuning fork were kept in a
f1 f2 Frequency f1 f2 Frequency
horizontal plane above the resonance tube
Sol.
(C) the amplitude of vibration of the ends of the prongs
is typically around 1 cm
(D) the length of the aircolumn at the first resonance
1
was somewhat shorter than the of the wavelength
4
of the sound in air

Corporate Head Office : Motion Education Pvt. Ltd., 394 - Rajeev Gandhi Nagar, Kota-5 (Raj.)
SOUND WAVES Page # 197

Sol. 20. Column I shows four systems, each of the same


length L, for producing standing waves. The lowest
possible natural frquency of a system is called its
fundamental frequency, whose wavelength is denoted
as f. Match each system with statement given in
column II describing the nature and wavelength of
the standing waves. [JEE 2011]
Column I Column II
(A) Pipe closed at one end (P) Longitudinal waves

O L
18. A stationary source is emitting sound at a fixed (B) Pipe open at both ends (Q) Transverse waves
frequency f 0 , whi ch i s refl e cted by two cars
approaching the source. The difference between the
frequencies of sound reflected from the cars is 1.2 O L
%of f0. What is the difference in the speeds of the (C) Stretched wire clamped (R) f = L
cars (in km per hour) to the nearest integer ? The at both ends
cars are moving at constant speeds much smaller than
the speed of sound which is 330 ms–1. [JEE 2010]
O L
Sol.
(D) Stretched wire clamped (S) f = 2L
at both ends and at mid point

O L (T) f = 4L
L/2
Sol.

19. A hollow pipe of length 0.8 m is closed at one


end. At its open end a 0.5 m long uniform string
is vibrating in its second harmonic and it resonates
with the fundamental frequency of the pipe. If the
tension in the wire is 50 N and the speed of sound
is 320 ms–1, the mass of the string is :
[JEE 2010]
(A) 5 g (B) 10 g (C) 20 g (D) 40 g
Sol.

21. A police car with a siren of frequency 8 kHz is


moving with uniform velocity 36 km/hr towards a tall
building which reflects the sound waves. The speed
of sound in air is 320 m/s. The frequency of the siren
heard by the car driver is [JEE 2011]
(A) 8.50 kHz (B) 8.25 kHz
(C) 7.75 kHz (D) 7.50 kHz

: 0744-2209671, 08003899588 | url : www.motioniitjee.com, : info@motioniitjee.com


Page # 198 SOUND WAVES

Sol. 23. Two vehicles, each moving with speed u on the


same horizontal straight road, are approaching each
other. Wind blows along the road with velocity w. One
of these vehicles blows a whistle of frequency f1. An
observer in the other vehicle hears the frequency of
the whistle of be f2. The speed of sound in still air is V.
The correct statement (s) is (are) [JEE 2013]
(A) If the wind blows from the observer to the source,
f2 > f1.
(B) If the wind blows from the source to the observer,
f2 > f1.
(C) If the wind blows from observer to the source, f2
< f1.
(D) If the wind blows from the source to the observer,
f2 < f1.
Sol.

22. A person blows into open-end of a long pipe. As a


result, a high-pressure pulse of air travels down the
pipe. When this pulse reaches the other end of the
pipe,
(A) a high-pressure pulse starts travelling up the pipe,
if the other end of the pipe is open.
(B) a low-pressure pulse starts travelling up the pipe,
if the other end of the pipe is open.
(C) a low-pressure pulse starts travelling up the pipe,
if the other end of the pipe is closed.
(D) a high-pressure pulse starts travelling up the pipe,
if the other end of the pipe is closed. [JEE 2012]
Sol.

Corporate Head Office : Motion Education Pvt. Ltd., 394 - Rajeev Gandhi Nagar, Kota-5 (Raj.)
SOUND WAVES Page # 199

Exercise - I OBJECTIVE PROBLEMS (JEE MAIN)

1. C 2. A 3. D 4. D 5. B 6. B 7. D
8. D 9. D 10. D 11. B 12. B 13. B 14. D
15. A 16. A 17. B 18. D 19. B 20. C 21. B
22. C 23. B 24. D 25. A 26. B 27. B 28. D
29. B 30. C 31. C 32. C 33. A 34. A

Exercise - II

1. A 2. A 3. A 4. A 5. B 6. C 7. D
8. C 9. C 10. D 11. D 12. B 13. A 14. C
15. B 16. D 17. D 18. B 19. D 20. A 21. A
22. B 23. C 24. A 25. A,B,D 26. A,D 27. C 28. B,C
29. A,B,C,D 30. C 31. A,B,C 32. A,B,D 33. B 34. A
35. B 36. C,D 37. C 38. B,D 39. B 40. C 41. C
42. C 43. C 44. B,D

Exercise - III (JEE ADVANCED)

 2
1. (a) (b) 2. (a) 1.7 × 10–5 (b) 1.08 × 10–4 3. 333 m/s 4. 310 m/s
2 35

 2  10 –9
5. 2c/3 6. W/m2 7. 6 8. 30 dB, 10 10 mm
4

9. (a) p (b) I  ( IA – IB )2  (25 / 312)2 10. 83 Hz 11. 420 Hz 12. /8

13. a [sin (kx + t) + 2 sin (kx – t)]


14. (a) f = 50 Hz,  = 0.2 m, v = 10 ms–1 (b) Pm = 62.8 Nm–2 = 20  Nm–2, B = 100 Nm–2
(c) /4 = 0.05 m (d) I = 20 2  200 wm–2
15. 735 N 16. 20, 80 cm, 200 Hz

v v
17. (a) vn = (2n + 1); six oscillations (b) vn = (n+1), also six oscillations ; Here n = 0, 1, 2, .....
4 2
18. 1:1 19. 3 cm 20. 33 cm and 13.2 cm 21. 2 %
40
22. (a) 95% (b)  10 3 kg / m 3 23. 2.5 ms–1 24. 485 Hz

25. 345, 341 or 349 Hz 26. (i) (V – Vw + Vs) / f (ii) (V + Vw – Vs) / f
(iii) (V – Vw – VD) / fr ; where fr = (V + Vw + V0/v + Vw – Vs) f (iv) (V – Vw – Vo/ V – Vw – VD) fr
2uf fr  fa
27. fb  28. 1 : 5 29. 30 dB, 10 10 mm 30.
vu 2
150
31. Lc = 0.75 m, Lo = m, 1.006 m
151

: 0744-2209671, 08003899588 | url : www.motioniitjee.com, : info@motioniitjee.com


Page # 200 SOUND WAVES

LEVEL - II JEE ADVANCED

1. (a) 2 /a, b/2, (b) y2 = ± 0.8 A cos (ax – bt), (c) max. = 1.8 bA, min = 0,
2. (a) l/6 ; (b) 150 Hz; (c) 1.28 3. (a) 2; (b) 9.28 m and 1.99 m
4. 1650 Hz, 1500 Hz 5. (a) 0.3 m, (b) 1320, (c) 332 m/s, (d) 0.2 m
6. 5.9 sec 7. f 8. 9.7 m

Exercise - IV PREVIOUS YEAR QUESTIONS

LEVEL - I JEE MAIN

1. B 2. C 3. B 4. C 5. D 6. C 7. C
8. D 9. B 10. A 11. C 12. B 13. A 14. D

LEVEL - II JEE ADVANCED

1. B 2. (a) 2.116, (b) 3/4 3. A 4. A 5. 336 m/s


6. C 7. D 8. D 9. Vs = 30 m/s
10. A 11. A 12. C 13. B 14. A 15. A
16. A 17. A,C,D 18. 7 19. B 20. A  PT ; B  PS ; C  QS ; D  QR
21. A 22. B,D 23. A,B

Corporate Head Office : Motion Education Pvt. Ltd., 394 - Rajeev Gandhi Nagar, Kota-5 (Raj.)

Вам также может понравиться